RR - Pediatrics

Réussis tes devoirs et examens dès maintenant avec Quizwiz!

One Step Further Question: What is the most common cause of neonatal hemorrhage?

Answer: Failure to administer vitamin K in the immediate postpartum period (associated with home births).

One Step Further Question: What is the upper limit of normal for a QTc interval?

Answer: For men, the upper level of QTc interval is 450 msec. For women, the upper level of QTc interval is 470 msec.

One Step Further Question: Besides canned goods, how do infants acquire botulism?

Answer: From ingestion of honey, home grown vegetables, or soil.

One Step Further Question: What is the most common pathogen implicated in bacterial vulvovaginitis?

Answer: Group A β-hemolytic streptococcus.

One Step Further Question: What is the major complication of nephrotic syndrome?

Answer: Infection

One Step Further Question: Does neonatal ophthalmic erythromycin prevent infection with Chlamydia trachomatis?

Answer: No.

One Step Further Question: What drug can be administered to reopen a recently closed ductous arteriosus?

Answer: Prostaglandin E1.

One Step Further Question: What vitamin should be supplemented in infants who are exclusively breast fed?

Answer: Vitamin D.

One Step Further Question: What test is diagnostic for pyloric stenosis?

Answer: Ultrasound.

One Step Further Question: What are the side effects of amphotericin B?

Answer: Renal injury, anaphylaxis, hypotension, fever, and headaches.

One Step Further Question: Which form of supplemental iron is most easily absorbed?

Answer: Ferrous sulfate.

One Step Further Question: What is the most common manifestation of renal disease in patients diagnosed with Henoch-Schönlein Purpura?

Answer: Hematuria.

One Step Further Question: What signs on chest X-ray signify ingestion of a button battery?

Answer: The double-rim or halo-effect.

One Step Further Question: At what age is the physiologic nadir for hemoglobin?

Answer: The lowest normal hemoglobin in a child occurs at three months of age, with a normal hemoglobin of 12.0 g/dL

One Step Further Question: Which virus is the most common cause of croup?

Answer: Parainfluenza virus.

One Step Further Question: What is the drug of choice to close a patent ductus arteriosus in premature infants?

Answer: Indomethacin.

One Step Further Question: What is the age distribution of osteosarcoma?

Answer: Bimodal (peaks in adolescence and adults over 65 years).

One Step Further Question: What is a common complication of Meckel's diverticulum?

Answer: Intestinal obstruction caused by intussusceptions or volvulus

One Step Further Question: What specific gastrointestinal disorder is associated with Henoch-Schönlein purpura?

Answer: Intussusception.

One Step Further Question: Which anemia often precedes lead toxicity?

Answer: Iron deficiency anemia due to its association with pica.

One Step Further Question: Which chromosome abnormality is Tetralogy of Fallot associated with?

Answer: It is associated with chromosome 22 deletions and DiGeorge syndrome.

One Step Further Question: What is the name of the spirochete that causes Lyme disease?

Answer: Borrelia burgdorferi, which is transmitted by the Ixodes tick.

One Step Further Question: What is the natural history of croup?

Answer: Stridor typically resolves after 48 hours, after which typical upper respiratory symptoms predominate

One Step Further Question: What diameter of induration constitutes a positive tuberculin skin test in children less than 4 years of age without tuberculosis risk factors?

Answer: 10 mm.

One Step Further Question: What procedure is used to treat paraphimosis when minimally invasive techniques have failed?

Answer: Dorsal slit procedure.

One Step Further Question: What condition should be considered in individuals diagnosed with a right sided varicocele?

Answer: Inferior vena cava thrombosis.

One Step Further Question: What viral infection is known to greatly increase the susceptibility to bacterial tracheitis?

Answer: Influenza A.

One Step Further Question: What is the most common nutritional deficiency in children?

Answer: Iron deficiency.

One Step Further Question: What is the treatment of choice for chronic otitis media?

Answer: Tympanostomy tubes.

One Step Further Question: What kind of prognostic indicator is the presence of B symptoms in Hodgkin lymphoma?

Answer: Poor outcome.

One Step Further Question: What reflex needs to disappear in order to allow the infant to roll over?

Answer: The asymmetric tonic neck reflex that is present at birth and disappears at approximately 3-4 months to allow for rolling.

One Step Further Question: What disease is characterized by the absence of UGT activity?

Answer: Crigler-Najjar Type I.

One Step Further Question: What is the initial treatment for prolonged bleeding in patients with von Willebrand disease?

Answer: Desmopressin (DDAVP), which causes release of VWF from cells.

A previously healthy 3-week-old male born full-term presents with sudden onset bilious emesis. There appears to be abdominal tenderness and distention on exam. Which of the following is the most accurate statement regarding the suspected diagnosis? Imaging will often show air in the biliary tree It is associated with maternal use of erythromycin during late pregnancy and breastfeeding The diagnosis is confirmed on upper GI series with failure of the duodenal-jejunal junction to cross midline The primary metabolic abnormality is hypochloremic metabolic alkalosis

Correct Answer ( C ) Explanation: Intestinal malrotation refers to a spectrum of congenital anomalies which involve failure to develop normal rotation of the intestinal tract. The formation of normal bowel in the embryo involves a rotation of elongated intestine into the abdominal cavity. The proximal small intestine develops a C-shaped contour, with the duodenum fixing to the left of the midline at the ligament of Treitz. The cecum undergoes counter-clockwise rotation to end up in the right lower abdomen. Incomplete rotation results in inadequate fixation of the intestinal mesentery. Bands may be formed which cause incomplete intestinal obstruction. If there is inadequate fixation of the small bowel, the intestine can twist on the axis of the superior mesenteric artery. The majority of patients present within the first year of life and of those, most present within the first month of life with symptoms of obstruction. The classic presentation is an otherwise healthy infant who develops sudden onset bilious vomiting. Patients may also present with hematochezia which indicates bowel ischemia and has a poorer prognosis. Intestinal malrotation is usually diagnosed with an upper GI series and confirmed by observing failure of the duodenal-jejunal junction to cross midline. Contrast studies often show a spiral or corkscrew appearance of the distal duodenum in cases of volvulus. Management of uncomplicated malrotation with or without volvulus is typically laparoscopic surgery (the Ladd's procedure), although patients with significant bowel ischemia may require an open approach. Recurrent volvulus after surgical repair is rare, but possible. After the first year of life, patients with malrotation present with more varied symptoms such as intermittent abdominal pain, diarrhea, and bilious or non-bilious emesis, and bilious vomiting is not commonly the predominant symptom. As a result patients are often misdiagnosed as having simple gastroenteritis, cyclic vomiting, or psychological disorders. Because prolonged volvulus may lead to more serious consequences such as malabsorption, failure to thrive, and intestinal ischemia, a high index of suspicion is required. Air in the biliary tree (A) is commonly seen in necrotizing enterocolitis (NEC). NEC most commonly occurs in infants born pre-term. Maternal use of erythromycin (B) during pregnancy or while breastfeeding is associated with development of pyloric stenosis, but it has not been linked to malrotation. Hypochloremic metabolic alkalosis (D) is associated with pyloric stenosis, not intestinal malrotation.

An 18-month-old male presents with a rash under his nose. The rash began as a papulovesicular eruption with surrounding erythema. The vesicles then ruptured and left behind a honey-crusted plaque. The rash has not spread, and the child has been otherwise well appearing. What is the most appropriate treatment? Amoxicillin Amoxicillin-clavulanate Topical mupirocin Trimethoprim-sulfamethoxazole

Correct Answer ( C ) Explanation: Topical mupirocin provides adequate treatment for mild, self-limited impetigo. In addition to ease of application, topical antibiotic ointment avoids the risk of development of antibiotic resistance that is associated with systemic antimicrobial therapy. Impetigo can be caused by both Strep or Staph spp. However, Staph aureus is the most likely cause of both bullous impetigo and crusted impetigo.

An 8-month-old infant with a ventricular septal defect has significant growth failure. He also suffers from recurrent pneumonia. Which of the following is the best management of this patient? Atenolol Furosemide Indomethacin Surgical repair

Correct Answer ( D ) Explanation: Infants or children with ventricular septal defects (VSDs) may or may not have symptoms based on the size of the defect. Smaller defects do not allow for a significant left-to-right cardiac shunt, therefore, many of these infants are asymptomatic and carry an excellent prognosis. For small defects, medication is usually not necessary. However, moderate and large defects typically require medication management for congestive heart symptoms. Options include diuretics and digoxin. If these patients fail medication management, surgical repair is recommended. Surgery is also recommended for any VSD that is associated with growth failure or recurrent respiratory infections.

One Step Further Question: What is Eisenmenger syndrome?

Answer: A septal defect leading to pulmonary arterial hypertension, causing a right-to-left shunt, bypassing the lungs and resulting in cyanosis and clubbing.

One Step Further Question: What is the cellophane tape test for pinworms?

Answer: A two- to three-inch piece of clear tape is applied serially to several perianal areas in the morning before washing. The tape is then applied to a glass slide.

One Step Further Question: Besides RSV, what three other viruses have been found to cause acute bronchiolitis?

Answer: Adenovirus, parainfluenza virus, and rhinovirus.

One Step Further Question: What finding on wet mount is diagnostic of bacterial vaginosis?

Answer: Clue cells (epithelial cells surrounded by bacteria).

One Step Further Question: A patent ductus arteriosus is actually beneficial in infants with which other congenital heart diseases?

Answer: Ductal dependent lesions include transposition of the great vessels, Tetralogy of Fallot, tricuspid atresia, hypoplastic left heart and severe coarctation of the aorta.

One Step Further Question: What are the three types of botulism infection?

Answer: Foodborne botulism, wound botulism and infant botulism.

One Step Further Question: What laboratory tests are used to assess for isoimmune hemolytic disease?

Answer: Hematocrit, reticulocyte count, and a Coombs test.

One Step Further Question: What kind of anemia is associated with Mycoplasma pneumoniae?

Answer: Hemolytic anemia secondary to cold agglutinins.

One Step Further Question: Which disease is characterized by intrinsic acute renal failure, microangiopathic hemolytic anemia, and thrombocytopenia?

Answer: Hemolytic uremic syndrome (HUS).

One Step Further Question: What is the first line antibiotic treatment for uncomplicated acute otitis media?

Answer: High-dose amoxicillin at 80-90 mg/kg/day.

One Step Further Question: What is test is absolutely necessary for any child diagnosed with pauciarticular juvenile idiopathic arthritis?

Answer: Ophthalmology referral, and slit lamp examination 4x times a year for 4 years.

One Step Further Question: What is the most common type of atrial septal defect?

Answer: Ostium secundum (75% of all cases), in which there is incomplete adhesion between the foramen ovale flap and the septum secundum.

One Step Further Question: What rash is associated with Henoch-Schonlein purpura, a common cause of intussusception?

Answer: Palpable purpura.

One Step Further Question: What is the most common cause of croup?

Answer: Parainfluenza virus.

One Step Further Question: What is the most common cause of croup (ie, laryngotracheobronchitis)?

Answer: Parainfluenza.

One Step Further Question: Infection with what common childhood virus can cause a transient aplastic crisis in patients with sickle cell disease?

Answer: Parvovirus B19.

One Step Further Question: What nerve distributions are affected by a Nevus of Ito?

Answer: Posterior supraclavicular and lateral brachiocutaneous nerves.

One Step Further Question: When do Koplik's spots occur in relation to the rash of measles?

Answer: Prior to the onset of rash.

One Step Further Question: What is the key treatment for critical coarctation of the aorta?

Answer: Prostaglandin E1.

One Step Further Question: How can purpura be differentiated from ecchymoses?

Answer: Purpura do not blanch under pressure.

One Step Further Question: The athlete who suffers multiple concussions and returns to play too soon is at risk for what syndrome?

Answer: Second impact syndrome.

One Step Further Question: What complication can occur secondary to prolonged application of lindane?

Answer: Seizures.

One Step Further Question: What alternative category of shock should be considered when suspected hypovolemic shock does not respond to fluid resuscitation?

Answer: Septic shock.

One Step Further Question: Which medications are recommended for treating WPW during pregnancy?

Answer: Sotalol or flecainide.

One Step Further Question: What is the most common etiology of bacterial tracheitis?

Answer: Staphylococcus aureus.

One Step Further Question: What is the most common organism associated with bacterial tracheitis?

Answer: Staphylococcus aureus.

One Step Further Question: What is the definitive form of treatment for infantile hypertrophic pyloric stenosis?

Answer: Surgery (pyloromyotomy).

One Step Further Question: What is the test to order to screen for hypothyroidism?

Answer: TSH.

One Step Further Question: What feature of tetralogy of Fallot determines the prognosis?

Answer: The prognosis of tetralogy of Fallot depends on the severity of right ventricular outflow tract obstruction.

One Step Further Question: Where should the umbilical cord be cut?

Answer: Three centimeters from the insertion point at the umbilicus.

One Step Further Question: How long does immunity last after vaccination or clinical infection?

Answer: Three to eight years after vaccination and 15 years after clinical infection.

One Step Further Question: What is the treatment for recurrent facial impetigo?

Answer: Topical mupirocin applied to both the rash and the inside of bilateral nares.

One Step Further Question: At what age is an infant expected to regain birth weight?

Answer: Two weeks of life.

Which of the following is an acyanotic congenital cardiac defect? Coarctation of the aorta Ebstein's anomaly Hypoplastic left heart syndrome Pulmonary atresia

Correct Answer ( A ) Explanation: Coarctation of the aorta is an acyanotic congenital heart lesion. Acyanotic congenital heart defects (CHD) should be divided into obstructive lesions and lesions with left-to-right shunts and increased pulmonary blood flow. Acyanotic lesions typically present in the first 6 months of life with congestive heart failure. Obstructive CHD includes pulmonic stenosis, aortic stenosis and coarctation of the aorta. Left-to-right shunt CHD includes VSD, ASD, patent ductus arteriosus and endocardial cushion defects. Coarctation of the aorta accounts for approximately 8% of all CHDs

Which of the following is most characteristic of a complex febrile seizure? Convulsions are focal in nature Convulsions lasting 12 minutes Second seizure occurs with second febrile illness Single seizure that occurs at a temperature of 40°C

Correct Answer ( A ) Explanation: Complex febrile seizures are diagnosed when multiple seizures occur during the same febrile illness, seizures are prolonged (>15 minutes), or the seizures have a focal component. Simple febrile seizures are associated with generalized convulsions. Seizures lasting under 15 minutes (B) are associated with simple febrile seizures. When multiple seizures occur within the same febrile illness, the seizures are classified as complex. However, a patient may have a recurrent febrile seizure during a subsequent febrile illness (C) and still be classified as a simple febrile seizure. A febrile seizure can occur at any temperature (D), but almost 50% of children have a documented temperature of <39.0°C.

23-year-old woman delivers her first child. Her family history is positive for three uncles who needed early-in-life surgery for "heart defects." You are asked to assess her 1-day-old infant who does not "appear well" according to the nursing staff. During your examination, which of the following findings most suggests the presence of congenital heart disease? Basilar crackles and peripheral edema Fever Symmetric brachial and femoral pulses Systolic murmur

Correct Answer ( A ) Explanation: Congenital cardiac defects occur in 8 out of 1000 live births. Up to one third of infants born with a congenital cardiac defect develop life-threatening symptoms within the first few days of life, with 80% of infants presenting with congestive heart failure (pulmonary or peripheral edema or both). The mortality rate in this critical period is 90%. The majority of these defects can be screened for in the prenatal period with four-chamber echocardiography. Consider screening in women with diabetes, a family history of congenital heart disease, indomethacin exposure or rubella exposure

The clinic receives a report that one of your patients has congenital hypothyroidism that was identified by newborn screening. You review the chart and the patient was last seen at around two weeks of age. At that time, the infant had regained birth weight and was breastfeeding well, and the mother did not have any concerns. Your physical examination at that time was normal. You call the mother, and she reports that the infant is doing well. Which of the following is the next best step? Obtain thyroid stimulating hormone (TSH) level Refer to a pediatric endocrinologist Repeat newborn screening Start levothyroxine

Correct Answer ( A ) Explanation: Most infants with congenital hypothyroidism are detected by newborn screening programs in the first few weeks after birth. Newborn screening is done through blood obtained by heel-prick between two and five days of life, which is placed on a filter paper card and sent to a central screening laboratory. Many newborn screening programs in North America and Europe measure levels of T4, followed by measurement of thyroid-stimulating hormone (TSH) when T4 is low. Early diagnosis and adequate treatment from the first weeks of life result in normal linear growth and intelligence. Infants with abnormal screening results are recalled for further testing. At recall, the infant should be examined and a serum sample obtained by venipuncture to confirm the diagnosis of hypothyroidism.

A 1-year-old boy is brought to the urgent care clinic for fever, cough, and difficulty breathing. His mother tells you he had an upper respiratory tract infection 3-days ago. His temperature is 40.0°C (104°F). Auscultation of the lungs reveals expiratory wheezing and crackles on both sides. Physical examination shows suprasternal and subcostal retractions. A chest X-ray shows hyperinflation of the lungs and peribronchial thickening. Which of the following is the most likely diagnosis? Acute bronchiolitis Acute bronchitis Croup Respiratory syncytial virus

Correct Answer ( A ) Explanation: This patient most likely has acute bronchiolitis. Bronchiolitis is a clinical syndrome that occurs primarily in children younger than two years of age. Patients typically present with fever, cough, and respiratory distress (eg, increased respiratory rate, retractions, wheezing, crackles). Acute bronchiolitis often occurs following a 1 to 3 day history of an upper respiratory tract infection. Radiographic features of bronchiolitis include hyperinflation and peribronchial thickening. In healthy infants and young children, bronchiolitis usually is a self-limited disease. Management in most cases consists of supportive measures to maintain oxygenation and hydration. Bronchodilator therapy may be used to provide short-term improvement. Acute bronchitis (B) primarily presents with a cough, which may include sputum production that lasts for more than 5 days. Croup (C) typically presents with inspiratory stridor, cough (barking cough), and voice hoarseness and results from inflammation in the larynx and subglottic airway. Respiratory syncytial virus (D) is the most common cause of bronchiolitis and the virus most often detected as the sole pathogen.

A two-year-old child is seen in the emergency room with recent onset of cough and progressive stridor. Respiratory cultures are taken, and you are concerned for bacterial tracheitis. What is the most common organism isolated on respiratory cultures in bacterial tracheitis? Haemophilus influenzae type B Staphylococcus aureus Streptococcus pneumoniae Streptococcus pyogenes

Correct Answer ( B ) Explanation: Bacterial tracheitis can be seen in healthy children between three months and six years of age with a peak occurrence between three and five years old. Parents will often describe URI symptoms prior to bringing their child in to be evaluated. Blood cultures rarely are positive, but respiratory cultures are commonly polymicrobial with staphylococcus aureus the most common isolated species. Bacterial tracheitis is usually diagnosed on a clinical basis. The classic presentation is a child with suspected croup who continues to worsen despite treatment. High fever develops. These patients are at increased risk for airway compromise. They should be immediately started on broad-spectrum intravenous antibiotics.

Parents bring their one-month-old infant in for evaluation because of excessive crying and fussiness. Which of the following is associated with a pathologic diagnosis? Continued crying after the application of tetracaine to the eye Finger swelling Mustard colored stool Soothing when put in parent's arms

Correct Answer ( B ) Explanation: Crying is the infant's response to physiologic needs like hunger, cold or discomfort. However, excessive crying may also suggest a serious illness and it is important for the clinician to differentiate these infants from others. Maternal history (e.g. medications, drugs) and infant history (birth history, stool habits, feeding) may help to guide the clinician. Physical examination findings revealed the diagnosis in less than half in a case series of crying infants. A swollen digit may be a clue to a hair tourniquet constricting the finger and causing pain. It may not be obvious to the parents until the finger becomes grossly swollen and discolored. This can result in local ischemia, necrosis, and autoamputation. In any infant with inconsolable crying, this syndrome must be excluded.

A 7-year-old boy presents with his mom because his teacher says he "zones out" all day at school. His mom denies hyperactivity or destructive, disobedient actions but says he often "stares off" during meals for a few seconds. An EEG shows a generalized 3-Hz spike and wave pattern. Which of these interventions is first-line for his treatment? Atomoxetine Ethosuximide Gabapentin Methylphenidate

Correct Answer ( B ) Explanation: Ethosuximide is the first-line medication for treating absence seizures as it is most likely to reduce frequency of epileptic spells without causing adverse effects on a child's attentiveness. Absence seizures typically present in childhood as frequent, sudden, brief periods of unconsciousness, with or without automatic movements. An EEG is critical to the diagnosis of absence seizures as it will demonstrate a classic 3 Hz generalized spike-and-wave pattern. Absence seizure are unique in their comparatively abrupt ending and lack of a post-ictal period.

A six-month-old boy presents with five days of nasal congestion and discharge that has now progressed to fever, coughing, and wheezing. You suspect acute bronchiolitis. What are you likely to find on physical examination? Cervical lymphadenopathy Expiratory wheezing Inspiratory stridor Pharyngeal exudate

Correct Answer ( B ) Explanation: Expiratory wheezing is one of the physical examination findings of acute bronchiolitis along with tachypnea, chest retractions, prolonged expiratory phase, and rales. Acute bronchiolitis is a viral infection most commonly caused by respiratory syncytial virus (RSV) and typically affects infants and children. The symptom course starts with upper respiratory symptoms of nasal congestion and drainage with a mild cough followed by lower respiratory symptoms of expiratory wheezing, tachypnea, and rales. A low-grade fever is also generally present. Diagnosis of acute bronchiolitis is typically clinical as laboratory tests and imaging are not useful. Treatment includes supportive measures of rehydration and humidified oxygen.

You are examining a newborn male. Scrotal palpation reveals the absence of a testicle. If the condition does not spontaneously resolve, which of the following is the most appropriate intervention? Oophorectomy Orchiopexy Preputioplasty Salpingectomy

Correct Answer ( B ) Explanation: Normal testicle development begins at 7 weeks gestation in the abdomen. At about week 28, they descend through the internal inguinal ring into the scrotum. An undescended testicle will remain in the inguinal canal, abdomen or retroperitoneal space. This is called cryptorchidism, which affects 3% of term and 30% of preterm infants. Nearly 67% of all undescended testes descend by the age of 4 months. About 10% of cases are bilateral. An empty scrotum and an inguinal hernia are the most common presenting signs. The diagnosis is mainly clinical, but may also require laparoscopy or ultrasonography. If the testis does not spontaneously descend by age 6 months, the necessary treatment is orchiopexy, the surgical repositioning of an undescended testicle into the scrotum with subsequent hernia repair. Repairing an undescended testicle improves fertility, but does not lower the risk malignancy. Oophorectomy (A) is the surgical removal of an ovary. Preputioplasty (C) is a plastic surgical operation used to reduce a non-retractile, constricting foreskin (phimosis). Salpingectomy (D) is the surgical removal of one or both fallopian tubes.

Which of the following is most likely to present as a ductal-dependent cardiac lesion? Atrial septal defect Coarctation of the aorta Isolated ventricular septal defect Mitral valve prolapse

Correct Answer ( B ) Explanation: Patients with coarctation of the aorta may present with circulatory failure and shock upon closure of a patent ductus arteriosus (PDA). In many cases, the coarctation of the aorta occurs juxtaductal (adjacent to the PDA). The PDA may serve to widen the juxtaductal area of the aorta so that blood may flow forward from the left ventricle. However, in other cases, the PDA serves as a conduit for right-to-left shunted blood from the right ventricle. In the latter case, infants classically present with differential cyanosis due to well-oxygenated blood reaching the upper body (pink) from the ascending aorta, and deoxygenated blood reaching the lower body (blue) via the PDA and descending aorta. This is because the PDA often inserts distal to the origin of the left subclavian artery from the aorta. When the ductus arteriosus closes, this can lead to circulatory failure and shock.

A 3-week-old boy presents with non-bilious, forceful emesis for two days. On examination, the infant appears active and feeds vigorously followed immediately by vomiting. Which of the following is the most appropriate diagnostic study? Abdominal radiograph Abdominal ultrasound CT scan of the abdomen and pelvis Urine dip

Correct Answer ( B ) Explanation: The patient presents with symptoms consistent with hypertrophic pyloric stenosis, which is most accurately diagnosed by ultrasound of the abdomen. Hypertrophy or hyperplasia of the pyloric sphincter is not present at birth but develops progressively over time. It typically presents in infants between the age of 2 weeks and 2 months. Pyloric stenosis is characterized by projectile, non-bilious vomiting that occurs immediately after feeding due to gastric outlet obstruction. Infants will appear very hungry in between feedings. On examination, peristaltic waves may be seen moving from left to right and the clinician may palpate an "olive" shaped mass in the right upper abdomen. Labs may reveal a hypochloremic metabolic alkalosis in patients with delayed presentations. The cornerstones of diagnosis are either ultrasound or upper GI series. Both have an accuracy of 95%.

Which of the following treatments decreases the need for repeat medical visits for children with croup? Codeine Cool mist therapy Glucocorticoids Nebulized racemic epinephrine

Correct Answer ( C ) Explanation: Glucocorticoids are a mainstay of treatment for children with mild, moderate, and severe croup. They provide a long-lasting anti-inflammatory effect and have been shown to decrease croup scores six hours after administration. Additionally, in meta-analyses, glucocorticoids have been shown to decrease the rate of repeat medical visits, emergency room visits, emergency room length of stay, hospital length of stay, need for administration of epinephrine. Dexamethasone is the glucocorticoid most commonly used for treatment of croup due to the necessity for only a single dose. It can be administered orally, intramuscularly, or intravenously. Efficacy is comparable between oral and intramuscular dexamethasone.

A 14-month-old boy presents to the emergency department with irritability. His parents report episodes of irritability that have been occurring every 2 hours over the past 12 hours. The episodes last for about 15 minutes then seem to resolve. He also seems sleepier than usual. His last stool was 2 days prior. On examination, the infant appears lethargic. He has mild abdominal distension and has involuntary guarding. Which of the following is the best diagnostic test? CT scan of the abdomen Hemoccult of the stool Ultrasound of the abdomen X-rays of the abdomen

Correct Answer ( C ) Explanation: Intussusception is best diagnosed by an abdominal ultrasound. Intussusception is a type of intestinal obstruction that occurs when part of the bowel telescopes into a nearby segment. It is most commonly seen in children aged 3 months to 6 years, and typically involves the ileocolic segment of bowel. Infants and children present with severe, intermittent abdominal pain and may develop into lethargy. Prolonged intussusception may result in shock. Affected children may have a recent history of emesis, and may have stools with blood and mucus. Once the obstruction occurs, stooling decreases. Diagnosis should be made by abdominal ultrasound, which has a sensitivity of 98-100% and a specificity of 88%. Ultrasound will show a tubular mass in longitudinal views, with a target sign in transverse views. Treatment consists of air, saline, or occasionally water-soluble contrast enemas. Complications of intussusception include bowel infarction, peritonitis, and death.

Which of the following is true regarding Meckel's diverticulum? Commonly seen at five years of age Diagnosed by abdominal ultrasound Diagnosed by Meckel's scan using Tc99m pertechnetate Presents with bilious vomiting

Correct Answer ( C ) Explanation: Meckel's diverticulum usually presents with painless rectal bleeding and is due to incomplete obliteration of the vitelline duct. It is located two feet from the ileal cecal valve, is two inches long and commonly seen in children younger than two years of age. Bleeding results from ulceration of the ileal mucosa caused by acid secretion from the gastric mucosa-lined epithelium. It is diagnosed via Meckel's scan using Tc99m pertechnetate, which is a radioactive chemical taken up by the gastric mucosa. The treatment is surgical resection.

Which of the following statements is correct regarding Osgood-Schlatter disease? Disorder requires surgical intervention Occurs commonly in women over age 30 Pain occurs over the tibial tuberosity Requires MRI for confirmation of diagnosis

Correct Answer ( C ) Explanation: Osgood-Schlatter disease manifests as pain over the tibial tubercle in a growing child. The patellar tendon inserts into the tibia tubercle, which is an extension of the proximal tibial epiphysis. Osgood-Schlatter disease is likely a traction apophysitis of the tibial tubercle growth plate and the adjacent patellar tendon. It occurs during late childhood or adolescence, especially in athletes, and is likely due to repetitive tensile microtrauma. There is often increased prominence of the tibia tubercle that is also firm. Radiographs can be normal or may show small spicules of heterotopic ossification anterior to the tibial tuberosity. This disorder is self-limited in most patients and resolves with skeletal maturity.

A 2-year-old boy presents from a rural Native American reservation with an episode of a 3 minute generalized seizure. On further questioning, he has had 3 days of diarrhea, tenesmus, and fever. He has no prior history of seizures. Which of the following is most likely causing his symptoms? Campylobacter Salmonella Shigella Yersinia

Correct Answer ( C ) Explanation: Shigella species are aerobic, gram-negative bacilli that are primarily transmitted person to person by the fecal-oral route or by ingestion of contaminated food or water. Children in daycare settings or crowded living conditions are most commonly affected. The onset of illness is several days after ingestion. Shigella infects the large intestine and causes watery or loose stools, fever, abdominal cramps or tenderness, tenesmus, and mucoid bloody stools. The characteristic bloody diarrhea appears after the fever subsides. Children under 2 years of age are more likely to develop high fever and seizures. Lab findings are notable for an increased number of bands on the CBC regardless of the actual white blood cell count. Due to high resistance to ampicillin and trimethoprim-sulfamethoxazole, azithromycin is the oral drug of choice for treatment in children. Rare complications include bacteremia, Reiter syndrome, hemolytic-uremic syndrome, toxic megacolon, intestinal perforation and toxic encephalopathy. Salmonella (B) is the most commonly implicated pathogen in food-borne gastroenteritis and usually causes a self-limiting disease but rarely can lead to complications including bacteremia, enterocolitis, meningitis, and osteomyelitis. Campylobacter (A) infections cause diarrhea, abdominal pain mimicking appendicitis or intussusception, malaise and fever but in neonates and young infants bloody diarrhea without fever may be the only manifestation. Yersinia (D) usually manifests as fever and diarrhea in young children and stool often contains leukocytes, blood and mucus. Seizures are not associated with Yersinia.

A boy aged 30 months presents with respiratory distress. He was diagnosed with croup one week ago and received a single dose of dexamethasone. His cough had nearly resolved until yesterday, when coughing worsened, and he developed nasal congestion. This evening, he developed inspiratory stridor, increased work of breathing, and a fever to 103°F. What is the most likely diagnosis? Bacterial tracheitis Bronchiolitis Foreign body aspiration Recurrent croup

Correct Answer ( A ) Explanation: Bacterial tracheitis is a rare upper airway obstruction in children. It may occur in isolation or following a viral respiratory illness, including croup. Bacterial tracheitis should be suspected in any child with clinical worsening after improvement of croup symptoms. In children without a preceding illness, symptom onset may begin insidiously with one to three days of viral upper respiratory infections prior to decompensation with respiratory distress, inspiratory stridor, and high fevers. Other children will not demonstrate prodromal symptoms and will present acutely toxic and in extremis. The most common causative organism is Staphylococcus aureus. Less common causes include S.pneumoniae, S.pyogenes, M.catarrhalis, and H.influenzae. Endoscopy is necessary to establishing a definitive diagnosis, but it should take place in a setting where airway supplies and personnel are readily available, such as the operating room or the intensive care unit. Bronchiolitis (B) would be expected to show signs of lower respiratory tract involvement such as crackles and wheezes on examination. Moreover, the child's acute worsening of symptoms of upper respiratory infection following an episode of croup should raise suspicion for bacterial superinfection rather than acquisition of a new viral respiratory infection Recurrent croup (D) would be unexpected in an otherwise healthy child who had been improving throughout the week following treatment with dexamethasone.

One Step Further Question: Does a Still's murmur radiate to the carotids?

No

One Step Further Question: What type of collagen is affected in Osteogenesis Imperfecta?

Answer: Type 1 Collagen.

One Step Further Question: What medication should be used in the treatment of RMSF in pregnant patients or those with allergies to tetracyclines?

Answer: Chloramphenicol.

One Step Further Question: What is the diagnosis if a second febrile seizure occurs in 24 hours?

Answer: Complex febrile seizures.

One Step Further Question: What is the leading cause of cardiac death among the young in developing countries?

Answer: Acute rheumatic fever.

One Step Further Question: What are considered confirmatory tests of group A streptococcal infection when evaluating a child with possible rheumatic fever?

Answer: Confirmatory testing may be either a positive throat culture, positive rapid antigen test, or rising streptococcal antibody titer (ASO).

One Step Further Question: What is Dance's sign?

Answer: Considered pathognomonic for intussusception: a sausage-like mass in the RUQ representing the actual intussusceptum and an empty space in the RLQ representing the movement of the cecum out of its normal position.

One Step Further Question: In paraphimosis, if manual reduction of the foreskin cannot be achieved, what is the next step?

Answer: Dorsal slit procedure.

One Step Further Question: What is the drug of choice for empiric outpatient treatment of bacterial community-acquired pneumonia in children younger than age 5?

Answer: High-dose amoxicillin.

One Step Further Question: What are three contraindications to receiving the MMR vaccine?

Answer: Pregnancy, immunocompromised individuals, and advanced malignancy.

One Step Further Question: What vaccine protects from a common cause of severe watery diarrhea in children?

Answer: Rotavirus vaccine.

One Step Further Question: What is the oral chelation medication for lead poisoning?

Answer: Succimer.

One Step Further Question: What are the complications of cryptorchidism?

Answer: Testicular carcinoma, subfertility and testicular torsion.

An obese 11-year-old Cherokee boy presents with symmetrical, hyperpigmented, velvety plaques in the intertriginous areas of the axilla, groin, and posterior neck. There are a few acrochordons scattered in the plaques. The plaques are painless, and do not itch. What is the most likely diagnosis? Acanthosis nigricans Addison's disease Dowling-Degos disease Pellagra

Correct Answer ( A ) Explanation: Acanthosis nigricans is a thickening and hyperpigmentation of the intertriginous regions of the skin. Acanthosis nigricans is more common in those with Native American, African-American, or Hispanic heritages than in Caucasians. The condition is commonly associated with other conditions such as insulin resistance and obesity, but it has also been associated with certain genetic syndromes, malignancies and drug reactions. Since acanthosis nigricans is often an asymptomatic disorder, cosmetic concerns are typically the primary indications for treatment. Treatment of the underlying cause is the preferred method of management. Otherwise, topical retinoids are the preferred treatment of the plaques because of their keratinolytic effects.

A 24-year-old pregnant woman presents to the urgent care clinic where you are working. During your assessment, she states that she saw her obstetrician when she first became pregnant but has not had any follow-up care since. You also find out that she has bipolar disorder and took lithium throughout the first trimester. What congenital abnormality is the fetus at risk of developing? Ebstein's anomaly Eisenmenger's syndrome Pulmonary atresia Tetralogy of Fallot

Correct Answer ( A ) Explanation: Although rare, Ebstein's anomaly is a congenital cardiac malformation that can in part be attributed to maternal lithium use during pregnancy, especially within the first trimester. Other causes are suspected to be multifactorial in nature, with genetics, environment, and other medications (benzodiazepines) during pregnancy shown to be possible contributors. During fetal development, there is a failure of the tricuspid valve to form properly, resulting in downward displacement of the leaflets and incompetency of the valve. This leads to a relatively small functional right ventricle, tricuspid regurgitation, and eventually symptoms of heart failure. Treatment options include observation and medical (heart failure management) for mild cases and surgical (valve replacement/repair, shunt repair) for more severe cases. Eisenmenger's syndrome (B) is the reversal of a left-to-right shunt due to increasing pulmonary pressures. Instead of simply recirculating oxygenated blood through the right side of the heart, the increased right-sided pressures causes a shunt of deoxygenated blood into the systemic circulation. Pulmonary atresia (C) refers to a constellation of congenital abnormalities that results in an unformed pulmonic valve. It presents acutely after birth as the ductus arteriorus closes, and blood is not able to be oxygenated. Tetralogy of Fallot (D) refers to four separate congenital abnormalities that are present in the same heart: ventricular septal defect (VSD), right ventricular hypertrophy, right ventricular output obstruction, and an overriding aort

A 14-year-old boy presents to the clinic for a well child check. He has been growing well and has no medical complaints. On exam, he is found to have a III/VI systolic crescendo-decrescendo murmur heard best over the left sternal border. The murmur increases when moving from a squatting position to a standing position, and increases with Valsalva maneuver. The point of maximal impulse is displaced laterally. Which of the following is the next best step in management? Cease all physical activity until further evaluation can be performed Encourage hydration during physical activity Perform genetic testing Prescribe a beta-blocker

Correct Answer ( A ) Explanation: Hypertrophic cardiomyopathy (HCM) is an autosomal dominant disease with variable expression that presents with a classic murmur. The murmur is usually delayed in onset, 3-4/6 grade, and has a crescendo-decrescendo pattern loudest over the left lower sternal border. The murmur increases when venous return is decreased. Therefore, the murmur increases when moving from a sitting to standing position. Classic ECG findings include left ventricular hypertrophy, prominent septal Q waves, and abnormal repolarization or strain (e.g. negative T waves in V6). An echocardiogram is diagnostic and will show septal and ventricular wall hypertrophy. The most feared complication is sudden cardiac death from a dysrhythmia. Treatment options include limiting tachycardia to preserve ventricular filling; therefore, options include avoiding physical activity and prescribing beta-blockers. Cardiac defibrillators and transplant are more definitive treatment options.

A four-year-old boy presents to the office with seven days of fever. On HEENT exam you note bilateral conjunctivitis, note some fissuring of his lips and palpate a cervical lymph node. The cardiovascular and pulmonary exams are normal. Skin exam reveals a macular rash on his chest and skin desquamation on his feet. Which of the following is the most likely diagnosis? Kawasaki disease Lyme disease Measles Scarlet fever

Correct Answer ( A ) Explanation: Kawasaki disease typically presents in children younger than four years, most commonly at age two. It is more common in males and occurs more frequently in the winter and spring months. The diagnosis requires the criteria found in the image below. Laboratory findings may reveal thrombocytosis, leukocytosis, and a normocytic anemia. Acute phase reactants (CRP, ESR, and alpha-1 antitrypsin) are often elevated. Antinuclear antibodies (ANA) and rheumatoid factor (RF) are not elevated in Kawasaki disease. The main complication is coronary artery disease. Treatment includes high dose aspirin (80 mg/kg/day) and IVIG. Aspirin maintenance (5 mg/kg/day) is continued for two months after high dose aspirin is administered for two days. A 2D-echocardiogram should also be performed at the time of diagnosis, two to three weeks and six to eight weeks after the diagnosis is made.

A 10-month-old girl is brought to the ED by parents because they noted her to be tugging her right ear. On otoscopy, you note the finding seen in the image above. Which of the following is most closely associated with acute otitis media? Bulging tympanic membrane Erythema of the ear canal and erythematous tympanic membrane Low-grade fever and tugging at the ear Opacification of the tympanic membrane

Correct Answer ( A ) Explanation: Otitis media is a general term for middle ear inflammation; acute otitis media (AOM) is associated with an infection, whereas otitis media with effusion (OME) is associated with fluid behind the tympanic membrane without an infection. The incidence of AOM is bimodal, with a 1st peak between the ages of 6 and 18 months and the 2nd, smaller peak at 5 years. A bulging tympanic membrane is the single finding most closely associated with acute otitis media. Bulging of the tympanic membrane has the highest predictive value for the presence of middle ear effusion.

Viral croup or laryngotracheitis will most commonly present with which of the following symptoms? Barking cough Drooling Dry cough of a two weeks' duration Muffled hot potato voice

Correct Answer ( A ) Explanation: Viral croup typically begins as an upper respiratory infection with coryza and a low grade fever for 12 to 72 hours. It is characterized by a barking (seal-like) cough. Other symptoms include stridor, retractions, and hoarseness.These symptoms are worse at night, during the first 24 to 48 hours and usually resolve over the course of one week. Treatment of mild croup is 0.15 mg/kg to 0.6 mg/kg oral dexamethasone as a one-time dose. If stridor is present at rest, nebulized epinephrine is an accepted treatment along with the dexamethasone. The most common viruses causing croup are parainfluenza types 1 to 3. Most episodes of croup are mild. Approximately 1 percent to 8 percent require hospitalization. Drooling (B) and a muffled hot potato voice (D) are symptoms seen more commonly in epiglottitis, a rapidly progressive, potentially life-threatening bacterial infection of the epiglottis and the surrounding structures. A dry cough of two weeks' duration (C) may be a symptom of a post-nasal drip commonly associated with seasonal allergies and allergic rhinitis.

A 17-month-old boy presents with bilious vomiting, fever, and abdominal distention for the past three days. The mother states she noticed blood in the last diaper she changed. What is the most likely diagnosis? Anal fissure Intussusception Milk protein allergy Pyloric stenosis

Correct Answer ( B ) Explanation: The presence of bilious vomiting should always raise concern for intestinal obstruction such as intussusception. Intussusception is commonly caused by a pathological lead point. This is a lesion or variation in the intestine that is trapped by peristalsis and dragged into a distal segment of intestine. This causes bowel obstruction and ischemia. The most common location is when the terminal ileum telescopes into the right colon. It is usually diagnosed via ultrasound, which shows a "coiled spring" or "target lesion" representing layers of intestines within the intestine. Surgery is indicated in patients who are acutely ill or have evidence of perforation. Otherwise, patients are treated with non-operative reduction using hydrostatic or pneumatic pressure by enema.

A 17-month-old boy presents with bilious vomiting, fever, and abdominal distention for the past three days. The mother states she noticed blood in the last diaper she changed. What is the most likely diagnosis? Anal fissure Intussusception Meckel diverticulum Milk protein allergy

Correct Answer ( B ) Explanation: The presence of bilious vomiting should always raise concern for intestinal obstruction such as intussusception. Intussusception is commonly caused by a pathological lead point. This is a lesion or variation in the intestine that is trapped by peristalsis and dragged into a distal segment of the intestine. This causes bowel obstruction and ischemia. The most common location is when the terminal ileum telescopes into the right colon. It is usually diagnosed via ultrasound, which shows a "coiled spring" or "target lesion" representing layers of intestines within the intestine. Surgery is indicated in patients who are acutely ill or have evidence of perforation. Otherwise, patients are treated with non-operative reduction using hydrostatic or pneumatic pressure by enema.

A 10-week-old boy is brought to the Emergency Department in late December by his mother who reports that he has had difficulty breathing for the past 24 hours. His mother and sister have a history of asthma, and his father has been sick with an upper respiratory tract infection. Vital signs are notable for a respiratory rate of 70 breaths/min and an oxygen saturation of 99 percent on room air. Examination reveals significant rhinorrhea, intercostal retractions, bilateral expiratory wheezes and crackles at the bases. Nasal suctioning is performed with minimal improvement. He has normal urine output. Which of the following is the most appropriate next step in management? Administer prednisolone and nebulized albuterol Admission to monitored bed for continued supportive care Discharge with recommendations for next-day pediatrician follow-up Perform a chest X-ray and administer empiric antibiotics

Correct Answer ( B ) Explanation: This infant has bronchiolitis. Bronchiolitis is an acute viral disease resulting in small airway inflammation and is associated with signs and symptoms of an upper respiratory infection. Patients are typically less than two years of age, and the disease is more frequently present in the winter months. Patients may be febrile, tachycardic, tachypneic, or hypoxic. Lung auscultation can demonstrate decreased air movement, crackles, or wheezing. Emergency Department management is focused on supportive care and risk assessment. Disease severity usually peaks around days two to three of illness and complications include apnea, hypoxia, and dehydration. Factors that predict increased risk for poor outcomes include age < 3 months, gestational age < 34 weeks, tachypnea (RR > 70 breaths/min), ill appearance, or inability to maintain hydration. Historical factors that increase the risk of poor outcomes include congenital heart disease, chronic lung disease, and immunocompromise. This patient is tachypneic and is younger than three months old. Therefore, admission to a monitored bed without further Emergency Department intervention is the best management strategy. Inpatient management is focused on frequent nasal suctioning and oxygen supplementation as needed.

A 5-month-old girl presents to the ED with her parents who are concerned about brief episodes of "whole body muscle contraction." On physical exam, the child has poor head control and absent palmar grasp. Her electroencephalogram shows hypsarrhythmia. Which of the following is the most likely diagnosis? Cerebral palsy Infantile spasms Kernicterus Tay-Sachs disease

Correct Answer ( B ) Explanation: This patient is presenting with signs and symptoms consistent with infantile spasms or West syndrome. Infantile spasms are defined by the triad of findings: (1) clusters of myoclonic seizures on awakening, (2) hypsarrhythmia pattern on electroencephalogram, and (3) developmental delay. Signs and symptoms of infantile spasms usually begin at 4-8 months of age. Infants will demonstrate brief contractions of the neck, trunk, and extremities lasting 5 to 10 seconds each, occurring in clusters. These episodes may be accompanied by a cry. Imaging of the brain with CT or MRI is often abnormal; however, it is not diagnostic of infantile spasms. These children are most frequently diagnosed by the presence of the characteristic electroencephalogram (EEG) finding of hypsarrhythmia, which has the appearance of high amplitude irregular waves and spikes in a background of chaotic and disorganized activity. A lumbar puncture should also be considered and performed to rule out meningitis or encephalitis. Management of infantile spasms is primarily with adrenocorticotropic hormone (ACTH), prednisone, and antiepileptic medications. Cerebral palsy (A) may present with seizures and developmental delay; however, the child will also typically have other findings including muscular spasticity. Hypsarrhythmia is also nearly pathognomonic for the diagnosis of infantile spasms. Kernicterus (C) will usually present with jaundice and hypotonia. Tay-Sachs disease (D) presents with muscle atrophy beginning at age 6 months and an abnormal funduscopic exam with blindness.

A 14-day-old neonate presents to clinic with bilateral eye discharge and conjunctival injection. What treatment is indicated? Ceftriaxone intramuscular Erythromycin ointment Oral erythromycin Warm compresses and lacrimal duct massage

Correct Answer ( C ) Explanation: Eye discharge is extremely common amongst neonates. The most common cause of neonatal eye discharge is lacrimal duct obstruction. However, accompanying conjunctival involvement indicates a probable infectious etiology. Chlamydia trachomatis is the most common cause of neonatal conjunctivitis. The organism is acquired during delivery and has an incubation period of five to fourteen days. Thus, any conjunctivitis occurring less than five days from the time of delivery is unlikely to be caused by C.trachomatis, and evaluation for alternative etiologies should be pursued. Often the first manifestation of the Chlamydial conjunctivitis is watery discharge, which later becomes mucopurulent. Conjunctival chemosis then develops, and the conjunctiva may become so friable that they may bleed. Eyelid swelling may also develop. The treatment of choice of Chlamydial conjunctivitis is oral erythromycin. After completion of treatment, most neonates have full resolution of symptoms. Untreated neonates may develop permanent corneal and conjunctival scarring. Ceftriaxone (A) is the treatment of choice for neonatal gonococcal conjunctivitis.

A 6-year-old boy presents with fever and on physical exam has a strawberry tongue, red cracked lips, bilateral conjunctival injection, and palmar erythema. Which of the following increases suspicion for Kawasaki disease? Centripetally spreading petechial rash Fine "sand paper" maculopapular rash Generalized non-bullous, non-vesicular rash Macular papular rash starting at the head and spreading caudally

Correct Answer ( C ) Explanation: Kawasaki disease is the leading cause of acquired heart disease in children in the USA. The peak incidence is between the ages 18 and 24 months, with the majority of patients <4 years old and boys are affected more frequently than girls. There are 3 phases of Kawasaki disease. The acute phase lasts 1-2 weeks. This phase presents with the classic symptoms of Kawasaki disease (fever, lymphadenopathy, conjunctivitis, oropharyngeal changes, rash, vasculitis). The subacute phase occurs during weeks 2-4 and involves the onset of thrombocytosis (platelets up to 500,000-1,000,000), hand or feet desquamation, and resolution of the fever. The convalescent phase generally begins after 2 months where scarring and calcification of affected coronary arteries may occur. The diagnosis of Kawasaki disease is based on the following criteria: The presence of a generalized non-bullous, non-vesicular rash raises the suspicion for Kawasaki disease. Other conditions that can present with a similar rash include streptococcal disease, which can present with tonsillar exudates (Strep. pharyngitis) or a fine maculopapular "sandpaper" rash (B) as in Scarlet fever. However, conjunctivitis and swelling or erythema of the hands and feet are uncommon. Measles presents with similar symptoms (fever, red eyes, erythematous oropharynx, and rash), however it is rare in vaccinated children and it has a characteristic macular papular rash that starts at the head and spreads caudally (D). Rocky Mountain spotted fever is usually characterized by a centripetally spreading rash that eventually becomes petechial (A).

A two-month-old male infant presents in August with a copious, bubbly, tenacious mucous discharge from his nose and mouth. He also has a repetitive paroxysmal cough terminated by an inspiratory "whoop" and followed by a tussive episode. The infant had upper respiratory symptoms for the past two weeks. His parents report a possible seizure before taking him to the emergency room. Other infants in his day care center have been diagnosed with respiratory infections associated with a "prolonged" cough. The infant is pale and cyanotic, tachypnic and bradycardiac as well as anxious looking. Physical examination of his chest is consistent with a lobar pneumonia. What is the most likely diagnosis in this infant? Adenovirus Chlamydia Pertussis Respiratory syncytial virus

Correct Answer ( C ) Explanation: Pertussis classically presents in its most severe form in infants younger than six months of age. There is usually a short catarrhal phase (less than two weeks) followed by typical and severe whooping cough manifest with hypoxia, which is sometimes fatal. Pneumonia, seizures, and encephalopathy are the most common severe complications, and this infant presents with evidence of the first two. Prompt diagnosis may allow for life-saving care, including oxygen and hydration.

A 13-year-old boy on chemotherapy for acute lymphoblastic leukemia presents with progressive lower back pain for 2 weeks. Per mom, he has had subjective fevers and a "bulge in his lower back" that is warm to touch. He is currently afebrile and has no focal neurologic deficits. An MRI is obtained as seen in the image above. Which of the following regarding this patient's condition is true? AAn appropriate antibiotic regimen is cefepime and metronidazole BDirect extension of skin and soft tissue infections is the most common cause CErythrocyte sedimentation rate is a sensitive screening tool DMost patients present with back pain, fever, and a focal neurologic deficit

Correct Answer ( C ) Explanation: The patient has a posterior epidural abscess with adjacent osteomyelitis and myositis. The most common cause of an epidural abscess is hematogenous spread of infection, not direct extension from skin or soft tissue infection. Major risk factors include diabetes, intravenous drug abuse, chronic renal failure, alcoholism, and immunosuppression. The most common organism involved is Staphylococcus aureus. Other organisms include streptococci, anaerobes, gram-negative bacilli, and Pseudomonas aeruginosa. Patients present with localized back pain with tenderness to percussion. Fevers are common (reported in up to 75% of patients). MRI is the diagnostic modality of choice, but erythrocyte sedimentation rate is a sensitive marker and may be used in conjunction with plain radiographs to screen for infectious spinal disease.

A 2-year-old boy presents to the ED with painful swelling in the scrotum for the past 4 hours. His parents report a bulge in the groin that comes and goes but now is not going away. On exam, you note engorgement of the right scrotum. The child screams in pain when you try to touch the mass. Which of the following is the most appropriate next step in management? CT scan Manual detorsion Morphine sulfate Urinalysis

Correct Answer ( C ) Explanation: The patient in this case has an incarcerated inguinal hernia. A hernia is a protrusion of peritoneal contents through a defect in the abdominal wall. Incarcerated hernias result from bowel becoming edematous and engorged, with the inability to be reduced. A strangulated hernia occurs when there is vascular compromise of the herniated contents. Inguinal hernias are due to a patent processus vaginalis. Incarcerated hernias are a surgical emergency. If there are no signs of strangulation or systemic toxicity, manual reduction should immediately be attempted. Adequate analgesia and sedation should be provided prior to attempting reduction. The patient in this clinical scenario is clearly in pain and should be administered analgesia—morphine sulfate. This will aid in performing the physical exam and allow an attempt at manual reduction.

A 5-day-old baby girl presents to clinic with the foot deformity pictured above. Which of the following is the most appropriate initial treatment for this deformity? Observation Passive stretching exercises performed by the parents at home Serial manipulation and casting Soft tissue release and tendon lengthening

Correct Answer ( C ) Explanation: The patient pictured above has congenital talipes equinovarus, or clubfoot. Muscle contractures create the following deformities, remembered by the mnemonic CAVE: midfoot Cavus, forefoot Adductus, hindfoot Varus, and hindfoot Equinus. The Ponseti technique of serial manipulation and casting is indicated in treatment of clubfoot in children under age 12-18 months of age who have not had previous surgery. In most cases, the equinus deformity cannot be fully corrected by casting alone and will require lengthening of the Achilles tendon, a minor surgical procedure. Following casting, the patient is treated with a brace full-time for two months followed by night-time bracing until four to five years of age.

A 4-year-old girl presents complaining of left knee pain. The patient was started on antibiotics one week ago for a simple ear infection. She now has a widespread morbilliform rash, urticaria, several swollen joints, and temperature of 39.3°C. Which of the following is the most likely diagnosis? Henoch-Schonlein purpura Postviral arthritis Serum sickness Still's disease

Correct Answer ( C ) Explanation: This patient has acute serum sickness. Her presentation is classic with the development of a rash, joint pain, and fever a week after beginning antibiotics. Serum sickness is an immune-complex deposition disease (type III hypersensitivity) that occurs after exposure either to animal proteins or serum or to a variety of drugs. Serum sickness has distinctive cutaneous findings. Erythema first occurs on the sides of the fingers, toes, and hands before a more widespread morbilliform appears, sometimes with urticaria. Rash, fever, constitutional symptoms, and arthralgia are the most frequent clinical findings. Symptoms usually start 12 to 36 hours after ingestion if there has been previous immune system priming; absent this, onset may be delayed 7 to 21 days after antigen exposure. Treatment is supportive with antihistamines and antipyretics. Postviral arthritis (B) is a fairly common condition of toddlers. It is characterized by a recent viral syndrome with manifestation of monoarticular arthritis once the patient defervesces. Still's disease (D) is an autoimmune condition that heralds the onset of juvenile rheumatoid arthritis. It is characterized by high fever, arthritis, and a faint salmon-colored fleeting rash. This is similar to this patient's presentation, but with the recent exposure to antibiotics and an urticarial rash, serum sickness is the most likely diagnosis.

A 7-year-old boy presents to the emergency department with cough and chest pain. The cough has been present for several days, but the chest pain has been present for only one day. His vital signs are notable for a temperature of 38.8°C, heart rate of 150, blood pressure of 78/49, respiratory rate of 50, and oxygen saturation of 94%. He is ill appearing, has coarse breath sounds on lung auscultation, a systolic murmur and a gallop on his cardiac exam, and his liver is palpable 4 centimeters below the right costal margin. His pulses are palpable and capillary refill is 5 seconds. Which of the following is the most likely complication? Aortic valve insufficiency Constrictive pericarditis Dilated cardiomyopathy Left ventricular outflow tract obstruction

Correct Answer ( C ) Explanation: This patient has acute systolic dysfunction in the setting of an upper respiratory infection, which is most likely secondary to myocarditis. Myocarditis can be caused by infections, connective tissues disease, genetic diseases, toxins, and granulomatous diseases. The most common viral etiologies are enterovirus, adenovirus, parvovirus, EBV, and CMV. Myocardial damage may occur from inflammation and direct damage to the myocardium. This results in ventricular enlargement, poor systolic function, and dilated cardiomyopathy. The systolic dysfunction may result in congestive heart failure, arrhythmias, and even death. Patients typically present with chest pain, fever, and fatigue, and in fulminant disease they may have respiratory distress, tachycardia, hypotension, a gallop rhythm, and a murmur. Rash and end-organ involvement (i.e. hepatitis) may be seen. Dilated cardiomyopathy is the most common type of cardiomyopathy, and is characterized by left ventricular systolic dysfunction and left ventricular dilation. In pediatrics, it is usually caused by genetic mutations, myocarditis, or Kawasaki disease. Aortic valve insufficiency (A) is a possible complication of rheumatic fever. Rheumatic fever is characterized by carditis, migratory polyarthritis, erythema marginatum, subcutaneous nodules, and chorea. Constrictive pericarditis (B) is associated with viral infections and chest pain. However, the clinical findings seen in this disorder would be muffled heart sounds, a narrow pulse pressure, jugular venous distension, and a pericardial friction rub. Left ventricular outflow tract obstruction (D) is a finding in hypertrophic cardiomyopathy. Hypertrophic cardiomyopathy would not be expected to develop after myocarditis.

Which of the following statements is true regarding febrile seizures? Administering acetaminophen and ibuprofen during a febrile illness have been shown to decrease the likelihood of seizure recurrence Children who have had a simple febrile seizure have the same rate of epilepsy as those who have not had a febrile seizure Older children with a febrile seizure are more likely to have a recurrence than younger children with a febrile seizure are Treatment with long-term anticonvulsants does not lower the long-term risk of developing epilepsy

Correct Answer ( D ) Explanation: Children with simple febrile seizures have a 2% to 3% chance of developing epilepsy, compared with a 1% rate of epilepsy in the general population. Children with complex febrile seizures have a significantly higher risk. Treatment with long-term anticonvulsants does not affect the long-term risk of developing epilepsy and is rarely warranted.

A 7-year-old boy presents with five days of abdominal pain, arthralgias, and a rash of the lower extremities. Mild diffuse abdominal tenderness is noted on examination. The left knee and right ankle are swollen and tender without warmth or erythema. The rash consists of purple, non-blanching macules and papules of both legs and thighs. Which of the following is indicated? Abdominal ultrasound to evaluate for intussusception Antinuclear antibody to evaluate for rheumatologic disease Complete blood count to evaluate for thrombocytopenia Urinalysis to evaluate for glomerulonephritis

Correct Answer ( D ) Explanation: The above patient's constellation of a purpuric rash, abdominal pain, arthralgia, and arthritis is concerning for Henoch-Schönlein Purpura (HSP), also known as IgA vasculitis. HSP is a small vessel vasculitis that typically follows upper respiratory infection by one to three weeks. The first symptom of the vasculitis is typically the rash, which may begin as macules or urticaria prior to evolving into palpable purpura. The abdominal pain of HSP results from vasculitis of the gastrointestinal vasculature and may rarely precede the development of the HSP rash. The gastrointestinal vasculitis may also serve as a lead point for intussusception, which presents with severe, intermittent, colicky abdominal pain. The arthritis of HSP is typically migratory and oligoarticular and most commonly affects the joints of the lower extremities. While swelling and tenderness of the affected joints is common, erythema and warmth are typically absent. The arthritis of HSP may also precede the rash by a few days. Glomerulonephritis is also a common feature of HSP. Thus, this patient requires a urinalysis to evaluate for glomerulonephritis, specifically to evaluate for blood, casts, and protein. Renal manifestations of HSP vary widely. While 20-30% of children develop gross hematuria, others develop a range of manifestations from asymptomatic hematuria or non-nephrotic range proteinuria to acute renal insufficiency. The child must be screened for development of renal disease over at least six months following presentation, as renal manifestations of HSP can continue to develop after other signs and symptoms have resolved. 91% of children who develop renal disease will present within six weeks and 97% by six months. If urinalysis is normal six months after presentation, there is no further risk of permanent renal damage. A complete blood count to evaluate for thrombocytopenia (C) is not necessary in a patient with HSP. While purpura may be caused by thrombocytopenic conditions such as Immune Thrombocytopenic Purpura, this patients constellation of acute onset of purpura, abdominal pain, and arthritis is more consistent with HSP.

An 8-year-old girl presents with complaints of difficulty walking over the past few days. She initially had numbness and tingling in her hands and feet which then progressed to a heavy, weak feeling in her legs. Physical exam is notable for the absence of deep tendon reflexes. Which laboratory finding is most consistent with this patient's diagnosis? Elevated CSF WBC count, normal protein Elevated serum WBC count with significant lymphocytosis Hematuria and proteinuria Normal CSF WBC count, elevated protein

Correct Answer ( D ) Explanation: The patient has Guillain-Barré Syndrome (GBS). Guillain-Barré Syndrome is a postinfectious demyelinating neuropathy that may occur after a viral respiratory (especially Epstein-Barr virus) or gastrointestinal infection (especially Campylobacter jejuni) and, rarely, immunizations. It is characterized by progressive motor weakness and areflexia. The progressive weakness begins in the lower extremities and ascends symmetrically ultimately leading to respiratory failure and cranial nerve disorders. Many patients also have subjective paresthesias in their hands and feet. Diffuse pain is common in children younger than 6 years of age. Autonomic instability (tachycardia, orthostatic hypotension) can also occur anytime during the illness. The CSF is often normal in the first few days of illness, but eventually shows albuminocytologic dissociation (elevated protein without pleocytosis). Children should be cared for in a pediatric ICU. Close monitoring of respiratory function is essential as respiratory insufficiency may occur rapidly. Monitoring oxygen saturation is inadequate. The negative inspiratory force (NIF) and vital capacity should be measured frequently. If the vital capacity falls below 25% of normal, the patient should be intubated. Treatment is mainly supportive. Adjunctive therapies include IVIG and plasmapheresis. Corticosteroid therapy is ineffective. Prognosis is generally good, but 10% have permanent disability.

A 2-year-old boy presents with a one-day history of inspiratory and expiratory stridor. According to his mother, he has a fever of 38.3°C at home and a cough that sounds like a barking seal. On evaluation, the child is smiling and playful. He is sitting upright, and his voice is hoarse but not muffled. He is not drooling. His heart rate is 138 beats per minute, respiratory rate 28 breaths per minute, temperature 38.5 ºC rectally, and oxygen saturation of 98% on room air. Which of the following is the most likely diagnosis? Bacterial tracheitis Epiglottitis Foreign body aspiration Retropharyngeal abscess Viral croup

Correct Answer ( E ) Explanation: This child has viral croup. It is the most common cause of stridor in children after the neonatal period. Peak incidence is at two years of age. Symptoms are the result of upper airway inflammation. Patients generally have a viral prodrome preceding onset of croup consisting of viral URI symptoms. A barking cough follows this and lasts for up to four days. Patients can have inspiratory and expiratory stridor. Symptoms are worse at night and typically improve with exposure to cold air. The diagnosis is made clinically. If there is concern for a more serious diagnosis, a soft tissue AP and lateral neck film and PA and lateral chest X-ray can be obtained. The subglottic area on the AP neck film will sometimes show the classic "steeple sign" due to symmetric narrowing of the trachea. Treatment is mainly supportive. In mild croup, supplemental oxygen can be given; fluid losses are replaced as needed, and oral dexamethasone is given. In moderate to severe croup, racemic epinephrine is administered as well. Disposition is guided by response to treatment. Children who receive racemic epinephrine aerosol should be observed in the ED for 3-4 hours following its administration to ensure that they do not return to their pretreatment stridorous state once the racemic epinephrine wears off.

A 6-week-old boy presents with a 3-week history of progressive non-bilious vomiting. There is no history of recent fever, diarrhea or blood in his stool. The physical exam reveals minimal weight gain over the past two weeks. Which of the following is the most likely diagnosis? Intussusception Jejunal atresia Midgut volvulus Pyloric stenosis

orrect Answer ( D ) Explanation: Pyloric stenosis is more common in boys however; a maternal history of pyloric stenosis increases the risk more than a paternal history. It is characterized by nonbilious vomiting, which is the initial symptom. The vomiting may or may not be projectile initially but is usually progressive, occurring immediately after feeding. The vomiting usually starts after 3 weeks of age, but symptoms can develop as early as the first week of life, and as late as the fifth month. After vomiting, the infant is hungry and wants to feed again. As vomiting continues, a progressive loss of fluid, hydrogen ion, and chloride leads to hypochloremic metabolic alkalosis with hypokalemia. Hyperbilirubinemia is also observed, most often unconjugated. On exam, a mass may be palpated above and to the right of the umbilicus in the mid-epigatrium beneath the liver's edge. The "olive mass" is best palpated after an episode of vomiting. A peristaltic wave may also be seen across the patient's abdomen after feeding. The diagnosis is most often made by ultrasound and treatment is surgical with a pyloromyotomy. Intussusception (A) occurs when a portion of the bowel telescopes into an adjacent segment. Ileocolic is the most common portion. This intestinal obstruction is most common between 3 months and 6 years of age. Most are idiopathic. Symptoms include severe colicky pain that tends to recur frequently with legs or knees flexed and straining efforts along with crying during the episode. Subsequently, lethargy is often noted. Vomiting is present early-on and infants may pass stool containing blood and mucus (currant jelly stool). An abdominal mass may be palpated in the RUQ. Intrauterine vascular accidents are generally thought to be the cause of jejunal atresia (B) where infarcted tissue is reabsorbed in the fetal intestine. Abdominal distension typically occurs within 12-24 hours of life after initiation of feedings. Vomiting also occurs and is often bilious. No masses are typically felt. Midgut volvulus (C) should be suspected in any infant with bilious vomiting. Most symptomatic intestinal malrotations are found in the first month of life. Initial symptoms are subtle and may be limited to irritability and progressive bilious vomiting. Late symptoms are abdominal distension and peritonitis. Guaiac-positive stool is usually present. No mass is usually felt on exam.

One Step Further Question: Which antiepileptic medication can cause hirsutism and gingival hyperplasia?

Answer: Phenytoin.

One Step Further Question: Forceful contraction of which muscle is responsible for causing the most common type of pelvic avulsion fracture?

Answer: Hamstring muscle.

One Step Further Question: What is the most common cause of lower intestinal obstruction in neonates?

Answer: Hirschsprung Disease.

One Step Further Question: What type of voice is associated with epiglottitis?

Answer: Hot potato voice.

One Step Further Question: What are the criteria for ultrasound diagnosis of pyloric stenosis?

Answer: Pyloric muscle thickness >4 mm and pyloric length > 14 mm.

One Step Further Question: What criteria on ultrasound are consistent with pyloric stenosis?

Answer: Pyloric wall greater than 4 mm wide or 14 mm long.

One Step Further Question: What unique GI finding is associated with Kawasaki disease?

Answer: Hydrops of the gallbladder; which is characterized by RUQ pain, jaundice, and boggy gallbladder on abdominal ultrasound.

One Step Further Question: What is a common side effect of ceftriaxone in neonates?

Answer: Hyperbilirubinemia.

One Step Further Question: What GI disorder is associated with erythromycin use in neonates?

Answer: Hypertrophic pyloric stenosis.

One Step Further Question: What is seen on electron microscopy of the glomerulus in patients with poststreptococcal glomerulonephritis?

Answer: Immune deposits in the subepithelial space

One Step Further Question: What is the gold standard of treatment in slipped capital femoral epiphysis?

Answer: In situ pinning with a single, large screw.

One Step Further Question: What pulmonary sequela may occur with dengue fever?

Answer: Increased capillary permeability allows pleural effusions to form as a sequela

One Step Further Question: What electrocardiogram finding is associated with macrolide antibiotic use?

Answer: QT interval prolongation

One Step Further Question: Which medication is used to close a patent ductus arteriosus, especially in a premature infant?

Answer: Indomethacin, a prostaglandin inhibitor.

One Step Further Question: Which manifestation of Kawasaki disease is typically the last to appear?

Answer: Indurated edema of the dorsal hands and feet.

One Step Further Question: How is a positive Galeazzi test defined?

Answer: Limb-length discrepancy.

One Step Further Question: What is the most common mechanism of injury in radial head subluxation?

Answer: Longitudinal traction of the arm with the elbow extended.

One Step Further Question: What rescue medication is used for upper airway obstruction?

Answer: Racemic epinephrine.

One Step Further Question: Endocarditis can be reduced in patients with ventricular septal defects with which non-pharmaceutical practice?

Answer: Maintaining good oral hygiene.

One Step Further Question: What important pathology classically presents with bilious emesis in infants?

Answer: Malrotation with volvulus must be suspected with bilious vomiting in infants.

One Step Further Question: What is the most common intracranial complication of acute otitis media?

Answer: Meningitis.

One Step Further Question: What is the preferred route of medication administration if intravenous or intramuscular access is not available for a seizing child?

Answer: Rectal; diazepam is the agent of choice for this purpose.

One Step Further Question: What is the most common etiologic agent of bronchiolitis?

Answer: Respiratory syncytial virus (RSV).

One Step Further Question: What is the most common virus associated with the development of acute bronchiolitis?

Answer: Respiratory syncytial virus (RSV).

One Step Further Question: What is the most common location of foreign body aspiration?

Answer: Right mainstem bronchus.

One Step Further Question: What sexually transmitted infection can present with a rash similar to pityriasis rosea?

Answer: Secondary syphilis

One Step Further Question: What type of toxin results in hemolytic uremic syndrome?

Answer: Shiga-like toxin.

One Step Further Question: How many café-au-lait macules are required for a diagnosis of neurofibromatosis?

Answer: Six.

One Step Further Question: What is the unique skin feature of Ehlers Danlos Syndrome?

Answer: Skin laxity or loose skin.

One Step Further Question: What abnormal heart sounds would be heard with Ebstein's anomaly?

Answer: Split S2 and a systolic murmur.

One Step Further Question: What radiographic sign is present in both croup and bacterial tracheitis?

Answer: Steeple sign.

One Step Further Question: What is the most common cause of meningitis in adult patients?

Answer: Streptococcus pneumoniae.

One Step Further Question: What is the characteristic sign of duodenal obstruction on abdominal radiograph?

Answer: The "double-bubble" sign.

One Step Further Question: What is the most concerning differential diagnosis for a febrile seizure?

Answer: Meningitis. Patients at greatest risk for meningitis are those under 18 months, those with a focal or prolonged seizure, a seizure in the ED, or who have seen a physician within the prior 48 hours.

One Step Further Question: What is the treatment of choice for trichomoniasis?

Answer: Metronidazole.

One Step Further Question: Enteroviruses are the most common cause of viral meningitis in populations immunized against what disease?

Answer: Mumps.

One Step Further Question: What is the normal respiratory rate of newborns and 1-year-olds?

Answer: Newborns 50 breaths per minute, 1-year-olds 30 breaths per minute

One Step Further Question: What is the classic radiographic abnormality seen in aortic coarctation?

Answer: The "figure-3" sign, characterized by prestenotic aortic dilation, coarctation indentation and poststenotic aortic dilation.

One Step Further Question: What is the classic pathognomonic finding seen on ultrasonography in patients with intussusception?

Answer: The "target" or "bull's eye" sign.

One Step Further Question: What radiographic sign is usually seen in patients with epiglottitis?

Answer: The "thumb sign"; typically seen on lateral C-spine X-ray

One Step Further Question: What is the difference between Crigler-Najjar syndrome types I and II?

Answer: The amount of UGT activity.

One Step Further Question: What is the treatment for absence seizures?

Answer: The most common medications used are ethosuximide and valproic acid.

One Step Further Question: What is the treatment for a fused labia minora?

Answer: Topical estrogen cream.

One Step Further Question: If macrolide antibiotics are contraindicated in a patient with suspected pertussis, what is the second line alternative medication?

Answer: Trimethoprim/Sulfamethoxazole.

One Step Further Question: At what ages is the DTaP vaccine recommended?

Answer: Two months, four months, six months, 15 to 18 months and four to six years.

One Step Further Question: What type of Salter-Harris fracture is present in slipped capital femoral epiphysis?

Answer: Type 1.

One Step Further Question: How long do tissue adhesives typically remain on the wound?

Answer: Typically, 5-10 days.

One Step Further Question: What is the initial study of choice for a patient with acute testicular pain?

Answer: Ultrasound with Doppler.

One Step Further Question: What is breast milk jaundice?

Answer: Unconjugated hyperbilirubinemia caused by a molecule in breast milk that increases enterohepatic recirculation of bilirubin. It presents after the first 3-5 days of life and peaks about 2 weeks after birth.

One Step Further Question: What is the most common cause of treatable subfertility in men?

Answer: Varicocele; present in 15% of men. This can be treated with Varicocelectomy.

One Step Further Question: What is the most common congenital heart disorder?

Answer: Ventricular septal defect.

One Step Further Question: Pellagra is caused by a deficiency in what nutrient?

Answer: Niacin. Pellagra is also known as niacin (vitamin B3) deficiency. Pellagra presents with a symmetric hyperpigmented rash, in the exposed areas of skin. The rash is a photosensitive dermatitis and thus only appears where there is sun exposure. The patient may also have a red tongue, diarrhea, vomiting, insomnia, anxiety, disorientation, delusions, dementia, and encephalopathy. Niacin deficiency is very rare in the Western world but may be seen with alcoholism, anorexia nervosa, gastric bypass or malabsorptive diseases.

One Step Further Question: Name two disorders that may present with hypertrophic cardiomyopathy in infancy?

Answer: Noonan syndrome and Pompe disease.

One Step Further Question: What is the most common treatment of molluscum contagiosum?

Answer: Observation and reassurance of spontaneous resolution.

One Step Further Question: What is the most common clinical manifestation of Kawasaki disease aside from fever?

Answer: Oral mucous membrane changes.

One Step Further Question: Which test reduces a dislocated hip in a newborn with developmental hip dysplasia?

Answer: Ortalani's maneuver.

One Step Further Question: Children with hereditary spherocytosis are at risk for aplastic crisis as result of an infection from which virus?

Answer: Parvovirus B19.

One Step Further Question: What chromosomal abnormality is Hirschsprung disease commonly associated with?

Answer: Down syndrome (trisomy 21).

One Step Further Question: What syndrome is associated with a VSD that results in right-to-left shunting?

Answer: Eisenmenger syndrome. Progressive high pulmonary vascular pressure alters flow from left to right through the VSD to right to left, leading to cyanosis.

One Step Further Question: What is the most common cause of pediatric hypertension?

Answer: Essential, or primary, hypertension is most common in both adults and pediatrics.

One Step Further Question: What is the microscopic appearance of Neisseria gonorrhoeae?

Answer: Gram negative intracellular diplococci, or "kissing cocci."

One Step Further Question: At what age should a child begin to take independent walking steps?

Answer: 12 months.

One Step Further Question: At which age is Tetralogy of Fallot typically surgically corrected?

Answer: 3 years.

One Step Further Question: What is the VACTERL condition?

Answer: A constellation of genetic congenital abnormalities: vertebral anomalies, anal atresia, cardiovascular anomalies, tracheoesophageal fistula, esophageal atresia, renal anomalies and limb defects.

One Step Further Question: What is the EMG finding in Guillain-Barré?

Answer: Acute denervation of the muscle.

Which of the following conditions would cause a positive Kussmaul's sign on physical examination? Left ventricular failure Pulmonary edema Coarctation of the aorta Constrictive pericarditis

Constrictive pericarditis Question 1 Explanation: Kussmaul's sign is an increase rather than the normal decrease in the CVP during inspiration. It is most often caused by severe right-sided heart failure; it is a frequent finding in patients with constrictive pericarditis or right ventricular infarction. Hint: Left ventricular failure results in the back-up of blood into the left atrium and then the pulmonary system so it would not be associated with Kussmaul's sign. Hint: Pulmonary edema primarily results in increased pulmonary pressures rather than having effects on the venous inflow into the heart. Hint: Coarctation of the aorta primarily affects outflow from the heart due to the stenosis resulting in delayed and decreased femoral pulses; it has no effect on causing Kussmaul's sign.

One Step Further Question: What is the Klein line in relation to SCFE?

Answer: A line drawn along the superior border of the femoral neck that should intersect the femoral head. If it does not, suspect SCFE.

One Step Further Question: How does a popliteal cyst differ in an adult patient versus a pediatric patient?

Answer: A popliteal cyst in a child usually does not communicate directly with the knee joint.

One Step Further Question: What is the name of the surgery used to correct hypertrophic pyloric stenosis?

Answer: A pyloromyotomy, known as the Ramstedt procedure.

One Step Further Question: What is the Miller Fisher variant?

Answer: A variant of Guillain-Barré Syndrome that is associated with gait ataxia, areflexia, and ophthalmoplegia. Antibodies to G1QB ganglioside are found in >90% of patients who have this disorder

One Step Further Question: What common viral infection is associated with intussusception?

Answer: Adenovirus.

One Step Further Question: How does one differentiate a pulmonary versus cardiac source of cyanosis?

Answer: Administration of oxygen to cyanotic infants with pulmonary disease typically raises pulse oximetry to 95% whereas in cardiac disease the oxygen saturation will likely remain below 90%.

One Step Further Question: What condition should be suspected in a dehydrated neonate with hyperkalemic, hyponatremic metabolic alkalosis?

Answer: Adrenal crisis.

One Step Further Question: What dangerous complication of parvovirus B19 infection is seen in patients with sickle cell disease?

Answer: Aplastic anemia.

One Step Further Question: What is the most common complication of Rubella infection?

Answer: Arthropathies and arthritis.

One Step Further Question: What two other disorders are associated with Tourette's syndrome?

Answer: Attention-deficit hyperactivity disorder (ADHD) and obsessive-compulsive disorder (OCD).

One Step Further Question: What is the mechanism of thrombocytopenia in idiopathic thrombocytopenic purpura?

Answer: Autoantibodies directed against platelets lead to platelet destruction.

One Step Further Question: Failure to treat slipped capital femoral epiphysis can lead to what complication?

Answer: Avascular necrosis.lipped Capital Femoral Epiphysis Obese African-American male adolescents (11-13) Left > right Limp, hip or groin pain Painful/↓ hip internal rotation AP/lateral X-ray of bilateral hips: abnormal Klein line, Bloomberg's sign Non-weight-bearing, orthopedic consultation Hip or Groin Pain

One Step Further Question: What medical procedure puts uncircumcised boys at risk for developing paraphimosis?

Answer: Catheterization of the urethra during for which the foreskin has to be retracted.

One Step Further Question: What is the treatment of choice for children with developmental dysplasia of the hip whom are older than 6 months of age?

Answer: Closed reduction.

One Step Further Question: What is the treatment for Milaria rubra?

Answer: Cooling baths and avoidance of overheating.

One Step Further Question: Can staph scalded skin syndrome be spread to other body parts by rupture of the bullae?

Answer: No, the fluid inside the bullae is sterile. The toxin is produced at a remote site and delivered to the skin via the bloodstream.

One Step Further Question: What is the classic ECG finding in a patient with an anomalous coronary artery?

Answer: Q waves in lead I and aVL.

One Step Further Question: What happens to the C3 and C4 complement levels in serum sickness?

Answer: They are markedly decreased.

One Step Further Question: What is one location of osteomyelitis in adults where hematogenous seeding is the most common mechanism of infection?

Answer: Vertebral osteomyelitis. Genitourinary and respiratory tract infections, IV drug abuse, and sickle cell disease all increase the risk of vertebral osteomyelitis. Infection is usually via hematogenous seeding of bacteria.

One Step Further Question: Which malignancy is associated with Epstein Barr Virus?

Answer: Burkitt's lymphoma.

12-year-old Hispanic girl is seen in the clinic with her parents because of painful joints. History reveals that arthralgias started six months ago which affected both knee joints and all metacarpophalangeal joints of the left hand. She has associated fatigue and occasional fever. Physical examination reveals oral mucosal ulcers, discoid rash with malar distribution, and vasculitic rash on the toes. Complete blood count showed white blood count 6.1 x 109/L, neutrophils 4 x 109/L, bands 0.9 x 109/L, lymphocytes 0.5 x 109/L, monocytes 0.4 x 109/L, eosinophils 0.3 x 109/L, and platelets 84 x 109/L. Which of the following tests is most specific for the diagnosis of the patient's condition? Anti-ribonucleoprotein antibody Anti-Smith antibody Antihistone antibodies Antinuclear antibody (ANA)

Correct Answer ( B ) Explanation: The patient is suffering from systemic lupus erythematosus (SLE). SLE is a chronic autoimmune disease characterized by multisystem inflammation and the presence of circulating autoantibodies directed against self-antigens. SLE occurs in both children and adults, disproportionately affecting females of reproductive age. Prevalence of SLE is highest among African-Americans, Asians, Hispanics, Native Americans, and Pacific Islanders for both adult and pediatric populations. SLE predominantly affects females, with reported 2-5:1 ratio prior to puberty, 9:1 ratio during reproductive years, and return to near prepubertal ratios in the postmenopausal period. Childhood SLE is rare before 5 years of age and is usually diagnosed in adolescence, with a median age at diagnosis of 11-12 years. Histologic features most suggestive of SLE includes findings in the kidney and skin, especially the discoid rash. The characteristic discoid rash is characterized on biopsy by hyperkeratosis, follicular plugging and infiltration of mononuclear cells into the dermal-epidermal junction. The most common presenting complaints of children with SLE include fever, fatigue, hematologic abnormalities, arthralgia and arthritis. Arthritis is usually present in the first year of diagnosis. It is often a symmetric polyarthritis affecting large and small joints. Presence of at least 4 out of the 11 criteria set by the American College of Rheumatology (ACR) 1997 Revised Classification for SLE, either simultaneously or cumulatively over time, establishes the diagnosis of SLE. The criteria includes the following: malar rash, discoid rash, photosensitivity, oral or nasal ulcers, arthritis (nonerosive, ≥ 2 joints), serositis (pleuritis, pericarditis or peritonitis), renal manifestations (consistent renal biopsy, persistent proteinuria or renal casts), seizure or psychosis, hematologic manifestations (hemolytic anemia, leukopenia, lymphopenia, or thrombocytopenia), immunologic abnormalities (positive anti-double-stranded DNA or anti-Smith antibody, false positive rapid plasma reagin test result, positive lupus anticoagulant test result or elevated anticardiolipin IgG or IgM antibody) and positive antinuclear antibody test. Four of the eleven criteria are present in this girl which establishes the diagnosis of SLE. Of the various immunologic tests, anti-Smith antibody is the most specific (about 98%) for the diagnosis of SLE, but it is not sensitive (40-65%).

One Step Further Question: How can you distinguish between bronchiolitis and chlamydial pneumonia?

Answer: Chlamydial pneumonia is typically associated with a staccato cough without fever at one to three weeks of age, while bronchiolitis will cause fever.

One Step Further Question: How is roseola diagnosed?

Answer: Clinically.

A father brings his 2-week-old newborn to the ED after a gagging episode at home where the infant "turned blue." The newborn was sleeping in his father's arms when he started choking, turned blue, and went limp. The father turned the baby over, did 5 back blows, and performed CPR for 5 minutes until the newborn started crying. On exam, the newborn appears sleepy but is easily arousable. Vital signs are HR 160, RR 30, T 37.6°C, and pulse ox is 99% on room air. Which of the following is the next best step in management? Admit to hospital for further workup Endotracheal intubation Epinephrine (1:10 000) IV Send home with reassurance

Correct Answer ( A ) Explanation: Brief resolved unexplained events (BRUE) [formerly ALTE] is an unexpected frightening episode characterized by some combination of apnea, color change, muscle tone change, choking or gagging, and fear that the child has died. The etiology is identified in only 50% of cases. The management of BRUE is variable, and admission criteria may vary between institutions. However, most experts generally agree that any infant who required resuscitation should be admitted for further workup. BRUE is not a diagnosis but a description of an event. Underlying causes include CNS infection, seizure, gastroesophageal reflux, intracranial hemorrhage, botulism, airway obstruction, electrolyte abnormality, and sepsis.

A 4-day-old girl presents to clinic for hospital follow up. She was born by spontaneous vaginal delivery. Pregnancy and delivery were uncomplicated, and she was discharged with her mother on the second day of life. Her mother is exclusively breastfeeding and reports that it is going well. Examination is normal except for jaundice from the face to the lower abdomen. Which of the following clinical features would support a diagnosis of breastfeeding jaundice in this patient? Dark, sticky stools Gestational age of 38 weeks Scleral icterus Weight loss of 3% from birth weight

Correct Answer ( A ) Explanation: Elevated serum bilirubin occurs in nearly all neonates during the first week of life. Bilirubin is a product of the breakdown of heme, which is primarily derived from red blood cells. Clinically, elevated bilirubin manifests as jaundice. Jaundice classically progresses from head-to-toe and is often most noticeable in the sclera and sublingual areas, particularly in infants with darker skin pigment. The most common etiology of jaundice is physiologic jaundice. Physiologic jaundice results from normal immaturity of the conjugation enzymes of the liver, increased hematocrit, shortened red blood cell lifespan, and increased enterohepatic circulation. All of these contributing factors naturally mature in otherwise healthy infants. In addition to physiologic jaundice, infants who are exclusively breastfed may experience breastfeeding jaundice. During the first few days of life, a mother may produce suboptimal amounts of breast milk, resulting in decreased nutrition and hydration of the neonate. Signs of low intake and dehydration include low urine output, meconium stools, weight loss, increased skin turgor, and excessive sleepiness. Meconium stools are first stools produced by an infant, and they consist primarily of sloughed gut mucosa. Meconium appears as dark, sticky stools. As the infant begins to digest breast milk, meconium stools will transition into green and then seedy yellow stools. When the infant's stools do not transition from meconium to green for several days, it indicates a relative stasis of stool within the intestines, permitting more time for enterohepatic recirculation of bilirubin. Once the infant begins to feed larger volumes, stools are expected to transition in color, and intestinal transit time should decrease. Infants with breastfeeding jaundice require a careful feeding history and assessment of breastfeeding, including positioning, latch, and efficacy of milk expression. Referral to a lactation specialist may be considered for challenging situations. Breastfeeding should be encouraged to occur at least every 2-3 hours and even more often if the infant desires to cluster feed. In addition to providing hydration and nutrition for the infant, breastfeeding will stimulate increased milk production in the mother. The physician should provide anticipatory guidance regarding signs of dehydration. A total serum bilirubin can be obtained to assess the infant's proximity to phototherapy levels based on the physician's clinical judgment.

Which of the following congenital heart diseases would benefit from palivizumab administration? Cyanotic congential heart disease Patent ductus arteriosus Pulmonic stenosis Secundum atrial septal defect

Correct Answer ( A ) Explanation: Palivizumab is a humanized monoclonal antibody administered intramuscularly to prevent respiratory syncytial virus (RSV) infections. RSV is the most common cause of lower respiratory tract infections in infants and children worldwide. Children with congenital heart disease (CHD) who develop RSV infections tend to have a higher rate of ICU admissions and require mechanical ventilation more frequently than do children without CHD. Therefore, it is important to prevent RSV infection in this special group. However, not every type of CHD will require palivizumab. CHD conditions that would benefit from palivizumab include cyanotic or complex CHD, congenital heart failure requiring medications, and moderate to severe pulmonary hypertension.

A 5-day-old former 39-week neonate is brought to clinic by her concerned mother for jaundice extending from her face down to her chest. The infant was born by cesarean section for prior maternal cesarean section. Her postnatal course has been uncomplicated, and she was discharged with her mother on the third day of life. She did not require phototherapy. She was told that she and the baby had the same blood type. She is exclusively breastfeeding on demand and mother feels that it is going well. The infant is having yellow-green stools after each feed and has a wet diaper approximately every four hours. She has lost 4% from birth weight. What is the most appropriate next step in management? Further assessment of breastfeeding Obtain total serum bilirubin, hematocrit, and direct antibody testing Phototherapy Transition to formula feeding

Correct Answer ( A ) Explanation: Unconjugated hyperbilirubinemia occurs in nearly every neonate, and it results in some level of jaundice. Jaundice typically progresses from head-to-toe and may be most noticeable in the sclera, especially in children with darker skin pigmentation. Physiologic jaundice is a normal process that results from increased hematocrit, decreased red blood cell lifespan, decreased activity of the enzymes of bilirubin conjugation, and increased enterohepatic recirculation. Infants who are exclusively breastfed are at risk of breastfeeding jaundice, which results from dehydration due to low intake of breastmilk during the first days of life. The low intake often results from suboptimal maternal production of breastmilk, but the infant may also have difficulty with initiating and sustaining a latch during feeding. Signs of dehydration and suboptimal intake include decreased urine output, persistent meconium stools, increased skin turgor, and excessive weight loss. The described infant is having normal urine output and transitional stools and has an acceptable degree of weight loss. However, she is at risk of breastfeeding jaundice, and, as such, further assessment of breastfeeding is suggested. The assessment should include a more detailed description of the frequency of breastfeeding, the duration of feeds, and signs of effective milk extraction from the breast, which include emptying of the breasts during feeds, milk noted on the infant's lips, and an audible swallow. To ensure adequate hydration and nutrition, breastfeeding should occur at least every 2-3 hours over the first several weeks of life, including overnight. Cluster feeding is also common during the first weeks of life.

Which of the following is a minor rather than a major Jones criterion in the diagnosis for acute rheumatic fever? Fever New murmur Polyarthritis Subcutaneous nodules

Correct Answer ( A ) Explanation: Rheumatic fever is a known delayed sequela of infection with group A Streptococcal infection. Fever is a minor criterion for acute rheumatic heart disease. The Jones criteria, created in 1944 and updated in 1992, are used to diagnose acute rheumatic fever (ARF). Rheumatic heart disease is an important yet uncommon sequela of rheumatic fever. Acute rheumatic fever incidence is declining in the U.S. and other developed countries and reported to be fewer than 20 cases per 100,000 individuals in school aged children. Diagnosis of ARF is possible using one major plus two minor Jones criteria or two major Jones criteria. Major criteria include carditis (new murmur), polyarthritis, chorea, erythema marginatum (non-pruritic, erythematous rings, found on trunk and inner limbs), and subcutaneous nodules (painless collagen collections on back of wrists, elbows, and knees). Minor criteria include arthralgias, fever, elevated acute phase reactants (erythrocyte sedimentation rate and C-reactive protein).

A previously healthy 6-month-old girl presents to clinic with parental concern for fatigue and weakness. Two days ago, she began having sleepiness with drooping eyelids. Since then, she has been having trouble sucking on her bottle and has seemed too weak to eat from a spoon. Her mother has tried multiple interventions to get her to feed. She has tried adding a variety of natural sweeteners to her formula and homemade canned fruits to encourage feeding. Her mother is worried that her lack of intake has caused her entire body to become weak and floppy, and she has not had a bowel movement in five days. On examination, the infant is sleepy. The anterior fontanelle is soft but slightly sunken. Respirations are shallow. Neurological examination is significant for diffuse hypotonia, including bilateral ptosis and ophthalmoplegia. The hypotonia is more severe in the upper extremities than the trunk and lower extremities. What is the most likely etiology of the infant's hypotonia? Botulism Guillain-Barré syndrome Neonatal myasthenia gravis Prader Willi

Correct Answer ( A ) Explanation: The differential diagnosis of hypotonia and weakness is broad, including both acute and chronic neurological and muscular disorders. In this infant, the onset of symptoms has been both acute and progressive. The infant's ptosis, ophthalmoplegia, and difficulty in feeding are suggestive of cranial nerve involvement. Her new-onset hypotonia is more prominent in her upper extremities than lower extremities, raising concern for descending weakness. Although her decreased stools may be related to poor oral intake, they may also be caused by decreased neuromuscular tone and decreased peristalsis of the gastrointestinal tract. Her shallow respirations may indicate diaphragmatic weakness. The mother's mention of homemade canned goods and natural sweeteners merits further exploration, as both home-canned goods and honey may be contaminated with Clostridium botulinum spores. In infants, C.botulinum spores may germinate to colonize the gastrointestinal tract. The bacteria then release a neurotoxin that prevents presynaptic neurotransmitter release, resulting in infantile botulism. Symptoms of botulism typically progress similarly to the symptoms of this child. Weakness and hypotonia descend from head to toe and commonly involve the cranial nerves. Feeding is often limited by weakness of the oropharyngeal muscles. Constipation is common. The child requires immediate transfer to a tertiary care center where she can receive botulism immune globulin Neonatal myasthenia gravis (C) results from transmission of maternal antibodies to the fetus. Although signs and symptoms of neonatal myasthenia gravis may be similar to those of botulism, they are almost always present within the first three days of life. In addition, the weakness of myasthenia gravis is typically fatigable, meaning it becomes more prominent with repetitive motions. This child's diffuse hypotonia and weakness are not suggestive of myasthenia gravis. Prader Willi (D) is a genetic disorder that causes diffuse hypotonia and failure to thrive in infants. The hypotonia of Prader Willi is chronic throughout infancy. The acuity and progression of this infant's hypotonia are not suggestive of Prader Willi.

A 17-year-old girl is seen in clinic due to vaginal discharge. She complains of yellow-green discharge and pruritus around her vaginal area. She is sexually active since last year and has had three partners. She denies taking any medication and uses condoms occasionally. Her last menstrual period was last week. On physical exam, you note frothy discharge with vaginal erythema and cervical hemorrhages. Which of the following is the next best step? Obtain a wet mount Obtain Herpes simplex virus PCR Obtain Rapid plasma Reagin (RPR) Provide reassurance and discharge home

Correct Answer ( A ) Explanation: The patient has signs and symptoms of acute cervicitis with findings most likely due to trichomoniasis. It presents with malodorous vaginal discharge, vulvovaginal irritation, dysuria, and dyspareunia. Physical exam may reveal a frothy discharge with vaginal erythema, and cervical hemorrhages (strawberry cervix) with the latter being a classic finding. The discharge usually has a pH of > 5. Trichomonas vaginalis may be recognized in vaginal secretions by using the wet mount technique which has a sensitivity of 60-70%. If motile trichomonads are not found, this should be sent for culture, which is the gold standard for diagnosis. Patients with trichomoniasis should be screened for other sexually transmitted infections such as chlamydia and gonorrhea. Metronidazole and tinidazole are used for treatment.

A 16-year-old girl presents to the ED for a minor laceration repair of her forehead after a picture frame fell off the wall and hit her. Her vital signs are blood pressure 175/75 mm Hg, HR 80, and RR 14. The patient states on review of systems that she has had headaches, chest pain, and fatigue over the previous few months. You note a systolic murmur in the left infraclavicular area and under the left scapula. Which of the following is the most important next step in management? Arrange for outpatient follow-up for repeat blood pressure Obtain blood pressure readings in the upper and lower extremities Order a renal ultrasound to evaluate for fibromuscular dysplasia of the renal arteries Send electrolytes and urinalysis in preparation to begin antihypertensive therapy

Correct Answer ( B ) Explanation: Coarctation of the aorta usually presents as congestive heart failure and cardiogenic shock in the neonatal period due to increased afterload caused by narrowing of the aorta. In mild cases, patients can develop arterial collateral vessels that partially bypass the aortic obstruction. These patients usually remain asymptomatic. The diagnosis is often made when incidental hypertension is noted during the evaluation of other problems such as trauma or routine illness. In older children, adolescents, and adults, coarctation of the aorta is best diagnosed clinically by simultaneous palpation of femoral and brachial pulses. Blood pressure in both arms and one leg must be determined; a pressure difference of more than 20 mm Hg in favor of the arms may be considered evidence of coarctation of the aorta. The chest radiograph in these patients may demonstrate rib notching secondary to collateral vessels.

A 3-year-old boy presents with severe vomiting and diarrhea. The exam reveals sunken eyes, skin tenting, and a capillary refill of three seconds. At his last well visit he weighed 15 kg, but weighs just 13 kg today. Using the Holliday-Segar method, what is his maintenance fluid rate per hour? 260.0 ml/hr 39.0 ml/hr 47.9 ml/hr 83.3 ml/hr

Correct Answer ( C ) Explanation: 47.9 ml/hr is calculated utilizing the Holliday-Segar method of 100 ml/kg/day for the first 10 kg, 50 ml/kg/day for the next 10 kg, and 20 ml/kg/day for each additional kilogram. The total is then divided by 24 hours. (100 ml/kg/day x 10 kg) + (50 ml/kg/day x 3kg) = 1150 ml/kg/day/24hr = 47.9ml/hr This method is sometimes shortened to the 4-2-1 method of 4 ml/kg/hr for the first 10 kg, 2 ml/kg/hr for the next 10 kg, and 1 ml/kg/hr for each additional kilogram. (4 ml/hr x 10 kg) + (2 ml/hr x 3 kg) = 46 ml/hr If isonatremic, sodium and potassium maintenance would need to be calculated, and added into this maintenance fluid for a proper order to be written.

A continuous systolic-diastolic murmur is auscultated in a 3-week-old dyspneic infant. She is tachypneic and diaphoretic. Her mother reports weight loss and poor feeding. Doppler color-flow imaging reveals high velocity jets in the pulmonary artery. This patient will most likely undergo which of the following corrective surgeries? Arterial-switch Ligation Shunting Valvulotomy

Correct Answer ( B ) Explanation: Patent ductus arteriosus (PDA) is an acyanotic congenital cardiac condition. In this condition, the fetal connection between the pulmonary artery and aorta remains open, causing a continuous, "machine-like" murmur that is heard in both systole and diastole. Diagnosis is confirmed by appreciating pulmonary artery high-velocity jetting during Doppler echocardiography. Significant cases commonly result in labored breathing, symptoms of congestive heart failure and failure to thrive. Symptomatic PDA is treated surgically, with ligation being the most preferred method of closure. The arterial-switch (A) procedure is used to correct great artery transposition, not PDA. In tetralogy of Fallot, a shunt (C) is placed between the subclavian and adjacent pulmonary arteries. Shunts are not used in treating PDA. Valvulotomy (D) is used to correct aortic or pulmonary stenosis, two other common congenital cardiac defects.

Closure of the ductus arteriosus begins when levels of which of the following substances increases in the immediate post-natal period? Adrenocorticotrophic hormone Bradykinin Prostaglandin E1 Pulmonary surfactan

Correct Answer ( B ) Explanation: Patent ductus arteriosus (PDA) is one of the acyanotic congenital cardiac defects. Prematurity is a major risk factor. It is also associated with Down's syndrome and maternal rubella infection. Fetal blood flow occurs normally through the ductus arteriosus from the pulmonary artery to the aorta, thus bypassing the pulmonary vasculature. When this passageway remains patent after birth, it can lead to symptoms of persistent respiratory difficulty and work of breathing (dyspnea), recurrent respiratory infections, tachycardia, cardiomegaly, bounding pulses (widened pulse pressure), machine-like murmur and poor weight gain. After the first breath, pulmonary vascular resistance decreases and stimulates bradykinin release from the infant's lungs. This causes smooth muscle contraction about the ductus arteriosus, beginning the narrowing process until this structure scars close to become the ligamentum arteriosum.

A 3-year-old boy presents to the ED with three days of fever, cough, and runny nose. On exam, you note conjunctival injection and an erythematous, nonblanching, nonvesicular, maculopapular rash behind his ears and on his hairline, with a few spots on his chest. Which of the following is the most likely diagnosis? Roseola Rubella Rubeola Varicella

Correct Answer ( C ) Explanation: Rubeola, or measles, is associated with fever and rash with cough, conjunctivitis, coryza, and Koplik spots. The characteristic rash is erythematous, nonblanching, and maculopapular. It begins on the head, usually behind the ears and around the hairline, with subsequent spreading down the face, to the trunk, and extremities (centrifugal spread). The rash may coalesce into salmon-colored patches and typically disappears within one week. Koplik's spots or pinpoint-sized white lesions on a red background that appear on the buccal mucosa opposite the molars are pathognomonic.

A 2-year-old boy presents to the Emergency Department with his grandmother due to concern for dehydration. He has had two wet diapers and eight watery, malodorous, green stools in the past 24 hours. He has been extremely thirsty and drinking plenty of water and juice. Serum chemistries are significant for a sodium 151 mEq/L, chloride 117 mEq/L, potassium of 3.0 mEq/L, and bicarbonate of 19 mEq/L. Urinalysis is significant for 1+ ketones but is otherwise normal. What is the most likely cause of this child's hypernatremia? Diabetes insipidus Diarrhea Salt poisoning Water deprivation

Correct Answer ( B ) Explanation: The child presents with dehydration and labs significant for hypernatremia, hyperchloremia, hypokalemia, and an anion gap metabolic acidosis. Hypernatremia may result from one of two broad categories: excessive water loss or excessive salt intake. This child's history is significant for poor urine output in conjunction with diarrhea. Diarrhea results in excessive free water loss if it is hypotonic. The hypernatremia may be augmented if diarrheal losses are primarily replaced with hypotonic solutions such as water, soda, or juice. Diarrhea also results in potassium losses, as well as loss of bicarbonate. Of note, the metabolic acidosis of diarrhea alone typically does not result in an elevated anion gap. However, dehydration may cause lactic acidosis, which causes an anion gap metabolic acidosis. The child's hyperchloremia is likely a result of compensation for his bicarbonate losses. Diabetes insipidus (A) may result in hypernatremic dehydration. Although the child is very thirsty, he does not display the typical polyuria and polydipsia of diabetes insipidus. His polydipsia is more likely secondary to his diarrheal losses than diabetes. Salt poisoning (C) is a rare cause of hypernatremia. It often results from inadequate formula mixing in young infants, or it may result from intentional over administration of salts as a form of medical child abuse. Salt poisoning is very low on the differential of a child with hypernatremic dehydration and diarrhea. Water deprivation (D) is also a very rare cause of hypernatremia in children. Per the grandmother's history, this child is actually drinking significant water in an effort to compensate for his excessive stool losses.

A 12-year-old previously healthy adolescent presents to clinic with one week of cough and fatigue. He has also developed intermittent fevers with a Tmax of 101°F. His cough and fatigue have been progressively worsening. Breath sounds are coarse throughout all lung fields. What is the treatment of choice? Amoxicillin Azithromycin Cefdinir Oseltamivir

Correct Answer ( B ) Explanation: The most likely etiology of pneumonia in adolescents are atypical bacteria such as Mycoplasma pneumoniae and Chlamydia pneumoniae. Atypical bacterial pneumonia, also known as "walking pneumonia," has a more gradual onset and indolent course than typical bacterial pathogens such as Streptococcus pneumoniae. Symptoms of atypical bacterial pneumonia often begin with generalized malaise and progress to cough and fever. Breath sounds may be diffusely coarse and include scattered crackles or wheezes, but focal findings are typically not present. Chest film is likely to reveal perihilar and interstitial infiltrates. The treatment of choice for atypical bacterial pneumonia is five days of azithromycin. Tetracyclines may also be used in rare cases of azithromycin allergy.

An 18-month-old boy is brought in by his parents for shortness of breath. The parents woke to him coughing a low-pitched cough. They also noted other noises when he was breathing in that resolved upon walking outside. The patient is frequently coughing but has no abnormal sounds on auscultation of the neck or lungs. What is the most appropriate treatment? Albuterol Dexamethasone Racemic epinephrine Ribavirin

Correct Answer ( B ) Explanation: This patient is suffering from croup, or laryngotracheobronchitis. Croup is a viral infection of the upper airway, most commonly caused by parainfluenza virus. It is the most common cause of upper airway distress and obstruction in children. The peak incidence of croup is 2 years and the classic age range of children affected is 6 months to 6 years. Most cases occur in late fall, winter and early spring. Patients have a characteristic barking cough that sounds like a seal. Symptoms occur because of edema and exudate of the subglottic space. Croup is a clinical diagnosis based on the presence of the barking cough, hoarse voice and sometimes stridor. Preceding the development of these symptoms, children may have had a prodrome of fever and URI symptoms. The administration of dexamethasone is the mainstay of therapy and has been shown to decrease the duration of symptoms, decrease return visits to the ED, decrease length of stay in the ED and decrease the need for epinephrine. In patients with stridor, the administration of racemic epinephrine leads to rapid improvement of symptoms in most cases. Patients who have received racemic epinephrine need a period of observation after treatment for 1-2 hours to ensure that stridor and retractions do not recur. On X-ray, the steeple sign is present due to a narrowing of the upper trachea from the infection.

A 3-year-old boy presents with severe vomiting and diarrhea. The exam reveals sunken eyes, skin tenting and a capillary refill of 3 seconds. He weighs 13 kg. He was 15 kg at his last well visit. Using the Holliday-Segar method, what is his maintenance fluid rate per hour? 260 ml/hr 39 ml/hr 47.9 ml/hr 83.3 ml/hr

Correct Answer ( C ) Explanation: 47.9 ml/hr is calculated utilizing the Holliday-Segar method of 100 ml/kg/day for the first 10 kg, 50 ml/kg/day for the next 10 kg and 20 ml/kg/day for each additional kilogram. This total is then divided by 24 hours. (100 ml/kg/day x 10 kg) + (50 ml/kg/day x 3kg) = 1150 ml/kg/day/24hr = 47.9ml/hr This method is sometimes shortened to the 4-2-1 method of 4 ml/kg/hr for the first 10 kg, 2 ml/kg/hr for the next 10 kg and 1 ml/kg/hr for each additional kilogram. (4 ml/hr x 10 kg) + (2 ml/hr x 3 kg) = 46 ml/hr If isonatremic, sodium and potassium maintenance would need to be calculated and added into this maintenance fluid for a proper order to be written.

A four-year-old child presents to the emergency room with high fever, barking cough, stridor and moderate retractions at rest. Which of the following is the most appropriate treatment for this condition? Albuterol nebulizer Ceftriaxone Nebulized racemic epinephrine Oral oseltamivir

Correct Answer ( C ) Explanation: A child with moderate croup has a barking cough, stridor, and moderate retractions at rest. Nebulized epinephrine is the treatment of choice as the rapid local vasoconstriction of the subglottic mucosa reduces swelling and provides rapid improvement. Oral or intravenous steroids should be administered as well to prevent the progression or airway edema as well as rebound edema that may occur up to three hours after the administration of racemic epinephrine. Humidified oxygen can also be given to help control airway edema. Evidence is poor, but in severe cases, children may benefit from heliox. This is a mixture of helium and oxygen which is less dense than air. It assists in air flow through a narrowed airway. In patients that require intubation, an uncuffed endoctracheal tube at least 0.5 mm narrower than the standard for age should be used given concern for significant airway swelling.

An 18-month-old boy presents to the emergency department with two days of cough, low-grade fevers, and congestion. Today the cough has become hoarse and barking. What is the most likely diagnosis? ABronchiolitis BCommunity acquired pneumonia CCroup DLaryngeal foreign body aspiration

Correct Answer ( C ) Explanation: Croup is an infection of the larynx and trachea and is most commonly caused by parainfluenza virus. It is most common in children 6 to 36 months of age. The illness typically begins as classic symptoms of an upper respiratory tract infection, such as coryza, cough, and fever. Symptoms may then progress to a barking cough, hoarseness, and inspiratory stridor due to upper airway obstruction. Stridor may occur solely with activity or with both activity and rest. Stridor that occurs at rest indicates significant upper airway obstruction. Bronchiolitis (A) is inflammation of the bronchioles due to viral infection, most commonly Respiratory Syncytial Virus. While children with bronchiolitis begin with symptoms of upper respiratory congestion and cough, they do not exhibit the classic hoarseness or barking cough associated with croup. In fact, nasal and chest congestion is prominent features of bronchiolitis, and as such, the cough is often wet sounding. Community acquired pneumonia (B) is a common pediatric diagnosis. Symptoms may begin as a classic upper respiratory tract infection before progressing to signs of lower respiratory tract infection. Hoarseness and a barking cough are not classic features of Community Acquired Pneumonia

Highlight Strike Erase Flag Normal LabValues «PreviousNext» A one-year-old girl is seen in your clinic for a routine visit. Her mother states that she has been eating well and drinking more than 32 ounces of whole milk a day. You order blood work and find a hemoglobin level of 9 g/dl and a mean cell volume of 74. What is the most likely diagnosis? Beta thalassemia trait Hereditary spherocytosis Iron deficiency anemia Lead poisoning

Correct Answer ( C ) Explanation: Dietary iron deficiency is most common between the ages of one and three years of age. This is because cow's milk is a very poor source of iron. A milk intake greater than 16 ounces a day raises the concern of iron deficiency. Milk is associated with satiety and delayed gastric emptying, which limits the consumption of other iron-containing foods. In these patients, the physical exam may only be significant for mild pallor. Universal screening at one year of age is routinely done to identify and address iron deficiency before leading to severe side effects, such as developmental and mental delay. Initial hemoglobin studies may be normocytic and normochromic and then progress to hypochromic and microcytic as the low iron levels begin to take effect. Iron studies done for verification will show a low iron level and high total iron-binding capacity (TIBC). Treatment of iron deficiency anemia depends on the etiology. Children who have gastrointestinal bleeding should have imaging and possible endoscopy to determine the etiology. In dietary iron deficiency, parents should be counseled to limit whole milk intake to less than 16 ounces a day. Treatment consists of the administration of oral iron sulfate. This is continued until one month after hemoglobin levels normalize. The additional one month allows for the replenishment of the body's iron stores.

A 10-year-old boy presents after noticing red streaks in his stool that began yesterday. He also complains of crampy periumbilical pain and his stools have been looser than normal since yesterday. A stool guaiac is positive. Exam is notable for non-blanching pinpoint macules on his ankles that appear slightly urticarial. What is the most likely etiology of the child's hematochezia? Bacterial enteritis Hemolytic uremic syndrome Henoch-Schönlein purpura Meckel's diverticulum

Correct Answer ( C ) Explanation: Henoch-Schönlein Purpura (HSP) is an IgA-mediated vasculitis. The classic rash of HSP is purpuric. It typically begins on the bilateral lower extremities and may initially look petechial or urticarial prior to becoming classic palpable, non-blanching reddish-purple macules and patches. In addition, the vasculitis of HSP may involve the abdominal vasculature as well, resulting in purpura along the intestinal mucosa. The intestinal purpura may cause both abdominal cramping and bloody stools and they may serve as a lead point for intussusception. Arthralgias and arthritis are also typical of early HSP and renal abnormalities may develop during several months following diagnosis. Notably, the skin findings of HSP may be delayed until days after the abdominal pain, hematochezia, or arthritis have developed. As such, clinicians must maintain a high suspicion for a diagnosis of HSP in a child with abdominal pain and rectal bleeding even if purpura or arthritis are not yet apparent on examination. Bacterial enteritis (A) may result in crampy abdominal pain and bloody stools. However, in this child, the concomitant development of a petechial and urticarial rash favors a diagnosis of HSP rather than bacterial enteritis. Hemolytic uremic syndrome (HUS) (B) is a complication of bacterial enteritis, most commonly caused by E.coli O157:H7. HUS results in hemolytic anemia, thrombocytopenia, and acute kidney injury. Although HUS-induced thrombocytopenia may manifest as petechiae, the onset of hemolytic uremic syndrome typically does not occur until symptoms of gastroenteritis have been present for at least five days. An urticarial rash would be very atypical. A Meckel's diverticulum (D) may cause rectal bleeding, but it is typically painless. Moreover, it is not associated with rash.

A 2-year-old boy presents because of high-grade fever of two days and drooling of his saliva. He has been irritable as well and refuses to eat or drink. Physical examination reveals multiple vesicles and ulcers surrounded by an erythematous ring on the uvula, soft palate, anterior tonsillar pillars and posterior pharyngeal wall. Examination is otherwise unremarkable. Which of the following is the most likely diagnosis? Aphthous ulcers Hand, foot and mouth disease Herpangina Herpetic gingivostomatitis

Correct Answer ( C ) Explanation: Herpangina is characterized by sudden onset of high-grade fever, sore throat, dysphagia, and lesions in the posterior pharynx. Characteristic lesions, present on the anterior tonsillar pillars, soft palate, uvula, tonsils, posterior pharyngeal wall, and, occasionally, the posterior buccal surfaces, appear as discrete vesicles and ulcers surrounded by erythematous rings that enlarge over a span of three days. Fever generally lasts up to four days, and resolution of symptoms occurs within seven days. Diagnosis is made clinically and can be confirmed by serologic studies or PCR; however, this is rarely done due to the high index of suspicion in diagnosis. Supportive therapy is the mainstay of treatment, while milrinone proves to be a useful agent in severe enterovirus 71 involving cardiopulmonary disease. Aphthous ulcers (A) are painful, circumscribed ulcerative lesions with a white necrotic base surrounded by a red halo. Hand, foot and mouth disease (B) presents with painful lesions on the tongue, anterior oral cavity, hands, and feet caused by enterovirus. Herpetic gingivostomatitis (D) is characterized by erythematous, mucosal hemorrhages and clusters of small vesicles erupting throughout the mouth.

A 15-year-old boy who runs on his high school cross-country team presents to your office complaining of pain just below his right knee. He states that the pain started 3 months ago and is worse at night. He denies any recent injuries or trauma. You order an X-ray and obtain the image below. Which of the following is the most likely diagnosis? Ewing sarcoma Osgood-Schlatter disease Osteosarcoma Patellar tendonitis

Correct Answer ( C ) Explanation: Osteosarcoma is the third most common malignancy of adolescents and is the most common malignant bone tumor in adolescents. It often occurs around the time of growth spurts (adolescents) and patients usually present after several months of pain. Teenagers who are active in sports tend to complain about pain in their lower femur, or right below the knee. If the tumor is large, it can appear as a swelling. The affected bone is not as strong as normal bones and may fracture with minor trauma. There are no systemic symptoms. Most commonly affected bones are the metaphyses of long tubular bones (femur, tibia or humerus). The classic finding on X-ray is a "sunburst" pattern due to the aggressive lytic bones lesions that form that are associated with periosteal reaction. MRI is the most sensitive test to determine the extent of the tumor, but a tissue sample is required to confirm the diagnosis. Micrometastases are common at presentation so treatment often includes surgical resection of the tumor and chemotherapy.

During a well-child visit, a 9-year-old boy and his father ask about health safety and sports activity. The boy wants to play baseball. His medical history is significant for tetralogy of Fallot, which was surgically corrected when he was 3-years-old. His last echocardiogram shows a right ventricular pressure to be < 50 mm Hg. Which of the following recommendations do you make? A functional capacity evaluation is needed first He may only play leisure sports, like golf or cycling It is safe for him to play any sport, including baseball The boy should abstain from all sports and physical education class

Correct Answer ( C ) Explanation: Physical activity limitations may be recommended for children with a past history of congenital heart disease. There are no restrictions on any sport activity, including competition and contact sports, in patients who have had surgical correction of tetralogy of Fallot and whose right ventricular pressure is < 50 mm Hg.

A 3-year-old girl presents because she is not using her left arm. Her father was playfully swinging her around by her forearms earlier in the day. What is the most likely diagnosis? Posterior elbow dislocation Radial head fracture Radial head subluxation Shoulder dislocation

Correct Answer ( C ) Explanation: Radial head subluxation, also known as nursemaid's elbow, is the most common elbow injury in children. It typically occurs in children ages one to four years and is caused by traction of a pronated forearm while the elbow is extended. This occurs when a parent is swinging a child around while holding onto their hands or forearms, or when pulling a child's arm to prevent the child from falling. Patients with a radial head subluxation present with little distress and hold their arm close to their body with the elbow either fully extended or slightly flexed with a pronated forearm. Edema and ecchymosis at the site are uncommon. Supination of the forearm elicits increased pain. Diagnosis is made clinically, but radiographs may be useful to rule out fracture. Treatment includes reduction of the radial head by supinating and flexing the forearm. Radial head fracture (B) is a fracture of the radius bone in the forearm that most commonly occurs after a fall onto an outstretched arm. Patients typically present with pain, swelling, and tenderness to palpation over the lateral elbow and range of motion may be limited. Radiographic imaging is important to detect a fracture and a "sail sign" seen on X-ray is indicative of an occult, nondisplaced radial head fracture

A 5-year-old boy presents to your office with rapidly developing sore throat, high fever, restlessness, lethargy and a muffled voice.The patient appears uncomfortable and is in a sitting position leaning forward. A lateral extended neck radiograph shows evidence of a "thumbprint" sign. Which of the following is the most likely diagnosis? Airway foreign body Croup Epiglottitis Peritonsillar abscess

Correct Answer ( C ) Explanation: Signs and symptoms of epiglottitis include rapidly developing sore throat, high fever, restlessness, and lethargy. A "supraglottic," muffled voice is common. Many patients have difficulty with their saliva and drool. Classically, these patients are in a sitting position leaning forward, because this position tends to alleviate obstructive symptoms from the supraglottic swelling. They may show signs of "air hunger" or may have stridor. Epiglottitis (or "supraglottitis") is a condition that requires prompt attention by the physician. Epiglottitis results from bacterial (and rarely viral) infection of the supraglottic structures (epiglottis and arytenoid cartilages). A high level of suspicion is necessary to make a diagnosis and avoid significant morbidity. Patients may rapidly decompensate due to airway compromise. The physician should always be suspicious when a patient presents with fever, sore throat, and difficulty swallowing. Croup, tonsillitis, peritonsillar abscess, and other neck infections may be incorrectly diagnosed in these patients. Epiglottitis occurs mainly in children age 2 to 7 years, although infants, older children, and adults can be affected. Mortality rates of 6% to 7% have been reported in adults. The incidence has significantly decreased since the use of the Hib vaccine. Most cases are now seen in the adult population.

An asymptomatic 4-year-old boy presents for a routine check-up. On examination you hear a systolic heart murmur that is heard best in the lower precordium and has a low and short tone. It does not radiate and decreases in intensity with inspiration. The remainder of the examination is normal. Which one of the following is the most likely diagnosis? Eisenmenger's syndrome Mitral stenosis Still's murmur Venous hum

Correct Answer ( C ) Explanation: Still's murmur is a benign murmur of childhood. It can occur at any age, but is most commonly heard between ages 2 and 8 years. The murmur usually resolves spontaneously by adulthood. It has no association with physiologic or anatomic abnormalities. It is low-grade mid-systolic murmur localized near the apex to lower left sternal border. It decreases in intensity with inspiration, sitting up, or standing. Most pathologic murmurs do not change significantly with standing except the murmur associated with hypertrophic cardiomyopathy. The sound is often described as musical or having a relatively pure tone in quality or squeaky. Because they are low pitched, they are best heard with the bell of the stethoscope. The cause of Still's murmur is unknown, but it may be due to vibrations in the chordae tendinae, semilunar valves, or ventricular wall. Venous hum (D) consists of a continuous low-pitched murmur caused by the collapse of the jugular veins and their subsequent fluttering, and it worsens with inspiration or diastole.

A 1-year-old boy is brought to the emergency department for severe abdominal pain, vomiting, and diarrhea. His mother says he has had several episodes of intermittent abdominal pain occurring about every 15 to 20 minutes. He appears to be in distress and is continuously crying. Physical examination reveals a "sausage-shaped" abdominal mass palpated on the right side of the abdomen. Which of the following is the most likely diagnosis? AGastroenteritis BHirschsprung disease CIntussusception DMeckel diverticulum

Correct Answer ( C ) Explanation: The constellation of severe intermittent abdominal pain, vomiting, diarrhea, and a "sausage-shaped" abdominal mass most likely suggests intussusception. Intussusception refers to the invagination (telescoping) of a part of the intestine into itself. It is the most common abdominal emergency in early childhood, particularly in children younger than two years of age. Patients typically present with sudden onset of intermittent, severe, crampy, progressive abdominal pain, accompanied by inconsolable crying. The episodes usually occur at 15 to 20 minute intervals. Palpation of the right side of the abdomen usually reveals a "sausage-shaped" mass. Treatment usually involves surgery for patients that are acutely ill. Meckel diverticulum (D) is considered the most common congenital abnormality of the small bowel. It is caused by incomplete obliteration of the vitelline duct and typically presents with painless rectal bleeding.

A 6-week-old infant is brought to the Emergency Room with parental concern for one week of cough and congestion. Although her cough has worsened, she remains afebrile. Chest radiography reveals bilateral interstitial infiltrates and hyperexpansion. What is the treatment of choice? Amoxicillin Cefotaxime Erythromycin Reassurance and return precautions

Correct Answer ( C ) Explanation: The infant has classic findings of pneumonia caused by Chlamydia trachomatis. The onset C.trachomatis pneumonia typically occurs between two and nineteen weeks of life, and almost all affected infants have symptoms by eight weeks of life. Symptoms often begin with nasal congestion and progress to a "staccato cough" and tachypnea. A lack of fever is classic and helps to differentiate chlamydial pneumonia from other causes of pneumonia in this age group. Examination typically reveals rales, but wheezing is uncommon. Chest radiography shows hyperinflation and interstitial infiltrates. The treatment of choice is fourteen days of oral erythromycin.

At a 4-week well-child check, a mother raises concern for a rash on her infant's scalp. For the last two weeks, the scalp has been shedding greasy scales, and now the area anterior to the ears has begun to scale, too. No family members have the rash. The family has a dog and cat a home. Which of the following is the most likely diagnosis? Atopic dermatitis Langerhans cell histocytosis Seborrheic dermatitis Tinea capitis

Correct Answer ( C ) Explanation: The infant's greasy scaling is consistent with seborrheic dermatitis, also known as Cradle Cap. Seborrheic dermatitis is very common amongst infants and typically presents with greasy scaling and erythematous plaques. The rash clusters around areas with high concentrations of sebaceous glands such as the scalp, ears, center of the face ("T zone"), and diaper area. Less commonly, the folds of the neck and axilla may be involved, and the rash in these areas tends to be more of an erythematous plaque than scaling. Seborrheic dermatitis is a benign and self-resolving skin condition, but its appearance may cause significant distress to families. A non-medical treatment option is applying an emollient such as mineral oil or petroleum jelly to soften the scales, followed by combing them out with a fine-tooth comb or brush. Medical options include a short course of low-potency topical steroids or ketoconazole 2% shampoo. Parents should be reassured that most cases fully resolve in 12 months even without therapy.

A 4-year-old boy presents to clinic with his mother who is concerned about a lump in the back of his knee which was first noticed two months ago. The patient has no history of injury to the knee and he does not complain of pain. On physical exam, the patient has a mass in the popliteal fossa that is nontender to palpation, not warm to the touch, and non-erythematous. The patient has full range of motion of the knee. The mass transilluminates. Which of the following is the most appropriate management? Arthrocentesis and intra-articular injection with glucocorticoids Aspiration of the cyst Observation Surgical excision

Correct Answer ( C ) Explanation: The patient's history and clinical exam findings are consistent with a benign popliteal (Baker's) cyst. Popliteal cysts in children are not typically associated with trauma and are asymptomatic in most cases. The mass is usually located medially and distal to the knee crease and is more apparent with extension of the knee. An ultrasound can be obtained to confirm the diagnosis. Observation is the mainstay of treatment. The family should be counseled that no intervention is needed and the majority of cases will resolve spontaneously.

A 12-year-old girl presents for her annual well child check. She had menarche eight months prior, and has had menses every month. Each cycle lasts two weeks. The patient uses at least six pads on most days of the cycle. On exam, she is pale but otherwise well appearing. Her heart rate is 80 beats per minute, blood pressure 110/65, and respiratory rate 14. The remainder of her exam is within normal limits. Which of the following are the most likely laboratory findings? Hemoglobin 12 g/dL, mean corpuscular volume 80, RDW 10 Hemoglobin 6 g/dL, mean corpuscular volume 60, RDW 12 Hemoglobin 6 g/dL, mean corpuscular volume 60, RDW 18 Hemoglobin 8 g/dL, mean corpuscular volume 100, RDW 10

Correct Answer ( C ) Explanation: This patient has iron deficiency anemia secondary to dysfunctional uterine bleeding. Iron deficiency anemia is characterized by a microcytic anemia (low hemoglobin and low MCV). Approximately two percent of adolescent females experience iron deficiency anemia due to menstrual blood loss and the adolescent growth spurt. Most children with iron deficiency anemia are asymptomatic. The most common sign is pallor, which usually does not occur until the hemoglobin level is < 8 g/dL. Iron studies in this patient show reduced iron stores (low ferritin), increased iron-binding capacity (high transferrin and elevated TIBC), and decreased transferrin saturation. The mean corpuscular volume (MCV) describes red blood cells as microcytic, normocytic, or macrocytic. A normal MCV in an adolescent female is approximately 80. The red cell distribution width (RDW) is a measure of the variability in size of red blood cells. Iron deficiency anemia is characterized by an elevated RDW (11.5-14.5).

A 9-year-old boy presents with an inability to retract his foreskin. He has not been able to urinate for two days. The visible portion of the glans appears dusky. Which of the following therapies is most appropriate? Circumcision Corticosteroid cream Dorsal slit procedure Intracavernosum phenylephrine

Correct Answer ( C ) Explanation: This patient presents with phimosis and evidence of vascular compromise requiring performance of a dorsal slit procedure. Phimosis occurs when the foreskin of an uncircumcised penis becomes constricted preventing retraction of the prepuce from the glans. It results in urinary outlet obstruction, glans ischemia and infarction. Many cases are physiologic resulting from normal development. Patients typically present with an unretractable foreskin and symptoms consistent with urinary obstruction including decreased urinary stream. Management of phimosis can be difficult. Dilation of the prepuce can be performed using forceps but often does not result in relief of phimosis. When there are signs of glans vascular compromise (i.e. discoloration) a dorsal split procedure should be performed. During this procedure, the foreskin is anesthetized and incised dorsally allowing for retraction.

A 3-week-old infant presents with projectile vomiting. Mom reports he has vomited after each feed for the last 24 hours. What electrolyte abnormality do you expect to see? Hyperchloremic, hyperkalemic metabolic alkalosis Hyperchloremic, hypokalemic metabolic alkalosis Hypochloremic, hyperkalemic metabolic alkalosis Hypochloremic, hypokalemic metabolic alkalosis

Correct Answer ( D ) Explanation: Hypertrophic pyloric stenosis is the most common cause of intestinal obstruction in infants beyond one month of age. Classically, infants present between 2 and 6 weeks of age with progressive emesis that ultimately becomes projectile. Since the stenosis is at the level of the pylorus, emesis is non-bilious. Infants finish a feeding and then regurgitate the entire volume of feeding and continue to be hungry. On examination of the abdomen, the hypertrophic pylorus may be palpable in the upper abdomen and is often described as an olive. With continued vomiting, infants lose both hydrogen ions and chloride present in the gastric juices. As the condition continues, renal compensation occurs with an exchange of hydrogen ions for potassium leading to the metabolic alkalosis. The resulting metabolic condition is a hypochloremic, hypokalemic, metabolic alkalosis. Ultrasound is the diagnostic modality of choice demonstrating a thickened pylorus. If an upper GI series is performed with contrast, the "string sign" may be present as a small amount of contrast passes through the hypertrophied pylorus.

Which of the following is a contraindication to the use of air-contrast enema in the reduction of pediatric intussusception? Air contrast enema for a prior episode of intussusception in the last 24 hours Fever greater than 39°C Lethargy Peritonitis

Correct Answer ( D ) Explanation: Intussusception is the invagination of a bowel segment into a distal segment of bowel. It is the most common cause of intestinal obstruction in early childhood. The majority of cases of pediatric intussusception are idiopathic, but risk factors include conditions that cause a lead point for intestinal telescoping. The classic triad of intussusception is intermittent abdominal pain, bloody "currant jelly" stools, and a sausage-shaped abdominal mass. However, children may present only with progressive lethargy and altered consciousness. In uncertain presentations, the diagnosis should be confirmed with ultrasound, which classically shows a "target sign." Children with high suspicion for intussusception should undergo air-contrast enema, which is both diagnostic and therapeutic. Contraindications to the use of air-contrast enema include hemodynamic instability with shock, free air under the diaphragm, and peritonitis. Children with these features need emergent surgical intervention.

A 15-year-old boy is brought by his father to your clinic for evaluation of a limp. This is accompanied by right hip and right knee pain, which he has noticed for about two weeks after he slipped while trying to sit on a chair. He denies fever and does not participate in sports. On physical exam, his weight is greater than the 95th percentile, and the right hip appears externally rotated with restriction of internal rotation, abduction, and flexion. Antero-posterior and frog-leg radiographs reveal widening of the right physis and a crescent-shaped area of increased density of the femoral neck. Which of the following is the most likely diagnosis? Developmental dysplasia of the hip Hip fracture Legg-Calve-Perthes disease Slipped capital femoral epiphysis

Correct Answer ( D ) Explanation: The boy has clinical and radiographic findings that are consistent with slipped capital femoral epiphysis (SCFE). SCFE involves failure of the physis and displacement of the femoral head relative to the neck that commonly affects adolescents aged 11 to 16 years. Acute SCFE presents with prodromal symptoms, occurring for less than three weeks. The patients complain of groin, thigh, or knee pain and may report a relatively minor injury, which is not sufficiently violent to produce an acute fracture of this severity. Chronic SCFE presents with a few-month history of vague pain and limp. Patients can have a limp with a lower extremity that is externally rotated. Acute-on-chronic SCFE presents with features of both conditions. Physical examination findings in SCFE show a restriction of internal rotation, abduction, and flexion. Diagnosis is made using the radiographic antero-posterior and frog-leg views. Findings include irregularity and widening of the physis, decrease in epiphyseal height, a crescent-shaped area of increased density in the proximal portion of the femoral neck, and a double density created from the anteriorly displaced femoral neck overlying the femoral head. Legg-Calve-Perthes disease (C) is the syndrome of idiopathic avascular necrosis of the femoral epiphysis showing fragmentation, sclerosis, and subchondral collapse of the ossification center of the femoral head on radiographs.

A 2-year-old girl presents to the emergency department for ingestion of a penny. An older brother witnessed the event at 5:00 pm. She is currently asymptomatic. A chest X-ray at 7:00 pm shows a round radiopaque object in the mid esophagus. Other than keeping the child from eating, what is the next step in management? Encourage the patient to drink water to push the coin into the stomach Immediately remove the object endoscopically Induce vomiting with syrup of ipecac Repeat chest X-ray in 4 hours

Correct Answer ( D ) Explanation: The chest X-ray should be repeated to evaluate movement of the coin. About 90% of blunt objects ingested by children pass through without intervention. Ingestions of blunt objects can be observed for 24 hours after which time it should be removed if it has failed to pass through the stomach. If the object passes through the stomach, the patient can be observed for another week without intervention. On chest X-ray coins appear in the coronal view if lodged in the esophagus and in the sagittal view if in the trachea. Foreign body ingestion peaks in children aged 6 months to 3 years. The majority of children are asymptomatic. Common places for obstruction include the cricopharyngeal area, the middle third of the esophagus, the lower esophageal sphincter, the pylorus and the ileocecal valve. While metal objects can be detected on X-rays, wooden, plastic and glass objects as well as fish bones may not be seen on radiographs.

A 6-year-old boy presents to the Emergency Department with periorbital swelling. His mother notes that for the past three days the periorbital swelling would appear in the morning and gradually decrease throughout the day. The boy also presents with loss of appetite, abdominal pain, and loose bowel movements. The physical examination reveals an fussy child with blood pressure of 100/70 mmHg and temperature of 37oC. He has periorbital edema, anicteric sclerae, pink palpebral conjunctiva, clear breath sounds, a nontender abdomen, no bipedal edema, and capillary return less than 2 seconds. Urinalysis is positive for 3+ proteinuria and red blood cell count of 10 cells/hpf. Serum cholesterol and triglyceride levels are elevated with serum albumin less than 2.5 g/dL. What is the initial drug of choice for the most likely diagnosis? Cyclophosphamide Cyclosporine Levamisole Prednisone

Correct Answer ( D ) Explanation: The child presents with manifestations of nephrotic syndrome, the most common type of which is minimal change nephrotic syndrome. Other types include mesangial proliferation, focal segmental glomerulosclerosis, membranous nephropathy, and membranoproliferative glomerulonephritis. It is more common in boys than in girls and occurs between 2 to 6 years of age. Children commonly present with periorbital edema which is worse in the morning and gradually decreases throughout the day. Hypertension and gross hematuria, while more common in glomerulonephritis, rarely occur in idiopathic nephrotic syndrome. Laboratory results are distinctive and include 3+ or 4+ proteinuria and microscopic hematuria seen on the urinalysis. A spot urine protein:creatinine ratio exceeds 2, and urinary protein excretion exceeds 50 mg/kg/day. There is also hypoalbuminemia with serum albumin level of less than 3 g/dL and hyperlipidemia with elevated serum cholesterol and triglycerides. Prednisone is given at a dose of 60 mg/m2/day in a single daily dose for 4 to 6 consecutive weeks and is gradually tapered for the next two months. Eighty to ninety percent of children with minimal change nephrotic syndrome respond well to steroid therapy within the first five weeks of treatment.

A 3-year-old boy presents with left anterior cervical lymphadenitis and fever that developed over three days. The area surrounding the node is tender, warm, and swollen. What is the most likely etiology of the lymphadenitis? Atypical mycobacteria Bartonella henselae Epstein-Barr Virus Staphylococcus aureus

Correct Answer ( D ) Explanation: The most common causes of infectious lymphadenitis in children are Staphylococcus aureus and Streptococcus pyogenes. These two pathogens alone cause up to 80% of cervical lymphadenitis in children < 5 years old. Staph and Strep species cannot be differentiated by clinical presentation alone. Thus, appropriate antimicrobial therapy should cover both Strep and Staph species, including methicillin-resistant Staph aureus. Patients who fail to show improvement after 2-3 days of appropriate antibiotics should be evaluated for the development of fluctuance, which indicates a developing abscess. Atypical mycobacteria (A) are a common cause of lymphadenitis. In comparison to the acute onset of Staph or Strep lymphadenitis, the presentation of atypical mycobacterial lymphadenitis is subacute. The involved nodes are classically non-tender and may develop overlying skin changes and draining sinus tracts. The most common location of atypical mycobacterial lymphadenitis is submandibular. Bartonella henselae (B) is the cause of Cat Scratch Disease, which commonly causes axillary or cervical lymphadenitis in children. History includes exposure to kittens, cats, or fleas within the prior weeks. Often an erythematous papule can be found at the inoculation site for several weeks after acquisition of Bartonella. Epstein-Barr Virus (C) mononucleosis is a common cause of fever and lymphadenopathy in children and adolescents. The lymphadenopathy associated with EBV typically is bilateral, involves the posterior cervical lymph nodes, and does not result in overlying skin changes. Sore throat is a common clinical complaint, and exudative pharyngitis is often noted on exam. The lymphadenopathy caused by EBV may also be generalized and may be accompanied by splenomegaly or hepatomegaly.

A 9-year-old girl presents with scalp itching. Physical examination reveals the finding seen in the image above. What is the first-line treatment for this disease? Clotrimazole Ivermectin PO Lindane Permethrin

Correct Answer ( D ) Explanation: This patient presents with pediculosis capitis or head lice and should be treated with permethrin. Pediculosis capitis is caused by lice infection. These parasitic lice infest and lay eggs at the base of the hair shaft. Transmission is from person-to-person. It is common in children but uncommon after puberty. Patients will present with intense pruritus, which coincides with the lice feeding and inspection reveals nits (immature lice) firmly attached to the base of hair shafts. The heaviest infection is typically seen behind the ears. Diagnosis is made on clinical grounds or with microscopy. The preferred treatment for pediculosis capitis is with permethrin, which kills the adult louse. Subsequently, the nits must be removed with a vinegar solution and fine-tooth comb. The scalp should be reexamined one week later and repeat therapy at this point may be required.

A 12-year-old girl presents to the office with anal itching that seems to be worse at night. She has no issues with constipation or any other changes in her bowel habits. On physical exam you see some excoriations around the anus but no tear or palpable hemorrhoid. This first occurred a few days after returning home from a summer camp in northern Michigan, where she was in a cabin with 15 other girls for one month. Which of the following is the most likely diagnosis? Internal hemorrhoid Lyme disease Pinworms Scabies

orrect Answer ( C ) Explanation: Pinworm is the most common helminthic parasite encountered by primary care providers in developed nations. It is acquired by ingesting parasite eggs, and most people remain asymptomatic after being colonized. Pinworm (E. vermicularis) is a quintessential intestinal parasite with no geographic constraints. It is transmissible by close contact with colonized persons. People have had pinworm for thousands of years, and before modern sanitation, colonization by pinworm probably was universal. E. vermicularis has a simple life cycle with a "hand to mouth" existence. The worm is acquired by ingesting parasite eggs. Most often these eggs are on the hands of the host; however, the small eggs also may become airborne, inhaled, and then swallowed. During the night, egg-laden females migrate out of the anal canal and onto the perianal skin. Each female deposits up to 17,000 eggs which mature rapidly, becoming infective within six hours causing extreme pruritus. Infestation typically causes perianal itching and scratching gathers eggs onto the hands, promoting reinfection and transmission to others. Diagnosis is made by the cellophane tape test and all members of the household should be treated. Pinworm infection is readily treated with a single 100-mg dose of mebendazole or a 400-mg dose of albendazole.

One Step Further Question: Which congenital heart diseases cause right ventricular hypertrophy?

Answer: Atrial septal defect, Eisenmenger's syndrome and pulmonic stenosis.

One Step Further Question: What medication is indicated in the treatment of infant botulism?

Answer: Botulism Immune Globulin Intravenous (Human) (BIG-IV) has been shown to reduce hospital length of stay and time on mechanical ventilation.

One Step Further Question: How long before there is a response to oral iron therapy in a patient with iron deficiency anemia?

Answer: Four to eight weeks.

One Step Further Question: What are the microscopic characteristics of Streptococcus pneumoniae?

Answer: Gram-positive cocci with a "lancet shape."

One Step Further Question: What is the treatment of tinea capitis?

Answer: Griseofulvin.

One Step Further Question: What in-office physical exam technique may aid in triggering an absence seizure in susceptible patients?

Answer: Guided hyperventilation.

One Step Further Question: What are complications of Campylobacter gastroenteritis?

Answer: Guillain-Barre syndrome, reactive arthritis, and erythema nodosum.

One Step Further Question: What is the most effective prophylaxis for the prevention of acute otitis externa?

Answer: Instillation of dilute alcohol or acetic acid (2%) immediately after swimming or bathing.

One Step Further Question: How is the Measels virus spread?

Answer: It is spread through infectious droplets and is highly contagious.

One Step Further Question: What oral agent can be used to treat scabies?

Answer: Ivermectin.

One Step Further Question: At what penile length do you diagnose micropenis?

Answer: Less than 2 cm.

One Step Further Question: What is a non-suppurative, post-infectious sequelae of Group A Strep impetigo?

Answer: Post-streptococcal glomerulonephritis.

One Step Further Question: What blood smear finding is suggestive of microangiopathic hemolytic anemia?

Answer: Schistocytes (helmet cells).

One Step Further Question: For what other lung problem are cystic fibrosis patients at risk?

Answer: Spontaneous pneumothorax.

One Step Further Question: What are the most common medications that trigger Stevens-Johnson syndrome?

Answer: Sulfa drugs.

One Step Further Question: What ligament gets displaced during a radial head subluxation?

Answer: The annular ligament.

One Step Further Question: What is the appropriate pediatric dosing for intravenous fluid boluses in hypotensive pediatric patients?

Answer: The dosing is 20 mL/kg.

One Step Further Question: Where are the beta cells located in the pancreas?

Answer: The islets of Langerhans.

One Step Further Question: Is eosinophilia seen in patients with enterobiasis?

Answer: Unlike most other helminthic infections, eosinophilia is not seen in enterobiasis.

One Step Further Question: Which vitamin abnormality is associated with pseudotumor cerebri?

Answer: Vitamin A deficiency or hypervitaminosis A.

Which of the following infants is at highest risk for developmental dysplasia of the hip? A female infant born in the breech position A first-born male with a birth weight of 3200 grams A male infant born in the breech position A second-born male born at 37-week gestatio

Correct Answer ( A ) Explanation: A female infant born in the breech position would have the highest risk for developmental dysplasia of the hip. Developmental dysplasia of the hip (DDH) is a spectrum of abnormal hip development disorders that range from malunion of the acetabulum and proximal femur to mechanical hip instability. DDH typically occurs in otherwise healthy infants. During the newborn period, there is physiological laxity of the hip joint, which resolves in most cases. When the acetabulum is not in contact with the femur, the joint fails to develop normally. Risk factors for abnormal contact between acetabulum and femur include female sex, breech birth position, oligohydramnios, prolonged swaddling, and family history of DDH. Screening for DDH is recommended at birth and at every well-visit until the child begins to walk normally. During the first three months of life, DDH is screened for using the Barlow and Ortolani maneuvers. After three months of age, the Galeazzi and Klisic tests are better indicators of hip instability. In children older than two to three months, limited hip abduction is the most reliable clinical finding. Children who have unilateral DDH and are of walking-age, may have a positive Trendelenburg test. DDH is bilateral in approximately 20 percent of cases. Ultrasonography is the imaging test of choice for evaluating DDH. Plain radiograph use is limited in children younger than six months of age. Splinting of the hips in an abducted position with a soft splint is the treatment of choice for DDH. Orthopedic referral is required in all suspected cases. Prognosis is good with early intervention and treatment.

A newborn male has a skin eruption on his forehead, nose, and cheeks. The lesions are mostly closed comedones with a few open comedones, papules, and pustules. No significant erythema is seen. Which one of the following is the most likely diagnosis? Acne neonatorum Erythema toxicum neonatorum Herpes simplex Milia

Correct Answer ( A ) Explanation: Acne neonatorum occurs in up to 20% of newborns. It typically consists of closed comedones on the forehead, nose, and cheeks, and is thought to result from stimulation of sebaceous glands by maternal and infant androgens. Open comedones, inflammatory papules, and pustules can also develop. Erythema is generally not seen. Acne neonatorum is a clinical diagnosis and parents should be counseled that lesions usually resolve spontaneously within 4 months without scarring. Treatment generally is not indicated, but infants can be treated with a 2.5% benzoyl peroxide lotion if lesions are extensive and persist for several months. Parents should apply a small amount of benzoyl peroxide to the antecubital fossa to test for local reaction before widespread or facial application. Findings in erythema toxicum neonatorum (B) include papules, pustules, and erythema. Erythema toxicum neonatorum is the most common pustular eruption in newborns. Typical lesions consist of erythematous, 2- to 3-mm macules and papules that evolve into pustules. Each pustule is surrounded by a blotchy area of erythema, leading to what is classically described as a "flea-bitten" appearance. Herpes simplex (C) lesions usually present in an ill-appearing neonate with vesiculopustular lesions. Milia (D) consists of 1- to 2-mm pearly keratin plugs without erythema, and may occur on the trunk and limbs.

A neonate is brought to clinic for jaundice. Pregnancy and delivery were unremarkable, and the infant was discharged with her mother on the third day of life. She is exclusively breastfeeding approximately every 2-3 hours. Which of the following features favors a diagnosis of breast milk jaundice over breastfeeding jaundice? Age of 10 days Dark green stools Three wet diapers in the past 24 hours Weight loss of 8% from birth weight

Correct Answer ( A ) Explanation: Almost all neonates develop some degree of unconjugated hyperbilirubinemia during the first week of life. The causes of this physiologic jaundice include increased bilirubin production, decreased bilirubin clearance, and increased enterohepatic recirculation. Additional processes may augment one or more etiologies of physiologic jaundice. Breast milk jaundice and breastfeeding jaundice are two of the most common causes of unconjugated hyperbilirubinemia in otherwise healthy infants. Breast milk jaundice is thought to result from a factor within breast milk that increases the enterohepatic recirculation of bilirubin. The onset of breast milk jaundice is typically in the latter half of the first week of life, with a peak bilirubin level around two weeks of life. The bilirubin then slowly declines over the next several months. Infants with breast milk jaundice continue to feed, void, grow, and develop normally despite their prolonged jaundice. Of the answer choices above, only an age of 10 days is consistent with breast milk jaundice. In contrast, the onset of breastfeeding jaundice is typically within the first few days of life when breast milk production may be suboptimal to provide hydration to the neonate.

A mother brings her 7-year-old boy into the clinic. She reports that the child has been complaining of fatigue, headache and myalgias since he returned home from a camping trip with his father. His mother points out an annular rash with central clearing on the child's torso. Which of the following is the most appropriate treatment for this patient's condition? Amoxicillin Azithromycin Doxycycline Erythromycin

Correct Answer ( A ) Explanation: Amoxicillin is the first line treatment of Lyme disease in children. Early disease usually occurs a few days to one month after an Ixodes tick bite and is typically characterized by an erythema migrans skin lesion with or without nonspecific complaints resembling a viral syndrome. For adults and children ≥ 8 years of age with early Lyme disease, doxycycline, amoxicillin, or cefuroxime is recommended. Doxycycline should be avoided in children < 8 years of age and pregnant or lactating women. Such patients should receive amoxicillin.

A ten-year-old girl presents with drooping eyelids for the past month. The patient reports some difficulty completing her schoolwork due to blurry vision and hand weakness. On initial observation, her exam is notable for bilateral ptosis. Which of the following is the most likely pathophysiology behind her symptoms? Acetylcholine receptor antibodies Acetylcholinesterase deficiency Anti-thyroid antibodies Toxin exposure

Correct Answer ( A ) Explanation: Antibodies to the acetylcholine receptor are the most common cause of myasthenia gravis. Myasthenia gravis is a chronic, autoimmune disease characterized by fatigue of striated muscle. The first clinical signs are ptosis or extraocular muscle weakness or both. Later symptoms include bulbar muscle weakness, limb-girdle weakness, and hand weakness. This disease is notable for the rapid fatigue of muscles, which may be demonstrated by sustaining an upward gaze for 30-90 seconds or repetitive opening and closing of the fists. Symptoms increase throughout the day. Myasthenia gravis is a progressive disease and may be fatal due to respiratory muscle compromise or aspiration. A clinical diagnosis may be made with the administration of edrophonium chloride (Tensilon test), which causes immediate improvement in the ptosis and ophthalmoplegia. In infants, Neostigmine should be used to make the diagnosis, as edrophonium has increased risk of cardiac dysrhythmias in infants. Cholinesterase inhibitors are the first line of therapy. Plasmapheresis and IVIG, as well as thymectomy, can be used in refractory cases. Anticholinesterase deficiency (B) at the motor endplate is a less common cause of myasthenia gravis; this deficiency is associated with familial myasthenia gravis. Myasthenia gravis may be associated with Hashimoto's thyroiditis (C), and thyroid levels should be checked in all patients. Treatment of hypothyroidism usually leads to resolution of myasthenia symptoms. However, this patient does not report concurrent symptoms of hypothyroidism. Toxin exposure (D) must be considered in the differential of neuromuscular blockade. These toxins include organophosphate chemicals, botulism, and tick paralysis. This patient has no history of exposure to these toxins.

A 4-year-old girl is brought to the ED after 48 hours of vomiting and diarrhea. Her parents are concerned that she is becoming dehydrated. Which of the following is a sign of moderate dehydration? Decreased urine output Hypotension Lethargy Mottled extremities

Correct Answer ( A ) Explanation: Assessment of the degree of dehydration is a critical skill within pediatrics, as untreated hypovolemic shock may lead to cardiovascular collapse and death. In 2003, the CDC revised the categories of dehydration to the following: no to minimal dehydration (< 3% loss of body weight), mild-to-moderate dehydration (3-9% loss of body weight), and severe dehydration (> 10% loss of body weight). At less than 3% loss of body weight, most children are able to maintain a normal mental status, heart rate, perfusion, breathing, tear production, moisture of mucous membranes, skin turgor, and urine output. In other words, children with up to 3% fluid loss may have no signs or symptoms of dehydration. Children develop clinical signs of dehydration during progression to 3-9% loss of body weight. Signs and symptoms of progression of dehydration may include the development of fatigue or fussiness, thirst, tachycardia, slightly diminished pulses, prolonged capillary refill, cool distal extremities, tachypnea, slightly sunken eyes with decreased tear production, dry mucous membranes, increased skin turgor, and decreased urine output. As dehydration continues to progress to severe (> 10%), mental status further declines to a lethargic state. The child is markedly tachycardic with thready pulses, very prolonged capillary refill, cold and mottled extremities, deep breathing, very sunken eyes with absent tears, markedly increased skin turgor (> 2 seconds recoil), and minimal urine output.

You suspect a cardiac structural defect in a 6-month-old infant with peripheral edema and hypertension. Skin color is normal. A murmur is not appreciated. However, the blood pressure is markedly higher in the arms than legs. Which of the following is the most likely diagnosis? Aortic coarctation Great artery transposition Patent ductus arteriosus Tetralogy of Fallot

Correct Answer ( A ) Explanation: Congenital cardiac defects can be defined into two classifications: cyanotic and acyanotic. The most common acyanotic lesions are coarctation of the aorta, pulmonary stenosis, aortic stenosis, atrial septal defect, ventricular septal defect, atrioventricular canal and patent ductus arteriosus. Coarctation of the aorta is characterized by narrowing of the aorta anywhere along its length but most commonly around the left subclavian artery takeoff. This acyanotic lesion presents with congestive heart failure, cardiomegaly, hypertension and shock. The classic exam finding is blood pressure higher in the arms than legs, and pulses bounding in the arms and decreased in the legs. Rib notching (inferior aspect of the rib) is the classic radiographic finding in patients with coarctation of the aorta. This occurs as a result of dilatation of intercostal arteries. Surgical correction is usually required, and is most commonly performed between 2 and 4 years of age.

A newborn is being evaluated for cyanosis. Physical examination shows a prominent right ventricular impulse, a systolic thrill, and clubbing of the fingers and toes. A crescendo-decrescendo murmur with a harsh systolic ejection quality is heard along the left upper sternal border. Based on these findings, which of the following is the most likely diagnosis? Tetralogy of Fallot Transposition of the great arteries Tricuspid atresia Truncus arteriosus

Correct Answer ( A ) Explanation: Congenital heart diseases that present with cyanosis include tetralogy of Fallot (TOF), tricuspid atresia, truncus arteriosus, total anomalous pulmonary venous return (TAPVR), and transposition of the great arteries (TGA). Based on the constellation of findings, this newborn most likely has TOF. TOF presents with a ventricular septal defect (VSD), right ventricular hypertrophy (RVH), an overriding aorta, and right ventricular outflow tract obstruction from infundibular stenosis. The murmur in TOF is primarily due to the right ventricular outflow tract obstruction and is usually heard as a crescendo-decrescendo murmur with a harsh systolic ejection quality. The classic pathognomonic finding on chest X-ray is the "boot shaped" heart with an upturned apex. Transposition of the great arteries (B) is an abnormal spatial arrangement of the pulmonary artery and aorta. Cardiac auscultation will typically reveal a loud second heart sound due to aortic valve closure. Tricuspid atresia (C) is complete absence of the tricuspid valve and is typically associated with a holosystolic murmur (due to a VSD). Truncus arteriosus (D) occurs when blood is pumped from the heart through a single truncal valve into a truncal artery, which gives rise to the aorta and the pulmonary arteries. Chest X-ray typically shows an enlarged heart and increased pulmonary vascular markings.

A two-year-old previously healthy boy is witnessed having a tonic-clonic seizure in the emergency room. Mom originally brought him in due to a fever of 39°C. The episode lasted five minutes, and the patient returned to baseline mental status. Which of the following is the most appropriate next step in management? Administer antipyretics Initiate anticonvulsant therapy Perform a lumbar puncture Perform an EEG

Correct Answer ( A ) Explanation: Febrile seizures are seizures that occur between the ages of six months and 60 months with a temperature of 38°C or higher, that are not the result of a central nervous system infection or any metabolic imbalance and that occur in the absence of a history of prior afebrile seizures. A simple febrile seizure is a primary generalized, usually tonic-clonic attack associated with fever, lasting for a maximum of 15 minutes and not recurring within a 24-hour period. A complex febrile seizure is more prolonged (>15 min), is focal or recurs within 24 hours. Treatment is mainly supportive for simple febrile seizures and involves evaluation of the fever source, control of fevers with antipyretics such as acetaminophen (not aspirin), and reassurance of the parents.

3-year-old girl presents to the ED with 1 hour of a barking cough and inspiratory stridor at rest. On exam, she has mild retractions but is not hypoxic. Which of the following interventions has been shown to reduce hospital length of stay in moderate to severe croup? Dexamethasone Heliox Humidified air Racemic epinephrine

Correct Answer ( A ) Explanation: Glucocorticoids are considered the mainstay of treatment for moderate to severe croup. They are the only treatment that alters the natural course of the illness. Dexamethasone has been shown to reduce the use of epinephrine, severity scores at 6 hours, hospital lengths of stay, and readmissions. In some studies, it has also reduced the rate of intubation

What is the most common causative organism of meningitis in a 1-week-old child? Group B Streptococci Haemophilus influenzae Listeria monocytogenes Staphylococcus aureus

Correct Answer ( A ) Explanation: Group B Streptococci (GBS) is the most common causative pathogen for meningitis in neonates. There are many bacteria that can cause meningitis. The most likely causative organism changes based on host factors including age, comorbidities and immune status. It is important to know the most common causes in different patient groups in order to tailor antibiotic regimens. In neonates, group B streptococci account for more than 75% of cases of meningitis. Other common organisms include Streptococcus pneumoniae, Neisseria meningitides and Listeria monocytogenes. GBS is transmitted from the mother during childbirth. As such, rates of GBS have fallen with the wide spread practice of intrapartum prophylaxis. Neonates with meningitis will present nonspecifically with behavioral changes, neurologic changes and vital sign abnormalities. Because of the nonspecific presentations all neonates with a fever should be considered for lumbar puncture to rule out the diagnosis.

A 10-year-old boy has recently moved to the United States from India with his mother. He previously received the Bacille Calmette-Guerin (BCG) immunization. He denies cough, fever, or weight loss. A tuberculin skin test is placed. 48 hours later, the area of induration at the injection site measures 10 mm in diameter. Which of the following is the most appropriate next step in management? Obtain a chest radiograph Place the patient in airborne isolation Reassure the family Repeat the skin test in six months

Correct Answer ( A ) Explanation: Injection-site induration of a tuberculin skin test (TST) is considered positive at greater than or equal to 10 mm diameter of induration in a patient who was born in a region with a high prevalence of tuberculosis. Many patients from high-prevalence regions have received the BCG vaccine prior to arrival in the United States. Receipt of the BCG does not impact the threshold of positivity for the TST. Whether a patient who has received the BCG vaccine will have a positive TST is extremely variable and depends on a number of factors, including age at vaccination, number of doses received, time lapse since immunization, the strain contained in the vaccine, the number of prior TST's, and patient's immune and nutritional status. Thus, it is any patient with a positive tuberculin skin test requires a chest radiograph regardless of vaccination status. Radiographic evidence of granulomas, calcifications, or adenopathy are consistent with tuberculosis and cannot be caused by the BCG vaccine.

A 6-year-old immigrant boy from Bangladesh presents with fever and rash. The fever started 3 days ago followed by a rash which started on the head and spread to the rest of the body. Examination reveals a well appearing child with a maculopapular rash and posterior cervical lymphadenopathy. Which of the following is the most likely causative organism for this disease? Measles virus Mumps virus Parvovirus B19 Rubella virus

Correct Answer ( D ) Explanation: This patient presents with classic symptoms of Rubella or German measles. Rubella is a mild, febrile illness that is associated with a diffuse maculopapular rash. The patient may also present with headache, malaise and posterior cervical and postauricular lymphadenopathy. Rubella is transmitted through respiratory secretions. It is highly communicable. The rash generally begins on the head and face and progresses downwards ("showering" rash). The disease itself is typically self-limited, however, the consequences of maternal infection during pregnancy can be severe for the fetus.

A 10-year-old boy presents to your office with elbow pain. The pain is located along the medial aspect of the elbow and is exacerbated while pitching during his Little League games. Your examination reveals mild swelling along the medial aspect of the right elbow. Radiographs show mild hypertrophy of the medial epicondyle. What is the initial treatment of choice for this condition? Complete rest from throwing for four to six weeks Eccentric exercise regimen for four weeks Orthopedic referral Physical therapy for four weeks

Correct Answer ( A ) Explanation: Little Leaguer's elbow is an inflammation of the apophysis of the medial epicondyle. It usually occurs in athletes between the ages of nine and 12 years old. There is pain along the medial aspect of the throwing arm. Examination often reveals tenderness to palpation, swelling and decreased range of motion along the medial aspect of the elbow. The treatment of choice involves cessation of all throwing activities for four to six weeks, then a gradual and progressive throwing program after the initial period of inactivity. Most players are able to return to throwing full-time after 12 weeks.

A 4-year-old boy who has not received routine childhood vaccinations presents with fever, cough, coryza, and conjunctivitis for three days. Today his mother noted an erythematous macular rash over his face, trunk, and extremities. Which of the following is the most likely diagnosis? Measles Mumps Roseola Rubella

Correct Answer ( A ) Explanation: Measles is spread by direct contact with infectious droplets or airborne dissemination. From exposure to the onset of symptoms takes 8-12 days; the onset of the rash, about 14 days. Clinically, measles is associated with fever, conjunctivitis, coryza, and cough (the 3 Cs). The rash is a discrete red maculopapular rash that first appears on the forehead, becoming coalescent as it spreads down the trunk to the feet. The rash fades in the same head-to-feet pattern as it appeared. Measles is also associated with Koplik's spots. These are 1 to 3 mm bluish-white spots on a bright red surface that appear first on the buccal mucosa opposite the lower molars. They are a pathognomonic exanthema of measles and appear approximately within 48 hours after the onset of symptoms. Roseola (C) is typically seen in children between six months and two years of age and presents with a history of high fever of three days' duration and mild symptoms. The fever abates abruptly, followed by the appearance of a macular rash on the trunk and thighs. It is caused by Human herpesvirus 6 (HHV6). Rubella (D) is associated with a maculopapular rash that first appears on the face and spreads downward to involve the trunk and extremities (similar to measles). The rash on the face fades on Day 2, and the rash on the trunk becomes coalescent. By Day 3, the rash disappears, which is why rubella is also called 3-day measles.

Which of the following is recommended by the American Academy of Pediatrics Task Force on sudden unexpected death of infancy to reduce the risk of sudden unexpected death of infancy in the general population? Parents should consider offering a pacifier at nap time and bedtime Parents should place infants in the prone position for sleep Parents should place infants to sleep on soft mattresses Parents should use home monitoring systems

Correct Answer ( A ) Explanation: Sudden unexpected death of infancy (SUDI) is defined as "the sudden death of an infant less than one year of age, which remains unexplained after a thorough case investigation, including performance of a complete autopsy, examination of the death scene, and review of clinical history." It is the third leading cause of death in infants and may occur at any time during the first two years of life, but it is rare in children younger than one month of age and in those older than one year of age. Most cases occur between two and four months of age. The American Academy of Pediatrics Task Force on sudden unexpected death of infancy recently published guidelines to help reduce the risk of SUDI in the general population.

A six-year-old boy was well until this morning when he awoke with papulovesicular eruptions over his arms, hands, legs, and feet. He refuses to eat or drink. What is the most likely etiology of his symptoms? Coxsackie virus Herpes virus Rickettsia rickettsiae Treponema pallidum

Correct Answer ( A ) Explanation: The above child has classic symptoms and signs of hand-foot-and-mouth disease, a common childhood infection caused by a Coxsackie or enterovirus. The clinical course begins abruptly with a maculopapular or maculopapular exanthem that includes the palms and soles, as well as the extremities. Involvement of the trunk may also be present. The oral enanthem typically involves the mucous membranes of the tongue and buccal mucosa and less commonly the palate or tonsils. The intraoral macules become vesicular and subsequently rupture to form ulcers with grey-yellow bases and circumferential erythema. Intraoral lesions are exquisitely painful, especially upon contact with salty or sour substances. As such, some children will refuse to drink and require admission for intravenous hydration. Preceding symptoms such as malaise or fever are typically absent. Herpes (B) may cause herpetic gingivostomatitis, the manifestation of primary infection with HSV-1. It is most common in children between six months and five years of age. Infection begins with prodromal symptoms such as fever and malaise. Subsequently, the child develops inflammation, bleeding, and friability of the gingiva, along with clusters of intraoral vesicles that progress to painful ulcerations. In contrast to hand-foot-and-mouth disease, the oral enanthem is often preceded by several days of non-specific symptoms. Children may spread herpes to other areas of their body through auto-innoculation with saliva, but the auto-innoculation would not be expected to affect both palms and both soles.

A 30-month-old male who was recently adopted from China presents to the Emergency Center after the rapid onset of difficulty breathing. His temperatures is 103°F. He preferentially sits leaning on his arms, which are outstretched in front of him, and he is drooling. What is the most likely etiology of his respiratory distress? Haemophilus influenzae type B Parainfluenza virus Respiratory syncytial virus Staphylococcus aureus

Correct Answer ( A ) Explanation: The child presents with symptoms of epiglottitis, a life-threatening upper airway obstruction that requires immediate recognition and careful airway management. Symptoms present abruptly and include respiratory distress, drooling, dysphagia, and a high fever. Affected children are toxic-appearing and often assume the "tripod" position to maximize upper airway patency. Although epiglottitis is rare in the era of routine vaccination, Haemophilus influenzae type B remains the most common cause amongst unvaccinated children. The above child should raise suspicion for incomplete vaccination because of his recent adoption from abroad. The first step in management of epiglottitis is to secure an airway. Every effort should be made to keep the child calm, and he should be taken to the operating room or intensive care unit for emergent intubation for airway protection. Empirical antibiotic therapy should include coverage of H.influenzae, as well as S.aureus, S.pyogenes, and S.pneumoniae.

A 3-month-old presents with three days of cough, congestion, and increasing work of breathing. On exam, respiratory rate is 60 breaths per minute with moderate subcostal retractions. Air movement is good, but there are scattered rhonchi, crackles, and expiratory wheezes. Heart rate is 145 beats per minute with a regular rate and rhythm and no murmurs. Oxygen saturation is 89% on room air. Which of the following is the next step in management? Initiate supplemental oxygen by nasal cannula Obtain a chest radiograph Trial of nebulized albuterol Trial of nebulized epinephrine

Correct Answer ( A ) Explanation: The above patient has bronchiolitis. His history is classic with initial upper respiratory symptoms of congestion and cough progressing to lower respiratory symptoms such as chest congestion and increased work of breathing. His examination reveals scattered crackles and wheezing, which are also typical findings of bronchiolitis. Bronchiolitis is a virally-induced inflammation of bronchioles that typically occurs in children < 2 years of age. Respiratory syncytial virus (RSV) is the most common cause of bronchiolitis, although others, including rhinovirus, adenovirus, human metapneumovirus, parainfluenza, influenza, and many others can cause bronchiolitis. Bronchiolitics display a wide range of disease severity, and those without hypoxemia, respiratory distress, or dehydration can be managed in the outpatient setting. However, the above patient has an oxygen saturation is below 90%. As per the 2015 American Academy of Pediatrics (AAP) Bronchiolitis Clinical Practice guidelines, qualifies him for initiation of supplemental oxygen by nasal cannula. Additionally, he has increased work of breathing as indicated by subcostal retractions. As such, he may benefit from high flow nasal canula to increase positive end expiratory pressure.

A five-year-old boy is brought by his parents to the clinic for a limp. He has been having fever and bone pain for two weeks. For the past few days, the parents noted that he was having difficulty walking. Physical exam reveals multiple inguinal nodes that are nontender, firm, rubbery, and 20 mm in diameter. There is no evidence of point tenderness on palpation. Laboratory testing shows anemia, thrombocytopenia, and neutropenia. There are lymphoblasts on peripheral smear. Which of the following is the most likely diagnosis? Acute lymphocytic leukemia Epstein-Barr virus infection Immune thrombocytopenia Osteomyelitis

Correct Answer ( A ) Explanation: The boy presents with limping, bone pain, fever, lymphadenopathy and lymphoblasts on peripheral smear. These findings are suspicious for acute lymphocytic leukemia (ALL), also known as acute lymphoblastic leukemia. Acute leukemia comprises approximately 30 percent of all childhood malignancies. Each year in the United States, approximately 2500 to 3500 new cases of ALL are diagnosed in children. The peak incidence occurs between two and five years of age. Also, it occurs more commonly among boys than girls. The most common presenting symptoms of ALL are nonspecific, which may include fever, bleeding, bone pain, lymphadenopathy. Unexplained persistence of any of these common signs or symptoms should prompt consideration of malignancy as a possible cause. Most children with ALL have anemia, thrombocytopenia, and lymphoblasts on peripheral smear. Approximately 50 percent of children have WBC counts < 10,000/microL, and 20 percent have an initial leukocyte count > 50,000/microL. Approximately one-half of children with ALL present with bleeding, and three-quarters have a platelet count < 100,000/microL at the time of diagnosis.

You examine a four-year-old boy for a limp. The parents note that the boy was gradually refusing to run, and later, there was the presence of a limp with walking that is especially prominent at the end of the day. Parents deny fever or rash. On exam, the boy appears well with an antalgic gait and limited hip rotation. Plain radiographs obtained were normal. Bone scan shows decreased perfusion at the left femoral head. Which of the following is the most likely diagnosis? Legg-Calvé-Perthes disease Osteomyelitis Slipped epiphysis Transient synovitis

Correct Answer ( A ) Explanation: The boy's complaints and ancillary studies are consistent with Legg-Calvé-Perthes (LCP) disease. There is osteonecrosis and femoral head deformity due to the temporary interruption of the blood supply to the proximal femoral epiphysis. The peak incidence is from ages four to eight years old. It commonly presents as a limp and activity-related pain that may be localized to the groin or referred to knee or thigh. The limp is described as an antalgic gait where there is shortening of gait phase on the injured side to alleviate weight-bearing pain. On examination, the hip motion is limited, especially internal rotation and abduction. Radiographs (antero-posterior and frog-leg views) are primarily used to diagnose, stage, and prognosticate LCP. Early changes include decreased size of the ossification center, lateralization of the femoral head with widening of the medial joint space, subchondral fracture, and physeal irregularity. In the fragmentation stage, the epiphysis looks fragmented. In the reossification stage, there is new bone formation. In the residual stage, there is gradual remodeling. If the changes are not evident on radiographs, radionuclide bone scanning with technetium-99m is helpful because it can reveal the avascularity of the femoral epiphysis.

A 3-year-old girl is brought in after her mother noticed a rash and bruising over her trunk and extremities. She also has intermittent epistaxis over the past few days. She had an upper respiratory illness 2 weeks ago but otherwise is well. Examination reveals a well-appearing child with scattered petechiae. Hemoglobin is 12 g/dL, WBC 8,000, INR 1.0, and platelets 8,000. Which of the following is the most appropriate initial treatment? Corticosteroids and intravenous immunoglobulin Observation Platelet transfusion Splenectomy

Correct Answer ( A ) Explanation: The child has idiopathic thrombocytopenic purpura (ITP), an acquired autoimmune disorder that results in platelet destruction. It often follows a viral illness and is more common in children than adults. It is characterized by thrombocytopenia in the absence of other bone marrow pathology. Signs and symptoms include petechiae, purpura, and gingival bleeding. Management depends on degree of thrombocytopenia and presence of bleeding, and should be performed in consultation with a hematologist. In general, patients with platelet count of 10,000-20,000 µL and mucosal bleeding or those with platelet counts < 10,000 µL and no bleeding are treated with corticosteroids or intravenous immunoglobulin (IVIG) or both. Asymptomatic patients with platelets > 20,000µL can be observed, as the condition is often self-limited.

A 12-year-old boy is brought by his father to the Emergency Department with a one day history of abdominal pain associated with malaise and loss of appetite. HIs abdominal pain started in the periumbilical area and later localized in the right lower quadrant. Physical examination reveals an uncomfortable febrile child with clear breath sounds, tachycardia, direct and rebound tenderness at the right lower quadrant, capillary return time of less than 2 seconds, and full pulses. Which of the following symptoms is most consistent with this boy's likely diagnosis? Anorexia Body malaise Diarrhea Vomiting

Correct Answer ( A ) Explanation: The child presents with the common manifestations of acute appendicitis. Appendicitis starts with the obstruction of the appendiceal lumen by fecal material, lymphoid hyperplasia, parasites, or tumors. Obstruction increases the intraluminal pressure within the appendix leading to lymphatic and venous congestion and edema which then impairs arterial perfusion. When there is no blood perfusing, ischemia ensues, and further bacterial invasion of the appendiceal wall results in necrosis. This process correlates with the clinical manifestations and progression from simple appendicitis to gangrenous appendicitis, and ultimately to appendiceal perforation. The classic presentation of acute appendicitis involves temporal progression of symptoms from vague mild abdominal pain, body malaise, and anorexia to severe localized abdominal pain, fever, and vomiting which typically occur within 24 to 48 hours. Abdominal pain is the first symptom characterized as initially vague and colicky located in the periumbilical area which later localizes in the right lower quadrant. Anorexia is a classic and consistent finding in acute appendicitis occurring in up to three-quarters of patients. Absence of anorexia makes the diagnosis of acute appendicitis less likely.

A 5-month-old girl is transferred from an outside facility with concerns of bilateral ptosis, ophthalmoplegia, weak suck, mild diffuse hypotonia, and constipation that have developed and worsened over the past three days. A detailed history reveals that the infant was born at term via an uncomplicated home delivery. She has been previously healthy, growing, and normally developing. Her vaccine schedule has been delayed due to parental hesitancy. She has had no recent illness, sick contacts, or trauma. She has one healthy older brother who is homeschooled and she does not attend daycare. Her parents are gradually introducing her to new fruits and vegetables from their supply of home-canned goods, and she has thus far tolerated them well. Initial laboratory evaluation included unremarkable blood counts and liver panel. Serum chemistries were significant for a slightly elevated bicarbonate level. A capillary blood gas revealed a respiratory acidosis with partial metabolic compensation. A non-contrast CT of the brain was normal and subsequent cerebrospinal fluid analysis was normal. Which of the following is indicated for treatment? Botulism immune globulin Intravenous immunoglobulin Vancomycin and ceftriaxone Vitamin K administration

Correct Answer ( A ) Explanation: The child's cranial nerve abnormalities, diffuse and progressive hypotonia, and new-onset constipation are concerning for infantile botulism. The parental report of feeding the child home-canned fruits and vegetables raises additional concern, as home-canned goods may contain Clostridium botulinum spores. In infants, C.botulinum spores may germinate into active bacteria, which colonize the gastrointestinal tract and produce neurotoxin. The neurotoxin inhibits presynaptic release of neurotransmitters in both skeletal and smooth muscle, resulting in muscle weakness and hypotonia. Of utmost concern is the potential for respiratory failure secondary to diaphragmatic weakness. This infant's respiratory acidosis raises concern for impending respiratory failure and must be monitored closely. In addition, the child should be treated with botulism immune globulin. Of note, confirmatory testing for botulism may be delayed, as testing the stool for botulinum toxin may be limited by the infant's constipation. Findings on EMG may support a diagnosis but are not pathognomonic. As such, infants with suspected botulism often require empirical treatment based on clinical suspicion.

A 14-year old girl is accompanied by her parents for amenorrhea. She never had menses. She denies sexual activity and does not take any medications. She is a straight A student in school and plays basketball. On physical examination, her height is at the 90th percentile, weight at the 50th percentile, breasts at Tanner II, no axillary hair and no pubic hair. The rest of her examination is normal. Which of the following is the most likely diagnosis? Complete androgen insensitivity syndrome Klinefelter syndrome Polycystic ovarian syndrome Turner syndrome

Correct Answer ( A ) Explanation: The girl in the vignette most likely has complete androgen insensitivity syndrome (AIS) (formerly known as testicular femininization). Complete AIS is an extreme form of failure of virilization wherein genetic males appear female at birth and are invariably reared accordingly. The external genitalia are female. The vagina ends blindly in a pouch, and the uterus is absent. This is due to the normal production and effect of antimullerian hormone by the testes. The testes are usually intra-abdominal although some may descend into the inguinal canal. At puberty, the diagnosis is suspected by the normal development of breasts, but menstruation does not occur and sexual hair is absent. Adult heights are commensurate with those of normal males. Management of patients with complete AIS involves removal of testes whose sexual orientation is unambiguously female. This can be done through laparoscopic removal of Y chromosome-bearing gonads. Klinefelter syndrome (B) has XXY chromosomes and presents with gynecomastia, small testicles, infertility, mild delays, and a low upper-to-lower segment ratio. Polycystic ovarian syndrome (C) presents with irregular menses, acne, excess body hair, and signs of insulin resistance. Turner syndrome (D) has XO chromosomes and presents with breast buds, short stature, pedal edema, wide-spaced nipples. They may have primary amenorrhea from streak ovaries and ovarian failure.

A 4-month-old infant is brought to clinic with concerns of rash. Over the past week, his mother noted a red rash over both of his cheeks. The rash has neither spread nor remitted. He has otherwise been healthy. On examination, he exhibits erythematous, scaling, excoriated plaques over both cheeks. What is the most likely diagnosis? Atopic dermatitis Impetigo Irritant contact dermatitis Seborrheic dermatitis

Correct Answer ( A ) Explanation: The infant has an eczematous rash over both cheeks. Both the description of his rash and its location are consistent with atopic dermatitis. Atopic dermatitis is an inflammatory skin condition that affects children and adults of all ages. It is a result of an overactive response to a variety of stimuli, including food allergens, inhaled allergens, irritant cosmetics, weather and humidity changes, pollution, and stress. Atopic dermatitis commonly causes diffuse xerosis, as well as eczematous plaques, papules, and crusting. In infants, plaques are most commonly distributed over the cheeks, as well as the extensor surfaces of the extremities. In older children, the face is less often involved, and the flexor surfaces of the extremities, such as the antecubital fossa, are frequently affected. In addition to causing pruritis and irritation, the compromised skin integrity in atopic dermatitis may result in bacterial and viral superinfections. Children with eczema are also predisposed to other atopic diseases such as asthma and allergic rhinitis, as well as to food allergies. Seborrheic dermatitis (D), also known as Cradle Cap, is a common pediatric skin condition that results in superficial scaling. The scales are classically greasy and yellow. Seborrheic dermatitis is distributed in areas where seborrheic glands are most concentrated such as the scalp, central face ("T zone"), preauricular skin, and intertrigenous area.

A four-week-old baby boy presents to the Emergency Department with projectile vomiting over the past three days. On physical exam, he is found to have a palpable olive-like mass in the right upper quadrant. On laboratory analysis he has a hypochloremic metabolic alkalosis. Which of the following is a risk factor for development of this condition? Erythromycin use Female sex Full-term birth Second born child

Correct Answer ( A ) Explanation: The infant has pyloric stenosis, and erythromycin use is a risk factor. Pyloric stenosis occurs when there is hypertrophy of the pylorus, which causes a gastric outlet obstruction. It is most commonly seen between three to five weeks after birth. Risk factors include first born, male sex, prematurity, and macrolide antibiotic use. Infants will present as a "hungry vomiter" with forceful nonbilious vomiting with a desire to feed again. On physical exam, a palpable olive may be found in the right upper quadrant. Repetitive vomiting will lead to a hypochloremic metabolic alkalosis from loss of stomach acid. If this continues for a prolonged period, hypokalemia will occur due to stimulation of aldosterone from hypovolemia. Abdominal ultrasound will show a "target sign" that is characterized by an increased pyloric muscle thickness, length, and diameter. Definitive treatment is surgical management with a pyloromyotomy.

A 3-day-old neonate is brought to clinic with copious eye discharge and eyelid swelling bilaterally. Her mother received minimal prenatal care and delivered the neonate at home. Which of the following is indicated? Ceftriaxone intravenous Gonococcal PCR of eye discharge Oral erythromycin Reassurance with outpatient ophthalmologic follow-up

Correct Answer ( A ) Explanation: The neonate has findings concerning for ophthalmia neonatorum, neonatal conjunctivitis caused by Neisseria gonorrhoeae. N.gonorrhoeae is typically acquired during vaginal delivery and results in ophthalmia neonatorum in 30-40% of neonates born to mothers with gonorrheal cervicitis. Although all mothers should be screened during pregnancy, the infection may be acquired between the time of testing and delivery. Additionally, mothers with inadequate prenatal care may not be tested. Symptoms of ophthalmia neonatorum begin with conjunctivitis and discharge typically begins in the first two to five days after birth. The mucopurulent discharge is typically copious, and the eyelids may be so swollen that the conjunctiva cannot be visualized. When the diagnosis is suspected, a conjunctival swab should be sent for culture. Evaluation for extension of infection merits blood and CSF cultures, as well as evaluation for co-infection with Chlamydia trachomatis, syphilis, and HIV. When ophthalmia neonatorum is suspected, ceftriaxone should be empirically administered intramuscularly or intravenously pending culture and pathogen sensitivities. Isolated eye involvement is treated with a single dose of ceftriaxone, whereas disseminated disease is treated with seven days of ceftriaxone or cefotaxime. discharge typically begins in the first two to five days after birth

A 1-week-old boy born at full term to a 16-year-old primigravida mother with no complications presents to the ED for lethargy. In the ED, his vital signs are T 37°C, HR 166, RR 82, and pulse oximetry of 80%. On exam, he is lethargic with diffuse pulmonary rales and rhonchi, cold extremity, and decreased peripheral pulses. Given concern for ductal-dependent cardiac lesion, you administer prostaglandin E1 (PGE1) infusion. Which of the following is a known adverse reaction of a PGE1 infusion? Apnea Hypertension Hypothermia Thrombocytosis

Correct Answer ( A ) Explanation: The patient in this vignette has a ductal-dependent cardiac lesion that requires a patent ductus arteriosus (PDA) to preserve blood flow from the aorta to the pulmonary artery. At approximately 1 week of age, the PDA may close and cause sudden cardiovascular collapse. PGE1 infusion is potentially life-saving, but it is important to be aware of its adverse reactions. For example, apnea is a known adverse reaction of PGE1 infusion; therefore, it is important to secure the airway with endotracheal intubation prior to its infusion. Other reactions include fever, not hypothermia (C); hypotension, not hypertension (B); decreased platelet aggregation, not thrombocytosis (D), seizures; bradycardia; and flushing. It is important to anticipate these effects and have additional IV fluid boluses, fresh frozen plasma, and seizure medications prepared. Most patients should be prophylactically intubated.

An 18-year-old man presents after a syncopal episode. The patient states he was running when he suddenly felt lightheaded and nauseated leading to him passing out. On presentation, he is asymptomatic with normal vital signs. Examination reveals a healthy man with a crescendo-decrescendo murmur heard at the left sternal border. What diagnosis should be suspected in this patient? Hypertrophic cardiomyopathy Hypovolemic state Vasovagal syncope Wolff-Parkinson-White syndrome

Correct Answer ( A ) Explanation: This 18-year-old man has exertional syncope raising the suspicion of hypertrophic cardiomyopathy (HCM). HCM is an autosomal dominant genetic abnormality in which the left ventricle (particularly the septum more than the free wall) is hypertrophied. The typical presentation is dyspnea on exertion in a young patient with no other cardiac disease. About 20-30% will present after a syncopal event and many present with sudden death. Physical examination may reveal a loud S4 gallop and a harsh mid-systolic crescendo-decrescendo murmur. The murmur can be increased by maneuvers that decrease left ventricular filling (standing or performing a Valsalva maneuver) and decreased by maneuvers that increase left ventricular filling (squatting, Trendelenberg). ECG is abnormal in 90% of patients and the classic ECG finding is left ventricular hypertrophy with deep, narrow Q waves particularly in the precordial leads. Diagnosis can be made by echocardiography but cardiac catheterization is the gold standard.

A 15-month-old old previously healthy boy presents to the office with 2-day history of right ear tugging and fever to 102°F. On otoscopy, his right tympanic membrane is erythematous and bulging. What is the first line treatment? Amoxicillin Amoxicillin-Clavulanate Oral decongestants Withhold antibiotics for 3 days to see if symptoms persist

Correct Answer ( A ) Explanation: This child has acute otitis media. The most common organisms that cause acute otitis media are non-typeable Haemophilus influenzae, Streptococcus pneumoniae, and Moraxella catarrhalis. The incidence of non-typeable Haemophilus influenzae is increasing while Streptococcus pneumoniae is decreasing. The first-line treatment in infants and toddlers (< 2-years-old) is amoxicillin because it covers both penicillin-susceptible and penicillin-nonsusceptible strains of S. pneumoniae as well as non-typeable H. influenzae that do not contain beta-lactamases. In patients with uncomplicated acute otitis media, treatment is warranted in individuals < 24 months of age, those with rapid onset of symptoms, and severe pain. Withholding antibiotic treatment for up to 3 days (D) to see if symptoms persist is accepted in uncomplicated patients over 24 months old. In these cases, you can provide a prescription for antibiotics and advise parents to wait up to 3 days. If symptoms persist, the parents can initiate treatment. The majority of cases of acute otitis media will resolve in two weeks without treatment.

A 15-year-old boy presents to the emergency department with lower back pain. He reports heavy lifting three weeks ago, with gradual onset of pain and no relief with symptomatic care. The pain is localized to the paraspinal muscles over his lumbar back. He reports normal urine output. Baseline laboratory tests are notable for a sodium of 140, potassium of 5.2, chloride of 110, bicarbonate of 25, BUN of 20, and creatinine of 2.3. A urinalysis shows hyaline casts, 5 white blood cells, and 1+ protein. Which of the following is the most likely pathophysiology of his laboratory findings? Acute interstitial nephritis Acute tubular necrosis Hypovolemia Obstructive nephropathy

Correct Answer ( A ) Explanation: This patient has acute renal failure (ARF), exhibited by his elevated creatinine level. The common causes of ARF can be subdivided into three categories: prerenal, intrinsic renal, and postrenal. Intrinsic renal pathology can be further subdivided into glomerular disease, interstitial disease, and tubular disease. This patient has intrinsic renal disease, as his BUN to creatinine ratio is less than 15 and his urine output is normal. The most likely etiology of his intrinsic renal disease is acute interstitial nephritis (AIN) secondary to overuse of non-steroidal anti-inflammatory drugs for back pain. Many drugs and toxins can cause AIN. It typically presents one to two weeks following onset of the inciting agent, and may be associated with symptoms such as fever, rash, arthralgias, and eosinophilia. Urine output is preserved. Urinalysis may show white blood cells and hyaline or granular casts, as well as mild proteinuria. Removal of the offending agent often leads to recovery. Acute tubular necrosis (B) is another type of intrinsic renal damage. It is typically seen in critically ill children with sepsis, hypotension, or exposure to nephrotoxic medications. ATN is characterized by elevated creatinine and a urinalysis showing brown granular casts.

9-year-old boy is brought in by his mom for bizarre behavior. She states that he has had intermittent fevers for the past week and is now complaining of joint pain and swelling. Most concerning for Mom is the writhing, purposeless, and uncontrollable movements of her son's hands that she observed this morning. On exam, a diastolic murmur is noted at the right upper sternal border. Which of the following tests is most likely to confirm the diagnosis? Antistreptolysin O titer (ASO) CBC ECG Echocardiogram Myocardial biopsy

Correct Answer ( A ) Explanation: This patient has acute rheumatic fever (ARF), a systemic disease triggered by a complex hyperimmune response to group A streptococcal (GAS) pharyngitis. The American Heart Association has developed and modified the Jones criteria to enable physicians to identify patients with ARF. To fulfill the criteria, the patient must exhibit evidence of recent streptococcal disease and have either two major criteria or one major and two minor criteria. This patient has three of the major criteria: chorea (writhing, purposeless, and uncontrollable movements of the hands); carditis (diastolic murmur suggestive of aortic insufficiency); and arthritis. Evidence of recent streptococcal infection is required to confirm the diagnosis. This can be accomplished by detecting an elevated or rising ASO titer or by obtaining a positive throat culture or rapid Strep antigen test.

3-year-old girl with a past medical history of constipation is brought to the ED for evaluation of a limp and left knee pain. Her physical exam is unremarkable. You obtain the knee radiograph seen above. What do you expect to see on her peripheral blood smear? Basophilic stippling Heinz bodies Hypersegmented neutrophils Schistocytes

Correct Answer ( A ) Explanation: This patient has lead lines on her knee radiograph, a classic finding of lead poisoning. These hyperdense lines occur at the metaphyses and are most commonly seen in young children on knee and wrist radiographs. Similar lines may also be seen in other heavy metal poisoning such as bismuth and arsenic, and in healing rickets or scurvy. Lead poisoning causes a microcytic, hypochromic anemia. On peripheral blood smear, the pathognomonic finding is basophilic stippling. Basophilic stippling represents clumps of degraded RNA that have not been eliminated due to lead-induced nucleotidase inhibition. This child's joint pain and constipation are likely related to lead poisoning. Heinz bodies (B) are inclusions within red blood cells made of denatured hemoglobin. They are seen in glucose-6-phosphate deficiency, liver disease, and alpha-thalassemia. Hypersegmented neutrophils (C) are seen in multiple forms of megaloblastic anemia such as that caused by alcohol abuse, folate deficiency, and vitamin B12 deficiency. Schistocytes (D) are fragmented red blood cells caused by shearing with fibrin strands in microarterioles. They are seen on smears in patients with artificial heart valves and in numerous disease processes, including hemolytic uremic syndrome, thrombotic thrombocytopenic purpura, disseminated intravascular coagulation, and congestive heart failure.

3-year-old girl is brought to the urgent care clinic for a 4-hour history of dysphagia, fever, drooling, and difficulty breathing. She is in acute distress, restless, and irritable. Her immunizations are up to date. Her temperature is 40.0°C (104°F). Auscultation of the lungs reveals inspiratory stridor. Which of the following is the most likely diagnosis? Acute epiglottitis Anaphylaxis Croup Diphtheria

Correct Answer ( A ) Explanation: This patient most likely has acute epiglottitis. Epiglottitis is inflammation of the epiglottis and adjacent supraglottic structures and results from bacteremia or direct invasion of the epithelial layer by the pathogenic organism. Haemophilus influenzae type b (Hib) is the most common infectious cause of epiglottitis in children. Although the incidence of Hib epiglottitis declined after Hib was added to the routine infant immunization schedule in the United States and other developed countries, Hib epiglottitis still occurs, even in immunized children. Additional causes of epiglottitis in children include other H. influenzae (types A, F, and nontypeable), streptococci, and Staphylococcus aureus, including methicillin-resistant strains. Without treatment, epiglottitis can progress to life-threatening airway obstruction.Correct Answer ( A ) Explanation: This patient most likely has acute epiglottitis. Epiglottitis is inflammation of the epiglottis and adjacent supraglottic structures and results from bacteremia or direct invasion of the epithelial layer by the pathogenic organism. Haemophilus influenzae type b (Hib) is the most common infectious cause of epiglottitis in children. Although the incidence of Hib epiglottitis declined after Hib was added to the routine infant immunization schedule in the United States and other developed countries, Hib epiglottitis still occurs, even in immunized children. Additional causes of epiglottitis in children include other H. influenzae (types A, F, and nontypeable), streptococci, and Staphylococcus aureus, including methicillin-resistant strains. Without treatment, epiglottitis can progress to life-threatening airway obstruction.

An 8-year-old girl is being evaluated for a persistent cough. She also has malaise, weight loss, and fatigue. She says her cough is worse at night and interferes with her sleep. She had an unknown illness about 10-days ago that consisted of coryza, lacrimation, and a low-grade fever. However, the cough has persisted and is becoming more severe as she has several repetitive spasmodic bursts of 5-10 coughs occurring about 20 times a day. Which of the following is the most likely cause of this patient's condition? Bordetella pertussis Haemophilus influenza Influenza virus Parainfluenza virus

Correct Answer ( A ) Explanation: This patient most likely has pertussis (ie, "whooping cough"). Pertussis is an acute infection of the respiratory tract caused by Bordetella pertussis. The name pertussis means "violent cough," which describes the most consistent and prominent feature of the illness. The inspiratory sound made at the end of an episode of paroxysmal coughing gives rise to the common name for the illness, "whooping cough." Classic pertussis is most often seen in preschool and school-age children. After an incubation period averaging 7-10 days, an illness develops that is indistinguishable from the common cold and is characterized by coryza, lacrimation, mild cough, low-grade fever, and malaise. After 1-2 weeks, this catarrhal phase evolves into the paroxysmal phase: the cough becomes more frequent and spasmodic with repetitive bursts of 5-10 coughs, often within a single expiration. The frequency of paroxysmal episodes varies widely, from several per hour to 5-10 per day. Episodes are often worse at night and interfere with sleep.

A 5-year-old boy presents with fever and rash for two days. Examination reveals a well-appearing child with marked erythema to both cheeks as seen above. What management is indicated? Acetaminophen for fever Cephalexin Mupirocin Topical hydrocortisone

Correct Answer ( A ) Explanation: This patient presents with symptoms consistent with erythema infectiosum and requires supportive care. Erythema infectiousum, or fifth disease is caused by infection with parvovirus B19. The disease is characterized by rash and mild systemic symptoms. The classic rash is deeply red on the face giving a "slapped-cheek" appearance with circumoral pallor. Additionally, a maculopapular, lacelike rash may be seen on the arms and progresses caudally. Rarely, parvovirus B19 has been associated with hepatitis. There is no specific treatment for erythema infectiosum and so management should focus on supportive care and parental reassurance. Acetaminophen or ibuprofen can be used to treat the fever

A 15-year-old girl presents to urgent care with a nonproductive cough and low-grade fever that developed insidiously over the previous week. Physical exam reveals diffuse rales on pulmonary auscultation, and chest X-ray shows diffuse non-focal infiltrates. Which of the following is the most accurate statement regarding this patient's condition? Extrapulmonary manifestations may include hemolysis and CNS involvement Mycoplasma pneumoniae is the most common etiology in all age groups Penicillin is the most appropriate treatment The formation of cold agglutinins is a positive confirmatory test

Correct Answer ( A ) Explanation: This patient's clinical presentation is characteristic of community-acquired pneumonia. There are numerous pathogens that cause community-acquired pneumonia in children with a predilection for causing disease in particular age groups. Mycoplasma pneumonia, Chlamydophila pneumonia, and S. pneumoniae are most common in school age through the teen years. S. pneumoniae is classically associated with "typical" pneumonia, which is marked by rapid onset of high fever accompanied by chills, a high white count (>15,000), and findings of lobar consolidation on chest X-ray. M. pneumoniae and C. pneumoniae are classically associated with "atypical" pneumonia and present with a more gradual onset of symptoms and a pattern of interstitial infiltrates on chest radiography. Atypical bacterial infections are also more likely to present with extrapulmonary manifestations such as skin rashes, hemolysis (secondary to the formation of cold agglutinin antibodies), CNS involvement (aseptic meningitis, cerebellar ataxia, cranial nerve palsies), and gastrointestinal symptoms. However, these typical and atypical features frequently overlap and cannot be used consistently to distinguish between different etiologic agents.

A young boy suffers from involuntary tics. During a detailed speech evaluation, you detect the intermittent use of varying obscenities. Which of the following speech disorders, which is associated with Tourette's syndrome, describes this finding? Coprolalia Echolalia Glossolalia Palilalia

Correct Answer ( A ) Explanation: Tics are repetitive, sterotyped movements or vocalizations. The most common forms are blinking, sniffing, grimacing, throat-clearing and throwing the head sideways or backward. Tourette's syndrome is a combination of multiple tics, including involuntary vocalizations and coprolalia, the involuntary, compulsive use of obscenities. One-third of cases are familial, and the underlying pathology is felt to lie within the caudate nucleus. There are no strong associations with psychiatric illness except for a small tie to obsessive-compulsive personality disorder. Treatment includes benzodiazepines or clonidine. Microdosing of haloperidol may be useful in severe cases. Echolalia (B) is involuntary repetition of words or phrases spoken by others. Glossolalia (C) is speaking elaborate but meaningless speech, or speaking an unknown language. Palilalia (D) is involuntary repetition or echoing of one's own words.

A mother brings her 4-year-old previously healthy daughter to clinic for vaginal itching and erythema. The mother is the primary care provider for the child and does not suspect abuse. On exam there is no discharge, the vulva appears erythematous, and the hymen is intact, thin and pinkish-red. Which of the following is the most appropriate management? Advise the avoidance of strong detergents and bubble baths Immediately call Child Protective Services Obtain gonorrhea and chlamydia DNA probes Prescribe acyclovir

Correct Answer ( A ) Explanation: Vulvovaginitis presents with vaginal irritation, pain, and pruritus. Irritant vulvovaginitis is the most common cause of genital complaints in prepubertal girls. Tight clothing is also a cause. Vulvitis refers to the external genital pruritus, burning, redness, or rash. Vaginitis implies inflammation of the vagina, which manifests as discharge with or without an odor or bleeding. Vaginitis can lead to vulvitis. Other cases of non-sexually transmitted vulvovaginitis include pinworms (Enterobius vermicularis), group A beta hemolytic Strep, Staph. Spp. and candida.

A 12-year-old boy with Wolff-Parkinson-White syndrome presents with palpitations for the past hour. His blood pressure is 110/62 mm Hg and pulse is 166 bpm. The ECG reveals a narrow-complex tachycardia. Vagal maneuvers are ineffective. Which of the following is the next most appropriate management? Adenosine Cardioversion Procainamide Radiofrequency ablation

Correct Answer ( A ) Explanation: Wolff-Parkinson-White syndrome is a congenital cardiac defect in which an abnormal conduction pathway exists between the atria and the ventricles. It is commonly accompanied by congenital cardiomyopathy. Infantile or childhood onset of paroxysmal tachycardia is the typical manifestation. These appear as a narrow complex tachycardia and are referred to as orthodromic. In orthodromic tachycardia, the normal pathway is used for ventricular depolarization, and the accessory pathway (bundle of Kent) is used for the retrograde conduction essential for reentry. Management begins with vagal maneuvers. If unsuccessful and the patient is hemodynamically stable, AV nodal blocking agents, such as adenosine or calcium channel blockers, can be administered.

An obese, 15-year-old boy presents with a history of a limp for 2 months with some vague non-specific hip pain. Physical examination of the affected hip reveals a restriction of internal rotation, abduction, and flexion. On exam you note that when examining the affected hip the patient refuses to allow any range of motion of the hip. The extremity is shortened, abducted, and externally rotated. There is no history of trauma. Which of the following is the most likely diagnosis? Femoral anteversion Slipped capital femoral epiphysis Tibial torsion Trochanteric bursitis

Correct Answer ( B ) Explanation: A fracture in the physis (growth plate) of the femoral head can lead to slippage of the overlying epiphysis, leading to the condition slipped capital femoral epiphysis (SCFE). The classic patient presenting with SCFE is an obese boy between the ages of 11 and 16 years. Girls present earlier, usually between 10 and 14 years of age. Patients with chronic and stable SCFEs tend to present after weeks to months of symptoms. Patients usually limp to some degree and have an externally rotated lower extremity. Physical examination of the affected hip reveals a restriction of internal rotation, abduction, and flexion. Commonly, the examiner notes that as the affected hip is flexed, the thigh tends to rotate into progressively more external rotation. Chronic SCFE is the most common form of presentation. Typically, an adolescent presents with a few-month history of vague groin, thigh, or knee pain and a limp. Radiographs show a variable amount of posterior migration of the femoral epiphysis and remodeling of the femoral neck in the same direction; the upper end of the femur develops a bending of the neck. Femoral anteversion (A) is when the top of the femur is rotated causing the foot to turn in. Intoeing due to excessive femoral anteversion is generally most noticeable when a child is between ages three and seven. Tibial torsion(C) is when the tibia is slightly twisted or rotated causing the foot to turn in. This may be due to the position your child was sitting in while in the uterus. Intoeing due to internal tibial torsion is generally most noticeable when a child begins walking. Trochanteric bursitis (D) is caused by inflammation or irritation of the bursae around the greater trochanter and other anatomic structures that are close to the greater trochanter, particularly gluteal tendons, which are subject to tendonitis. Key findings on history and physical examination include lateral hip pain (e.g. not being able to sleep on the affected side); tenderness over the area around the greater trochanter; no significant range of motion abnormalities in the hip joint; pain only at the extremes of hip movement; and a positive Patrick flexion, abduction, external rotation and extension (FABERE) test.

An 8-month-old boy has a painless swelling on the right side of his scrotum since birth that gets worse with crying. Transillumination of this side of the scrotum reveals increased fluid and the child does not cry on palpation of the testicle which lies in its normal position. Based on these findings, which of the following is the most likely diagnosis? Epididymitis Hydrocele Testicular torsion Varicocele

Correct Answer ( B ) Explanation: A hydrocele is a collection of fluid that accumulates in the tunica vaginalis. Communicating hydroceles result when the upper processus vaginalis fails to be obliterated, leaving an open tract between the peritoneum and the scrotum. The tract is closed in non-communicating hydroceles. Most hydroceles are right sided. Hydroceles may be present at birth, but they usually are painless and worsen with crying or exertion. Hydroceles often resolve spontaneously by the age of 18 months. Examination reveals transillumination and enlargement of the scrotum. Patients with a hydrocele that has persisted for more than 1 year or who are older than 18 months should undergo ultrasonography to ensure that the hydrocele is not a reactive hydrocele caused by testicular tumor or inflammatory process

Which of the following is the most common manifestation of abusive head trauma in infants? Epidural hematoma Retinal hemorrhage Subarachnoid hemorrhage Subdural hematoma

Correct Answer ( B ) Explanation: Abusive head trauma (previously referred to as shaken baby syndrome) is a form of inflicted head trauma and the leading cause of child abuse fatalities. It is a well-recognized clinical syndrome caused by violent shaking of infants, direct blows to the head, dropping or throwing a child, and asphyxia. It typical occurs in infants younger than 1 year, but may be seen in children up to 3 years old. The classic constellation of abusive head trauma includes subdural hematoma, traumatic brain injury, and retinal hemorrhages. Retinal hemorrhages are present in up to 75% of cases and are virtually pathognomonic. Absence of retinal hemorrhage does not rule out child abuse. The pathophysiology of retinal hemorrhages is uncertain. It is unclear whether bleeding is a result of increased intracranial pressure transmitted to the eye or occurs directly within the eye itself, perhaps through increased pressure along the retinal vein with subsequent disruption of the vessel. Retinal hemorrhages may involve the area in front of the retina (preretinal hemorrhages), the vitreous, and the subretinal space in addition to the retina. Hemorrhages may be described as "dot and blot" hemorrhages or flame or splinter hemorrhages.

A 2-year-old girl presents to the Emergency Department with her parents for a rash. She has lesions on her palms and on the soles of her feet, as well as in her oral cavity as shown above. Which of the following indicates that the child is no longer contagious and can safely return to daycare? Ability to tolerate oral intake Active skin lesion resolution Antibiotic therapy is complete Fever resolution

Correct Answer ( B ) Explanation: An oral exanthem plus a macular, maculopapular, or vesicular rash on the hands and feet suggests a diagnosis of hand, foot, and mouth disease (HFMD) caused by an enterovirus, most commonly Coxsackievirus A serotypes. This is a clinical diagnosis. Outbreaks are seen in daycare centers, schools, summer camps, hospital wards, and military installations. The most common age group affected are children less than five years of age. The virus is usually transmitted from person to person by the fecal-oral route. However, transmission can occur by contact with oral, respiratory, and vesicular secretions. The incubation period may last from 2-7 days. Patients may present with throat or mouth pain or refusal to eat. Fever is rare and is usually below 38.3ºC. Management is largely supportive with hydration and pain control. Infants and children with active skin lesions should be excluded from daycare. Once the active skin lesions resolve the child can return to daycare. It is also important to practice strict hand hygiene as enteroviruses are spread through stool for weeks following infection.

A 17-month-old girl presents to the office with two days of fever, congestion, and cough. On examination, inspiratory stridor is noted when the patient becomes fussy. What is the most likely etiology of her stridor? Bacterial tracheitis Croup Epiglottitis Laryngotracheal foreign body aspiration

Correct Answer ( B ) Explanation: Croup is inflammation of the larynx and trachea. The most common etiology of croup is parainfluenza virus, but a number of viruses have been implicated. Croup is common amongst children aged 6 to 36 months. It typically begins with symptoms of an upper respiratory tract infection, such as coryza, cough, and fever. Symptoms may then progress to a barking cough, hoarseness, and inspiratory stridor due to progressive upper airway obstruction. Stridor may occur solely with activity or with both activity and rest. Stridor that occurs at rest indicates significant upper airway obstruction. Mild cases of croup do not require treatment, however, those with significant upper airway obstruction will require hospital admission. Patients with bacterial tracheitis (A), epiglottitis (C), and laryngotracheal foreign bodies (D) have an acute and severe onset of upper airway obstruction.Bacterial tracheitis and epiglottitis typically also cause fever and a toxic appearance

A 4-year-old boy presents with a laceration to his distal forearm. You are considering using a topical tissue adhesive to close the wound. Which of the following is true of tissue adhesive use in lacerations? In wounds that require deep suture closure, the superficial layer should not be closed with skin adhesive Tissue adhesives have the equivalent strength of 4-0 nylon sutures Tissue adhesives should be avoided on areas of cosmetic importance Tissue adhesives should be used only for lacerations <2 cm in length

Correct Answer ( B ) Explanation: Cyanoacrylate tissue adhesives are liquid monomers that polymerize into a stable bond when they come into contact with moisture. They offer many advantages over traditional wound closure with sutures. Their use offers faster wound closure with decreased pain with comparable rates of infection. There is some evidence that indicates that adhesives not only provide their own dressing but also have antimicrobial properties and may decrease the rate of wound infection. Tissue adhesives are equivalent in strength to 4-0 nylon sutures; they should be applied only to areas with appropriate tension. Some limitations include the inability to use antimicrobial or other petroleum-based products on the wound, the recommendation not to swim to limit forces that may prematurely remove the adhesive, and the greater risk of dehiscence.

A five-year-old boy is brought into the clinic by his mother with complaints of perianal itching that is worse at night. She states that he is irritable during the day but thinks it is because he is not sleeping well. Physical exam reveals perianal excoriations without any other concerning findings. What is the most likely diagnosis? Ascariasis Enterobiasis Hookworm Trichuriasis

Correct Answer ( B ) Explanation: Enterobiasis (pinworm) is an infection with the nematode Enterobius vermicularis. The adult worms live in the colon and the female worms deposit eggs in the perianal area causing intense perianal pruritus. Egg laying occurs mostly at night causing patients to complain that the itching worsens at night. Symptoms are usually limited to perianal pruritus. Itching causes contamination of the fingers, which allows the life cycle to continue by autoinoculation or spread to other contacts. Treatment includes anti-helminth medications with re-treatment after two weeks to ensure that all newly hatched adults are eradicated as well. All members of a household should be treated at the same time and care should be taken to promote personal hygiene, hand washing, and frequent laundering of bedding and bedclothes. Ascariasis (A) is an infection with Ascaris, which are transmitted through moist soil in tropical or subtropical areas. The majority of infections are asymptomatic although there can be abdominal pain, weight loss, anorexia, diarrhea, or vomiting. Occasionally, intestinal obstruction has been reported in children with heavy infections. The diagnosis is made by sending a stool sample for analysis looking for ova and parasites. Hookworm (C) infection is caused by either Ancystoloma duodenale or Necator americanus, which are prevalent in tropical areas, especially in Asian countries. Blood loss is the major complication of infection with the adults attaching to the intestinal mucosa and feeding on blood. Infections can be asymptomatic but could be associated with diarrhea. Persistent infections can cause iron deficiency anemia, growth delay in children, and hypoproteinemia. Trichuriasis (D) or whipworm is another soil nematode that causes infection in humans living in warm, humid regions. Infections can be asymptomatic; however, heavy infections can cause abdominal pain, diarrhea, or rectal prolapse.

A four-year-old from Guatemala presents to the emergency room with a fever, drooling, and inspiratory stridor. Dad states that the child appeared well the day prior to presentation. The child is sitting with his neck extended. Which of the following is the most likely diagnosis? Croup Epiglottitis Laryngomalacia Tracheitis

Correct Answer ( B ) Explanation: Epiglottitis presents with rapid onset of respiratory distress, high fever, muffled voice, and drooling. Physical exam is significant for an anxious child with stridor. They are typically sitting in a tripod position, leaning forward with their neck extended. Epiglottitis is a clinical diagnosis; however, a lateral neck X-ray may aid in the diagnosis. The classic thumb print sign can be seen where the enlarged rounded epiglottis extends into the hypopharyngeal airway. Thickening of the aryepiglottic folds are also suggestive of epiglottitis. Patients should be observed closely for airway compromise. Some patients require advanced airway management, which should be performed in the operating room by an anesthesiologist. Parenteral antibiotics should be administered to cover Haemophilus influenzae B, Streptococcus pneumoniae, β-hemolytic Streptococci, and Staphylococcus aureus. Antibiotics can be narrowed based on culture and sensitivities. The rate of epiglottitis has decreased since the introduction of the HiB vaccine. However, this patient is from another country and may not have received the HiB vaccine.

A 3-year-old boy is referred to the Emergency Department due to concern for Kawasaki Disease. He has persistent high fevers for five days. He also has a generalized scarlatiniform rash, red and cracked lips, and three days of watery diarrhea. Today he developed bilateral conjunctival injection without exudate. In addition to the features described, generalized lymphadenopathy is also noted. Which of this patient's clinical features is inconsistent with a diagnosis of Kawasaki Disease? Diarrhea Generalized lymphadenopathy Non-exudative conjunctivitis Scarlatiniform rash

Correct Answer ( B ) Explanation: Generalized lymphadenopathy is not typical of Kawasaki Disease and should prompt consideration of alternative infectious, rheumatologic, or oncologic etiologies. The lymphadenopathy of Kawasaki Disease is typically confined to the anterior cervical region and is almost always unilateral. At least one node must measure > 1.5 cm, although a conglomeration of nodes may be noted on ultrasound. The involved nodes do not suppurate. Notably, lymphadenopathy is the least commonly fulfilled clinical criteria in Kawasaki Disease and is absent in up to 75% of cases.

In which of the following conditions is hypoxemia caused by a right-to-left shunt? Asthma Eisenmenger syndrome Patent foramen ovale Pulmonary embolism

Correct Answer ( B ) Explanation: Hypoxemia is abnormally low oxygen tension, defined as a PaO2 of < 60 mm Hg, and is caused by numerous mechanisms, including hypoventilation, shunting, ventilation-perfusion mismatch, impaired diffusion, and low inspired oxygen. Right-to-left shunt occurs when deoxygenated blood from the right side of the heart enters the systemic circulation. This can occur because of an anatomic shunt (i.e., congenital cyanotic heart disease), in which deoxygenated blood bypasses the lungs and directly enters the systemic circulation or a physiologic shunt, which occurs when blood traverses nonventilated segments of the lung. Physiologic shunt is seen in conditions of alveolar collapse, such as in acute respiratory distress syndrome (ARDS) or alveolar filling, such as in pulmonary consolidation. The hallmark of right-to-left shunt is that supplemental oxygen fails to increase arterial oxygen levels. This is in contrast to other causes of hypoxemia that respond to supplemental oxygen. Causes of right-to-left shunt include congenital cyanotic heart disease (e.g., truncus arteriosus, transposition of the great vessels, tricuspid atresia, tetralogy of Fallot, and total anomalous pulmonary return), ARDS, pulmonary consolidation, or atelectasis. In addition, an uncorrected left-to-right shunt, such as a ventricular septal defect, atrial septal defect, or patent ductus arteriosus, can eventually become a right-to-left shunt, a phenomenon known as Eisenmenger's syndrome. This occurs when increased pulmonary blood flow from a left-to-right shunt leads to pulmonary hypertension and compensatory right ventricular hypertrophy, and, over time, right ventricular pressures surpass left ventricular pressures, resulting in a change in direction of the shunt.

A mother brings her 11-month-old infant into a rural ED for inconsolable crying. You note the infant lying in the stretcher crying with his knees drawn to his chest. You perform a physical exam and leave the room to order blood work. When you return to the examination room, you note the infant now appears lethargic. An abdominal radiograph is obtained and interpreted by the radiologist as nonspecific. Which of the following diagnostic tests should be performed next? Abdominal CT scan Abdominal ultrasound Nasogastric tube lavage Upper GI series

Correct Answer ( B ) Explanation: In patients with suspected intussusception, ultrasound is a quick, noninvasive diagnostic modality commonly used for the diagnosis of intussusception. One study reported that the overall sensitivity and specificity of ultrasonography for detecting ileocolic intussusception was 97.9% and 97.8%, respectively. The authors concluded that ultrasonography should be used as a first-line examination for the assessment of possible pediatric intussusception. Ultrasonography eliminates the risk of exposure to ionizing radiation and can help to depict lead points and residual intussusceptions. It also helps to rule out other possible causes of abdominal pain. On the transverse ultrasound scan, the intussusception appears as a multilayered or wrapped complex mass; longitudinally, it appears as a tube within a tube.

A 4-month-old baby presents to your office with symptoms of worsening constipation, poor feeding, listlessness, and generalized weakness for two weeks. Physical exam findings include temperature of 98.6°F, ptosis, poor head control and poor ability to suck. Which of the following is the most appropriate initial therapy? Bisacodyl Human-derived botulism immune globulin Magnesium citrate Senna

Correct Answer ( B ) Explanation: Infant botulism, while rare, is the most common form of botulism seen in the United States. Raw honey is a frequently mentioned etiology of infant botulism. Other causes include corn syrup and soil and vacuum dust. The initial clinical manifestation is constipation, followed by motor function symptoms such as ptosis, facial and generalized weakness. Clinical presentation and electromyography findings consistent with infant botulism allow for a presumptive diagnosis while confirmatory stool studies are pending. Treatment in infants under age 1 year is with human-derived botulism immune globulin (BabyBIG) and should be administered as early as possible in the course of the infection. Equine-derived heptavalent botulinum antitoxin is used in non-infant cases of botulism.

A 5-year-old boy presents with his mother due to concerns for fever for the past six days. She states that she initially thought he had a virus; however, he seems to be worsening over time. On physical exam, his vital signs are significant only for a fever or 38.4ºC. He has bilateral conjunctival injection and fissured lips. There is a macular, erythematous rash on areas of the trunk and the extremities, as well as edema of the dorsal aspects of the hands. Which of the following is the most likely diagnosis? AAdenovirus BKawasaki disease CScarlet fever DToxic shock syndrome

Correct Answer ( B ) Explanation: Kawasaki disease is a vasculitis that affects medium-sized vessels such as the coronary arteries.The etiology is largely unknown. Symptoms include a fever lasting more than 5 days, bilateral nonexudative conjunctivitis, mucositis with fissured lips and a "strawberry tongue", and edema of the dorsal aspect of the hands and feet. Some children may also have lymphadenopathy and rash. The diagnosis can be made if a fever and at least four of the five other symptoms of Kawasaki disease are present. The rash presentation can vary, but is generally macular, morbilliform, or targetoid. Children may initially have upper respiratory or gastrointestinal manifestations with the onset of the fever. This can lead to the disease being initially mistaken for a viral illness. Diagnostic tests may reveal elevated inflammatory markers and reactive thrombocytosis. Treatment is initiated with intravenous immune globulin (IVIG) therapy and aspirin. Children with Kawasaki disease need a complete cardiac evaluation and monitoring as myocardial infarction, coronary artery aneurysm, and heart failure are serious complications.

An 8-year-old girl presents to the emergency department with a one-day history of rash. The rash is erythematous, macular, blanching, and covers her trunk and extremities. It is not painful or pruritic, and there are no vesicles, bullae or mucosal lesions noted. She has bilateral lymphadenopathy in the posterior cervical chains with splenomegaly on palpation of her abdomen. The rest of her physical exam is normal. Her pediatrician started an antibiotic last week for pharyngitis but she has not improved. What is the most likely etiology of her rash? DRESS Syndrome Mononucleosis Stevens-Johnson Syndrome Type I Hypersensitivity reaction

Correct Answer ( B ) Explanation: Mononucleosis is caused by the Epstein Barr virus (EBV) and presents clinically with fatigue, fever, pharyngitis, lymphadenopathy, and splenomegaly. Mononucleosis is associated with a faint, widespread, non-pruritic rash, which lasts for about a week. It is characterized by patches that may contain small bumpy red spots. It thought to be directly due to the virus. This rash often appears on the trunk and upper arms first, and a few days later extends to involve the face and forearms. This is in contrast to a more intense pruritic maculopapular or morbilliform which appears on extensor surfaces and pressure points 7 to 10 days after treatment with beta-lactam antibiotics such as ampicillin, amoxicillin and cephalosporins. This rash indicates a hypersensitivity reaction to the antibiotic. It is not a true allergy and does not occur if the antibiotic is given later on in the absence of EBV infection.The peak incidence of infection with EBV occurs in adolescence. The diagnosis can be made with EBV IgM and IgG or EBV PCR. Rare but life-threatening complications of mononucleosis include splenic rupture; therefore patients with mononucleosis should be instructed to avoid contact sports until their illness has resolved. Type 1 Hypersensitivity reactions (D) occur immediately after exposure to the antigen, rather than after 1 week of exposure. These reactions can present with urticaria, bronchospasm, or anaphylaxis. DRESS Syndrome (A), or drug-induced hypersensitivity syndrome, typically presents with fever, severe exanthematous rash that can be exfoliative, generalized lymphadenopathy, and hematologic abnormalities. It is usually associated with anticonvulsants, as well as minocycline, sulfonamides, nitrofurantoin, and allopurinol. This syndrome can be life threatening due to organ damage such as carditis, hepatitis, and nephritis. Eosinophilia is present in up to 50% of cases. Mucosal lesions are rarely seen in this disorder, differentiating it from Stevens-Johnson syndrome or toxic epidermal necrolysis.

Ocular erythromycin prophylaxis for newborns provides prophylaxis for which of the following? Chlamydia trachomatis Neisseria gonorrhoeae Streptococcus agalactiae Treponema pallidum

Correct Answer ( B ) Explanation: Ocular erythromycin is routinely administered to neonates to prevent the transmission of Neisseria gonorrhoeae from mother to child. Ocular prophylaxis is safe, effective, and inexpensive. Erythromycin ointment causes significantly less chemical conjunctivitis than silver nitrate, which was the previous standard of care for ocular prophylaxis. However, silver nitrate is more effective prophylaxis for patients with penicillinase-producing N.gonorrhoeae, and thus it is still indicated in areas where resistant N.gonorrhoeae is prevalent.

A 12-year-old uncircumcised boy presents to the emergency department with a complaint of inability to retract his foreskin to its normal position. While he was showering, he retracted his foreskin to clean his penis. He called his mother for help when he could not get his foreskin back into place. Which of the following is the most likely diagnosis? Balanitis Paraphimosis Phimosis Priapism

Correct Answer ( B ) Explanation: Paraphimosis occurs in uncircumcised males when entrapment of the foreskin behind the glans penis causes venous and lymphatic congestion, resulting in inability to retract the foreskin to its normal position. Risk factors include genitourinary procedures, sexual activity, penile trauma, and males who forget to replace the foreskin after cleaning or urination. Paraphimosis typically causes significant pain and is a true urologic emergency. Treatment includes the timely reduction of the foreskin back over the glans penis. Urology consultation should be ordered for any signs of arterial compromise of the glans penis. Balanitis (A) is an inflammation of the glans penis often caused by poor hygiene. Phimosis (C) occurs when the foreskin cannot be retracted over the glans penis. Priapism (D) is a prolonged erection of the penis.

Which of the following is true regarding pertussis? Childhood vaccination and clinical disease provide lifelong immunity Infectivity is greatest in the catarrhal stage Lobar consolidations are commonly seen on chest radiography The treatment of choice is oral amoxicillin

Correct Answer ( B ) Explanation: Pertussis (whooping cough) is an acute respiratory illness caused by Bordetella pertussis. Prior to the development of a vaccination, pertussis primarily affected children under the age of 10 years. Since the late 1990s, pertussis has become more prominent in adolescent and adult populations. The clinical course is divided into three stages. The first stage, or known as the catarrhal stage, is often indistinguishable from other respiratory infections. It begins after an incubation period of seven to 10 days and lasts for about a week. Symptoms include rhinorrhea, low-grade fever, malaise, and conjunctival injection. Infectivity is the greatest in the catarrhal stage. Stage two, known as the paroxysmal stage, begins as the fever subsides and the cough develops. Patients have paroxysms of a staccato cough followed by sudden forceful inhalation, producing the characteristic whoop. Only one-third of adult patients develop the whoop, and it is also rare in infants. Infants are at risk of apneic episodes during this stage. The paroxysmal stage lasts two to four weeks. The third, or convalescent stage, lasts several weeks to months and is characterized by a residual cough.

An 18-month-old girl presents to the Emergency Department with a rash. Mom reports that she has had a high fever for the last three days. Today, the fever broke, but she developed a maculopapular rash on her trunk as seen in the picture above. What is the likely diagnosis? Erythema infectiosum Roseola Rubeola Scarlet fever

Correct Answer ( B ) Explanation: Roseola infantum is caused by human herpesvirus 6 and the vast majority of cases are seen in children under two years old. The disease is characterized by a high fever (often as high as 41 oC) for 3-5 days, which ends abruptly and is followed by the onset of a diffuse fine maculopapular rash on the trunk which may spread to the face and extremities. There is no mucous membrane involvement. Children affected are typically not ill appearing. The illness is self-limited, with the rash lasting only a few days. Treatment consists of fever control as needed.

A 7-week-old boy born at 34 weeks gestation presents to the emergency department with a rash. Two weeks prior to presentation he was treated with mupirocin for a skin infection. One day prior to presentation he had nasal congestion. On exam, he is noted to be febrile to 38.4ºC and very irritable. His skin is diffusely erythematous and tender when touched. There is a sloughing rash on the anterior folds of his neck, the diaper region, and lips. What is the most likely diagnosis? Bullous impetigo Staphylococcal scalded skin syndrome Stevens-Johnsons syndrome Toxic epidermal necrolysis

Correct Answer ( B ) Explanation: Staphylococcal scalded skin syndrome (SSSS) is a toxin-mediated type of exfoliative dermatitis that causes separation of the epidermis from the dermis. Patients typically have widespread painful erythema and large, flaccid bullae and desquamation typically in the groin, body orifices and axillae. Exotoxins and epidermolytic toxins of Staphylococcus aureus that cleave desmoglein 1 cause it. The initial infection can range from purulent conjunctivitis, abscess, otitis media or occult nasopharyngeal infection. Initial symptoms usually include fever, irritability and a generalized erythematous, painful rash. Workup involves cultures of the nasopharynx, conjunctiva and areas of involved skin as well as blood culture. Treatment centers on treating the staphylococcal focus of infection and supportive care with emolients

A 10-year-old child presents for a well child check. He is currently living in a shelter with his family. A tuberculin skin test is placed subcutaneously. Which of the following meets the minimum criteria for a positive test in 48 hours? 10 mm diameter of erythema 10 mm diameter of induration 15 mm diameter of induration 5 mm diameter of induration

Correct Answer ( B ) Explanation: The above child has recently been living in a shelter environment. Exposure to a homeless population is a risk factor for exposure to tuberculosis. Other risk factors for exposure include birth in high-prevalence regions of the world or travel to high-prevalence regions of the world. Increased risk of tuberculosis exposure also occurs through exposure to adults with HIV infection, users of illicit drugs, residents of nursing homes, incarcerated persons, or institutionalized persons. Children who are at increased exposure risk should undergo tuberculin skin testing with follow up injection-site inspection at 48-72 hours. A positive test in a child with known tuberculosis risk factors is 10 mm diameter of induration or greater. Notably, only the diameter of induration can be used for measurement of the skin reaction; erythema alone cannot be included in measurement.

A 13-month-old boy is brought by his anxious parents to the clinic because of a rash. For the previous three days, he has been having high-grade fever that is lysed by ibuprofen. Then today, he became afebrile, and the mother noted the appearance of a rash on the neck and trunk. On physical examination, his temperature is 37.3ºC, heart rate is 104 beats per minute, respiratory rate is 25 cycles per minute. His skin examination reveals blanching maculopapular rash on the neck and trunk that has spread to the face and extremities. Which of the following is the most likely diagnosis? Measles Roseola Rubella Scarlet fever

Correct Answer ( B ) Explanation: The boy has a constellation of signs and symptoms that is suspicious for roseola. It is an illness of young children, with peak prevalence between seven and 13 months. Ninety percent of cases occur in children younger than two years. Roseola occurs equally in boys and girls. Classical roseola begins with a fever that may exceed 40oC and lasts for three to four days. The fever often is accompanied by irritability, although most children with roseola are otherwise appear well, active, and alert. As the child's fever abates, a blanching macular or maculopapular rash develops, starting on the neck and trunk and spreading to the face and extremities. Occasionally, the rash is vesicular. It is generally nonpruritic. The rash typically persists for one to two days, but occasionally may come and go within two to four hours.

A five-year-old boy is brought to the ED by his mother for leg pain. He was playing with his sister when he fell and cried. He cannot move his right lower leg. The mother had a hard time explaining how the boy sustained the injury. She said he had several fractures in the past. His father also has history of multiple fractures and hearing loss. On examination, the boy's height is at the 5th percentile, he has normal motor and sensory examination of the right foot and ankle with strong pulses, slight swelling of the right leg with tenderness at the tibia. You suspect a right tibial fracture. Which of the following is the most likely diagnosis? Child abuse Osteogenesis imperfecta Osteomalacia Rickets

Correct Answer ( B ) Explanation: The boy has history of multiple fractures with a family history of multiple fractures and hearing loss. These findings are suspicious for osteogenesis imperfecta (OI). OI or "brittle bone disease" is an inherited connective tissue disorder. Most patients have an autosomal dominant mutation in the gene that affects the structure of one of the two alpha chains of type I collagen. The severity of the clinical presentation depends upon the effect of the mutation. The clinical manifestations vary within families. Clinical manifestations of OI include excess or atypical fractures, short stature, scoliosis, basilar skull deformities, blue sclerae, hearing loss, opalescent teeth that wear quickly, increased laxity of the ligaments and skin, wormian bones, and easy bruisability. There are reports of elevated alkaline phosphatase and hypercalciuria. The clinical diagnosis is based on the signs and symptoms as described, which is straightforward in individuals with bone fragility and a positive family history or several extraskeletal manifestations. Osteomalacia (C) can present as bone pain, fractures, and alkaline phosphatase elevation. Hearing loss or blue sclerae are not present. Rickets (D) can lead to slow growth, bone deformities, elevation of alkaline phosphatase, defective bone mineralization, and abnormal tooth formation. However, scleral abnormalities and hearing loss typically do not occur.

A three-year-old girl presents with a diffuse maculopapular rash that is most prominent in the bilateral axillae. The rash is pruritic, worse at night, and excoriations are present over the extremities. What is the treatment of choice? Hydrocortisone Permethrin Reassurance Topical emollients

Correct Answer ( B ) Explanation: The child's presentation is most concerning for scabies. Scabies is caused by the mite Sarcoptes scabiei. It's incubation period is three to six weeks after the time of acquisition through direct contact. At this time, the affected individual develops an intensely pruritic, maculopapular rash. Individual lesions may also develop a fine crust. Lesions may also result in burrows, 2 to 15 mm linear streaking that results from movement of the mite within the skin. The rash classically involves the interdigital webs, wrist flexors, elbow and knee extensors, lower buttocks and thighs, and axillae. The treatment of choice is application of topical permethrin from the neck down. The cream should be left on overnight and be removed by washing eight to 14 hours later. All family members should be treated regardless of current symptomatology because re-infestation is otherwise very common. An alternative treatment of scabies is lindane, but due the risk of toxicity, lindane is recommended by the US CDC only for those who cannot tolerate topical permethrin. Linens should be washed in hot water and then dried on high heat. Items that cannot be washed should be placed in plastic bags for at least three days so that the mites will die.

A five-year-old boy presents to the emergency department with nasal bleeding lasting greater than two hours. In the emergency department, the nurse applied pressure for 20 minutes without successful resolution. The patient appears well and has a normal heart rate and blood pressure. The patient's mother reports that the child's father has a bleeding disorder for which he occasionally uses a nasal spray. The patient has never had testing, and this is his first physician encounter for bleeding. Which lab abnormality would you expect to find for the child's inherited condition? Decreased platelet count Increased bleeding time Increased platelet count Increased prothrombin time

Correct Answer ( B ) Explanation: The condition described is von Willebrand disease (VWD). It is the most common hereditary bleeding disorder. The most common type, type I, is inherited in an autosomal dominant fashion. Increased tendency to bleed is the result of a qualitative or quantitative defect in von Willebrand factor (VWF), a protein necessary for platelet adhesion function. The resultant platelet type bleeding commonly affects mucosa leading to dental and nasal bleeding as well as menorrhagia and increased bleeding after delivery of a child. Testing for VWD requires examination of the blood for both quantitative and qualitative deficiencies of VWF. VWF antigen assay evaluates quantity of von Willebrand factor. The level of functionality of VWF is tested with one of the following: glycoprotein binding assay, collagen binding assay, ristocetin cofactor activity (RiCof) or a ristocetin-induced platelet agglutination (RIPA) assay. Bleeding time is increased. PTT is normal or may be increased.

You are performing a newborn exam on a one-day-old baby girl. In doing so, you hear a continuous, rough, machinery-like murmur during cardiac auscultation. Which of the following heart lesions is most closely associated with the murmur you heard? Atrial septal defect Patent ductus arteriosus Still's murmur Ventricular septal defect

Correct Answer ( B ) Explanation: The ductus arteriosus (DA) in utero shunts blood to the aorta, bypassing fetal lungs, and normally closes within the first five days of life. When a DA remains open longer than one week, it is considered abnormal and is referred to as a patent ductus arteriosus (PDA). This condition is more common in females than in males, and has an increased association with maternal rubella. Clinical findings include a loud, continuous, machinery-like murmur best heard at the left upper sternal border. In addition, infants with a PDA may show signs of labored breathing, bounding pulses, and tire easy with feeds. Closure of the patent ductus arteriosus is stimulated by administration of prostaglandin synthesis inhibitors, such as indomethacin or aspirin, which is effective in premature infants. Surgical correction is often required for term infants. Still's murmur (C) is the most common innocent murmur in childhood. It typically occurs in children from age two until six. The murmur associated with a Still's murmur is musical and vibratory in nature. It is early, systolic, high-pitched, and heard best at the heart's apex. Most importantly it increases with the patient in supine position and diminishes when the patient sits or stands.

A 16-year-old girl is in the clinic for a follow-up for a fever. She has been having prolonged fever accompanied by fatigue, night sweats, and loss of appetite. On physical examination, she has a temperature of 38°C with an enlarged cervical lymph node on the right that is painless, rubbery, and firm. There is no hepatosplenomegaly. She is then referred for excisional biopsy of the enlarged lymph node that shows Reed-Sternberg cells. Which of the following is the most likely diagnosis? Epstein-Barr virus infection Hodgkin lymphoma Metastatic adenopathy Mycobacterial infection

Correct Answer ( B ) Explanation: The girl has constitutional symptoms with an enlarged lymph node showing Reed-Sternberg cells. These findings are consistent with Hodgkin lymphoma (HL). HL is a malignant lymphoma that accounts for approximately 7 percent of childhood cancers and 1 percent of childhood cancer deaths in the United States. The incidence of HL in childhood varies by age such that it is exceedingly rare in infants, but is the most common childhood cancer in the 15- to 19-year-old age group. Most children with HL present with painless lymphadenopathy, which is usually cervical, supraclavicular, axillary, or inguinal. The affected lymph nodes typically feel rubbery and more firm than inflammatory adenopathy. They may be sensitive to palpation if they have grown rapidly. Patients with HL may present with nonspecific systemic symptoms, including fatigue, anorexia, and weight loss. Patients are classified to have "B" symptoms when they present with fever (> 38.0°C), drenching night sweats, and weight loss (≥ 10 percent loss within six months before diagnosis). The B symptoms have an important implication for staging and prognosis. The diagnosis of HL is established by histologic examination, usually by excisional biopsy of an enlarged lymph node that demonstrates malignant classic Reed-Sternberg cells or their variants.

A six-year-old girl is brought by her mother to the clinic due to fever and sore throat. The girl has had a fever for five days with a maximum temperature of 38.5°C that is minimally responsive to antipyretics. She is irritable with decreased appetite. On examination, there is bilateral bulbar injection, erythema of oral mucosa, enlarged anterior cervical nodes, and macular rash on trunk and extremities. Which of the following is the most likely diagnosis? Adenovirus Kawasaki disease Rocky Mountain spotted fever Streptococcal pharyngitis

Correct Answer ( B ) Explanation: The girl in the vignette is manifesting signs and symptoms that are suspicious for Kawasaki disease (KD). KD is also known as mucocutaneous lymph node syndrome and is one of the most common vasculitides in children. There is widespread inflammation of medium-sized muscular arteries. Diagnosis of KD requires the presence of fever that lasts ≥ 5 days with four of the five physical findings without an alternative explanation. These findings include bilateral bulbar conjunctival injection, oral mucous membrane changes (injected or fissures lips, injected pharynx, or strawberry tongue), peripheral extremity changes (erythema of palms or soles, edema of hands or feet, or periungal desquamation), polymorphous rash, and cervical lymphadenopathy. Management of KD involves treatment with intravenous immune globulin (IVIG) within the first 10 days of illness that reduces the prevalence of coronary aneurysms.

A 12-month-old girl is in the clinic for a well-child check. She can walk and is eating more table foods. The parents have given her goat's milk for two months. Physical examination is normal. Laboratory screening obtained shows hemoglobin of 10 g/dL, mean corpuscular volume 77 fL and elevated red cell distribution width. Which of the following is the most likely diagnosis? Folate deficiency Iron deficiency anemia Thalassemia Vitamin B12 deficiency

Correct Answer ( B ) Explanation: The girl's complete blood count shows anemia that has low mean corpuscular volume (MCV) and elevated red cell distribution width that is suspicious for iron deficiency anemia (IDA). The World Health Organization (WHO) estimates that anemia affects one quarter of the world's population and is concentrated within preschool-aged children and women. The majority of the anemia is due to iron deficiency. Risk assessment for iron deficiency for all infants is recommended at 4, 15, 18, 24, and 30 months of age, and annually thereafter. The most important screening test is a focused dietary history, and it is more useful than hemoglobin levels. The primary risk factors for iron deficiency are: history of prematurity or low birth weight, use of low iron formula, or feeding of non-formula cow's milk, goat's milk, or soy milk for the milk-based part of the diet before 12 months of age or fewer than two servings per day of iron-rich foods (meats or fortified infant cereal) after six months of age. The American Academy of Pediatrics suggests universal laboratory screening for iron deficiency at approximately one year of age. A presumptive diagnosis of iron deficiency anemia (IDA) is made by the combination of risk assessment and laboratory screening (hemoglobin < 11 g/dL). A low mean corpuscular volume (MCV) and elevated red cell distribution width (RDW) further support the diagnosis.

A 4-month-old infant presents with erythematous plaques to the bilateral cheeks that have been worsening over the past three weeks. Similar plaques are noted over both elbow extensors. He has been growing and developing normally. What is the treatment of choice? Reassurance Topical emollients Topical mupirocin Topical triamcinolone

Correct Answer ( B ) Explanation: The infant has atopic dermatitis, also known as eczema. In infants, eczema is most common on the cheeks and extensor surfaces of the extremities. This distribution is in marked contrast to in children and adolescents who classically have involvement of the flexor surfaces, such as the antecubital and patellar fossas. The first-line treatment for eczema is a topical emollient. Thick, unscented creams and ointments and preferred to lotions. Emollients should be applied at least twice per day and re-applied after bathing, swimming, or hand-washing. If the eczema is persistent, a topical steroid may be applied in addition to topical emollients. The lowest effective potency of corticosteroid should be prescribed, and steroids should be applied only to affected skin to avoid the side effects of skin thinning, atrophy, and telangectasias. Desonide or hydrocortisone are good initial low-potency topical corticosteroids.

A female infant is born via spontaneous vaginal delivery to a 23-year old G3P3 with no prenatal care at 37 weeks gestation. On physical exam she is found to be small for gestational age and has hepatosplenomegaly, jaundice, and a grade II/VI continuous cardiac murmur. An ophthalmologic exam reveals micro-opthalmia and cataracts. Her CBC reveals thrombocytopenia. Which maternal prenatal screening labs would have indicated these exam findings? Group B strep cultures Rubella immunity status Varicella Zoster screening VDRL screening

Correct Answer ( B ) Explanation: The major complication of rubella is its teratogenic effects when pregnant women contract the disease, especially in the early weeks of gestation. The virus can be transmitted to the fetus through the placenta and is capable of causing serious congenital defects, abortions, and stillbirths. Fortunately, because of the successful immunization program initiated in the United States in 1969, rubella infection and congenital rubella syndrome rarely are seen today. Rubella immunity status is a part of the routine prenatal screening that is performed at the first prenatal visit. Sensorineural hearing loss is the most common manifestation of congenital rubella syndrome. Ocular abnormalities including cataract, infantile glaucoma, and pigmentary retinopathy occur in approximately half of children with congenital rubella syndrome. Both eyes are affected in almost all patients, and the most frequent findings are cataract and rubella retinopathy. Congenital heart disease is present in half of infants infected in the first 2 months' gestation. Other findings in congenital rubella syndrome include the following: intrauterine growth retardation, CNS abnormalities, hepatosplenomegaly, jaundice, blueberry muffin skin spots, bone lesions, hematologic disorders such as anemia and thrombocytopenia.

A mother brings her 9-year-old daughter to the ED because she is complaining of nighttime itching in the anal area ever since returning from summer camp one week ago. What is the appropriate treatment for this condition? Clotrimazole cream Mebendazole Mupirocin ointment Sitz baths

Correct Answer ( B ) Explanation: The patient has enterobiasis (pinworms). The treatment is with single-dose mebendazole (or albendazole), an oral anthelmintic. This infection is most commonly seen in children between the ages of five and fourteen. The condition is caused by ingesting eggs of Enterobius vermicularis. The eggs hatch within the intestinal tract, and female parasites lay new eggs in the perianal area, leading to itching (pruritus ani) that is characteristically worse at night. Secondary skin changes and bacterial infection may also be present. Other associated problems may include insomnia, enuresis, UTIs, and vaginitis. All family members should be treated. In households where more than one member is infected or where repeated, symptomatic infections occur, it is recommended that all household members be treated at the same time

An 11-year-old girl presents with a rash for the past 2 weeks. Her mother states that the rash began as a single red spot on the patient's upper back and then spread to the rest of her body. On physical exam, you note pink maculopapular oval patches on the patient's trunk. What treatment is indicated? Acyclovir Diphenhydramine Topical ketoconazole cream Trimethoprim-sulfamethoxazole

Correct Answer ( B ) Explanation: The patient has pityriasis rosea. It most commonly occurs in the spring and fall seasons and is seen in older children and young adults. The rash initially presents with a single red lesion on the trunk, known as a herald patch. In about one to two weeks, the rash progresses to pink maculopapular oval patches covering the trunk in a characteristic "Christmas tree" pattern. Patients with pityriasis rosea require symptomatic treatment, including diphenhydramine to relieve itching. The rash resolves within 3-8 weeks and is not contagious

A 16-year-old girl is seen in clinic due to vaginal discharge. She complains of gray discharge with a fishy odor. She is sexually active for the past six months and has had one partner. She uses an intrauterine device for contraception. On physical exam you note gray vaginal discharge with a vaginal pH of 5. You then perform a wet mount which shows clue cells. What is the most likely etiologic agent? Candida albicans Gardnerella vaginalis Lactobacillus spp. Trichomonas vaginalis

Correct Answer ( B ) Explanation: The patient has signs and symptoms of vaginitis. The cause is most likely due to bacterial vaginosis wherein the normal vaginal flora is replaced by an overgrowth of anaerobic microorganisms. The most common organism involved is Gardnerella vaginalis. Other organisms include Ureaplasma sp. and Mycoplasma sp. Bacterial vaginosis presents with a fishy odor and a discharge that can be white, yellow or gray. The vaginal pH is > 4.5 and wet mount shows clue cells or epithelial cells with borders obscured by bacteria. Treatment is indicated for relief of symptoms and for reducing the risk of acquiring sexually transmitted infections. Metronidazole or clindamycin can be used orally or intravaginally

A six-year-old girl presents with dark urine and pedal edema and is found to have an elevated blood pressure. She was treated for strep pharyngitis two weeks ago. Which of the following is the next best step? Obtain a blood count Obtain a urinalysis Obtain antistreptolysin titers Obtain serum complement levels

Correct Answer ( B ) Explanation: The patient in this vignette likely has poststreptococcal glomerulonephritis (PSGN). In any patient with concern for renal disease, a urinalysis should be obtained. In patients with renal disease, urine dipstick results show large amounts of blood and protein. Microscopic analysis usually yields leukocytes and white blood cell casts. Red blood cells may appear dysmorphic and can be present with red blood cell casts. Red blood cell casts may or may not be related to PSGN but is pathognomonic for glomerular disease in general.

What is the most common historical exposure found in cases of infantile botulism? Construction site dust with botulinum spores Contaminated honey Contaminated infant formula Contaminated wounds

Correct Answer ( B ) Explanation: There are three types of botulism: (1) Infantile where C. botulinum spores are ingested, leading to intestinal tract colonization and toxin release; (2) Foodborne where preformed toxins, usually in canned foods, are ingested; and (3) Wound where a wound is directly exposed to C. botulinum. Infant botulism is the most common type of botulism (> 70 percent of cases). Foodborne and wound botulism are not typically diseases of infancy or childhood. More than 20 percent of infant-type botulism is linked to honey exposure. While infantile botulism is linked to honey exposure, it is not caused by direct toxin ingestion and, therefore, is not considered foodborne. More than half of USA cases occur in California, and there is equal distribution between males and females. Clinically, botulism exerts its effects by the toxin it secretes. The toxin acts on pre-synaptic nerve terminals to block the release of acetylcholine by impairing the exocytosis process. This manifests in infants as bulbar palsies, hypotonia, and respiratory insufficiency requiring supportive ventilation. Diagnosis of infantile botulism cannot be made through serologic testing and requires stool culture and toxin assay. It is important to note that antibiotics are contraindicated since they can cause intestinal release of additional toxin.

A 7-year-old boy presents with a fever, cough, runny nose, and conjunctivitis for 3 days followed by the above pictured rash that initially appeared on his face. Exam of his oral mucosa reveals pinpoint gray spots. Which of the following is the most likely diagnosis? Erythema infectiosum Measles Rocky Mountain spotted fever Rubella

Correct Answer ( B ) Explanation: This patient has a typical presentation for measles (rubeola). The measles virus is a respiratory tract pathogen that is highly communicable. After exposure, the virus incubates for 10-14 days. The patient then develops cough, coryza and conjunctivitis along with fever. 2-4 days later, the characteristic rash begins on the face and spreads down the trunk then fades in a head-to-toe direction. Additionally, Koplik's spots, pinpoint gray spots with surrounding red inflammation, develop on the buccal mucosa and are pathognomonic for measles. Treatment is supportive Erythema infectiosum (A) is usually a nonfebrile disease. Rubella (D) is also characterized by a maculopapular rash and fever but does not have Koplik's spots. Rocky Mountain Spotted Fever (C) results from a tick bite and starts on the extremities and moves centrally.

6-year-old boy is brought in by his mother because his teachers noticed that he frequently tends to stare off while doing his schoolwork and will not respond to their questions. His mother states that she has seen him stare off during dinnertime but assumed that he was just ignoring her. What is your next step in the evaluation of this child? Order a MRI Order an EEG Reassure the mother and tell her it is likely nothing Send him to the ED for urgent treatment

Correct Answer ( B ) Explanation: This patient is having absence seizures (petit mal) and requires an EEG. Absence seizures are characterized by sudden-onset staring spells, usually lasting approximately 10 seconds, with immediate recovery. Clinically, patients may experience eye fluttering or altered postural tone that may occur many times per day. The diagnosis is confirmed by EEG, which shows characteristic generalized 3 Hz spike-and-wave discharges. Childhood absence epilepsy is an idiopathic generalized epilepsy that has an onset between 4 and 14 years of age with a peak at 5 to 7 years. The prognosis is generally favorable with approximately 80% of children outgrowing their seizures by adolescence.

A 10-year-old boy presents to your office with complaints of fever, malaise, and facial swelling that began four days ago. He is not up-to-date on his immunizations. Physical examination reveals a temperature of 99.8OF and enlarged parotid glands. Which of the following is the most appropriate therapy? Acyclovir Amoxicillin Ibuprofen Oseltamivir

Correct Answer ( C ) Explanation: Mumps is an acute viral illness caused by rubulavirus. Prior to the development of the MMR (measles, mumps, rubella) vaccine, mumps commonly affects grade school-aged children. Cases have decreased significantly due to a widespread vaccination program, although sporadic outbreaks can occur. Infection with the virus causes low-grade fever, headache, malaise, and swelling of bilateral salivary glands, typically the parotid glands. Diagnosis is based on the presence of parotiditis, along with a history suggestive of mumps. The course of illness is self-limited, with treatment being supportive measures, including ibuprofen or acetaminophen. Prevention of illness is through immunization with the mumps vaccine. Viral meningitis and unilateral acquired sensorineural deafness were the most common sequelae.

A previously healthy 17-year-old boy is brought to the ED by ambulance. He became very ill over the past few hours. His vital signs are T 39.4ºC, HR 142, BP 90/52, RR 20, and a pulse oximetry of 94% on room air. On exam, he has a stiff neck and the rash seen above on his leg. You establish an intravenous line, draw a blood culture, order a complete blood count and electrolyte assessment, and administer a fluid bolus. Which of the following is the most appropriate next step? Administer hydrocortisone to treat hemorrhagic adrenalitis (Waterhouse-Friderichsen syndrome) Administer intravenous ceftriaxone and vancomycin Perform a CT scan of the head Perform a lumbar puncture

Correct Answer ( B ) Explanation: This patient is presenting with severe meningococcal septicemia caused by the aerobic gram-negative diplococcus N. meningitidis, a natural organism living in the nasopharynx of humans, its only host. Patients with this infection may progress rapidly. The lumbar puncture may be deferred until the patient is stable and should not delay antibiotic administration. Although N. meningitides continues to be sensitive to penicillin, ceftriaxone and vancomycin should be administered to cover for N. meningitides and resistant pneumococcal meningitis, which may also cause purpura fulminans. Hemorrhagic adrenalitis is an adrenal gland failure due to bleeding into the adrenal glands, caused by severe bacterial infection (most commonly N. meningitidis). The bacterial infection leads to massive hemorrhage into one or (usually) both adrenal glands. Treatment with antibiotics is the primary treatment. Hydrocortisone (A) can sometimes reverse the hypoadrenal shock. Although the patient's mental status change can indicate an elevated intracranial pressure from meningococcal meningitis inflammatory changes, his presentation with low blood pressure and tachycardia is more consistent with severe meningococcal septicemia.

An eight-year-old girl presents to your office with complaints of increased thirst, increased volume of urine and weight loss. Physical exam is normal except for the patient appearing slightly ill and with a weight loss of seven pounds since her last well child exam. Hemoglobin A1C is 9.2%. Which of the following is the most appropriate therapy? Glipizide Insulin Metformin Repaglinide

Correct Answer ( B ) Explanation: Type 1 diabetes mellitus (DM) occurs when autoimmune destruction of the beta cells of the pancreas make the body unable to produce insulin. Onset of the condition occurs most commonly in children, but it can also develop in adults. Family history of type 1 DM is a risk factor for developing the disorder. The classic symptoms include polyuria, polydipsia, polyphagia, and weight loss. Onset can occur suddenly, and individuals often present with diabetic ketoacidosis as the initial presentation. Diagnosis can be confirmed with a random plasma glucose concentration of 200 mg/dL or higher in a patient with symptoms of hyperglycemia, a fasting plasma glucose concentration of 126 mg/dL or higher on more than one occasion, a positive oral glucose tolerance test or a glycated hemoglobin (A1C) of 6.5 percent or greater. Treatment is with exogenous insulin since the body is unable to produce insulin on its own.

A 16-year-old girl presents to the ED with difficulty breathing. She has had no fevers, rhinorrhea, or cough and her symptoms began abruptly two days ago while watching television. She is wearing a right shoulder immobilizer for a scapula fracture she sustained two weeks ago. Her only medications include oral contraceptive pills and ibuprofen. Her respiratory rate of 20 breaths/minute, heart rate 110 beats/minute, and oxygen saturation is 99% on room air. She is alert and her lungs are clear to auscultation. An X-ray reveals a shallow wedge-shaped opacity in the periphery of the lung with its base against the pleural surface. Of the following, which is the next most appropriate step once the likely diagnosis is confirmed? Intravenous ampicillin Intravenous furosemide Intravenous heparin Tube thoracostomy placement

Correct Answer ( C ) Explanation: A pulmonary embolism is an obstruction of a pulmonary artery or its branches due to a thrombus. Depending on the degree of obstruction this may present with varying degrees of symptoms. It can present mildly as it has in our patient or can present as severely as cardiopulmonary arrest. Consider a pulmonary embolism in patients with difficulty breathing and Virchow's triad including stasis (e.g. shoulder immobilization), venous injury (eg. recent trauma), and hypercoagulability (e.g. taking oral contraceptives). Physical exam findings are often absent, with the most common findings being tachycardia and tachypnea. The gold standard for diagnosis is pulmonary angiography although spiral CT with contrast is non-invasive, sensitive and specific and thus more frequently used. A peripheral infiltrate known as "Hampton's Hump" due to infarcted lung is rarely seen on X-ray and is easily confused with a bacterial pneumonia. Patients with a pulmonary embolism who are hemodynamically stable should be started on anticoagulation with low molecular weight heparin or unfractionated heparin.

A 9-year-old boy presents with penile pain. On genitourinary exam the distal prepuce cannot be reduced over the glans penis and there are thin adhesions to the glans, which is swollen. What is the next best step in management? Dorsal penile slit procedure Emergency circumcision Manual reduction after topical lidocaine Topical corticosteroids

Correct Answer ( C ) Explanation: Attempt at manual reduction is indicated for paraphimosis which describes the inability to reduce the foreskin over the glans penis to its natural position. It is a medical emergency and can cause venous engorgement and glans edema which can lead to ischemia. It most often occurs when the foreskin is forcibly retracted to clean the penis and the foreskin is not returned to its natural position. Boys younger than age four are particularly susceptible to paraphimosis because their foreskin is not fully mobile. To manually reduce the paraphimosis, pain should first be controlled and then the foreskin should be circumferentially compressed to force the edematous fluid along the penile shaft. Then, the foreskin is pulled back over the glans.

A 3-year-old boy is brought to the pediatric emergency department by his mother. She tells you he has had a fever and cough for the past 2 days. The boy appears to be agitated and is in respiratory distress. His temperature is 38.6°C (101.5°F) and his respiratory rate is 35/min. Physical examination shows subcostal retractions. Auscultation of the lungs reveals inspiratory stridor with a prolonged inspiratory phase. Chest X-ray shows subglottic narrowing. Which of the following is the most likely diagnosis? Acute epiglottitis Foreign body aspiration Laryngotracheobronchitis Respiratory syncytial virus

Correct Answer ( C ) Explanation: Based on the constellation of findings, this patient most likely has croup (ie, laryngotracheobronchitis). Croup typically presents with the classic triad of barking cough, stridor, and hoarseness. It is most often caused by a viral infection (parainfluenza). Treatment usually consists of oral steroids (eg, dexamethasone) or nebulized epinephrine for severe cases. Although chest X-rays are typically not required, if done, they will usually show subglottic narrowing (ie, "steeple sign") as in this case Acute epiglottitis (A) is caused by H. influenza type b. It is typically distinguished from croup by the absence of a barking cough. Additionally, these patients will present with difficulty swallowing and drooling.

A 2-year-old previously healthy female presents to the ED with complaints of 3 days of low-grade fever and congestion followed by noisy breathing and cough. Upon examination, you note a frequent barking cough, audible stridor at rest, and retractions. The child does not appear to be in any distress. Which of the following is the most appropriate next step in management? AThe patient has mild croup and should be discharged after single dose of steroid and 5 mg albuterol nebulizer BThe patient has moderate croup and should receive oral steroids and be discharged after racemic epinephrine nebulizer with next-day follow-up CThe patient has moderate croup and should receive oral steroids and racemic epinephrine nebulizer and be observed in the ED for 3 hours and discharged if symptoms abate DThis patient has mild croup and can be discharged after a single dose of oral steroids

Correct Answer ( C ) Explanation: Croup (laryngotracheitis) is the most common cause of infectious acute upper airway obstruction (stridor). The etiology is viral (parainfluenza, influenza, and RSV) with erythema and swelling of the trachea just below the vocal cords. Patients classically present with a barky or seal-like cough. The mean age of affected patients is 18 months. There is a seasonal increase in autumn and early winter. Because the lungs are not directly affected, oxygen saturation can be maintained, even in severe illness. Aerosolized epinephrine (R or L) decreases airway obstruction. It is indicated for children with stridor at rest or marked work of breathing (tachypnea, retractions, accessory muscle use). Maximal effect is seen within 30 minutes, with potential rebound to baseline within 3 hours. Patients without resting stridor after 3 hours can be safely discharged home. A single dose of oral dexamethasone decreases the need for hospitalization and return ED visits.

Which of the following congenital disorders results in a cyanotic child? Atrial septal defect Coarctation of the aorta Tetralogy of Fallot Ventricular septal defect

Correct Answer ( C ) Explanation: Cyanotic heart disease results from either right-to-left shunting of desaturated blood flowing into the systemic circulation or from decreased pulmonary blood flow. Tetralogy of Fallot is the classic CHD but this group also includes truncus arteriosus, dextroposition (or transposition) of the great arteries, tricuspid atresia and total anomalous pulmonary venous return. The majority of these disorders are identified when patients are still in the nursery.The abnormalities seen in Tetralogy of Fallot include 1) right ventricular outflow tract obstruction, 2) over-riding aorta, 3) ventricular septal defect (VSD) and 4) right ventricular hypertrophy (RVH). An easy mnemonic to remember this is "PROVe" (Pulmonary stenosis, RVH, Overriding aorta, and VSD). Patients typically present with cyanosis that is worse during feeding and crying. They often have poor feeding and in older children, they may squat to relieve symptoms. Patients may experience "tet spells" characterized by acute respiratory distress due to increased right outflow tract obstruction. Patients with severe right ventricular outflow obstruction may present early after birth with profound cyanosis requiring reopening of the patent ductous arteriosus. Patients will have a systolic ejection murmur along the left sternal border. Chest radiography typically shows the classic "boot-shaped" heart. Cyanosis can be recreated by any valsalva maneuver as this results in an increase in the right-to-left shunt across the VSD. Definitive treatment is surgical correction.

A family brings in their 3-month-old infant who was just adopted from Honduras. They are not aware of the infant's birth history and were told that the child was healthy and lived in an orphanage for the first three months of life. They are concerned that he has significant bruising over his back. On examination, a large blue patch is present over the sacrum. Similar patches are present over the bilateral shoulders. The remainder of the examination is benign. Which of the following is the most likely diagnosis? Child abuse Coagulopathy Dermal melanosis Nevus of Ota

Correct Answer ( C ) Explanation: Dermal melanosis, also known as a Mongolian spot, represents a benign persistence of dermal melanocytes in neonates. It presents as large blue patches that are most commonly located over the sacrum and shoulders. Rarely, the face and flexor surfaces of the extremities may be involved. It is more common amongst all non-Caucasian races. The natural history of dermal melanosis is a gradual fade during the first two years of life. Most lesions resolve completely during grade school. Extra-sacral locations are more likely to persist into adulthood. Persistent lesions remain benign, and no treatment is indicated. Nevus of Ota (D) is a rare dermal melanocytosis that preferentially involves the distribution of the first two trigeminal nerves. In contrast to dermal melanosis, its appearance is speckled or mottled and may be grey or blue-black in color. Underlying CNS melanosis or ocular melanoma may also occur, necessitating yearly ophthalmologic examination.

2-week old infant is brought to the office for a well child visit. The physical examination is completely normal except for a clunking sensation and feeling of movement when adducting the hip and applying posterior pressure. Which one of the following would be the most appropriate next step? Plain radiograph of the pelvis Reassurance that the problem resolves spontaneously in 90% of cases, and follow-up in 2 weeks Referral for orthopedic consultation Triple diapering and follow-up in 2 weeks

Correct Answer ( C ) Explanation: Developmental dysplasia of the hip is a common musculoskeletal condition in newborns. Developmental dysplasia of the hip encompasses both subluxation and dislocation of the newborn hip, as well as anatomic abnormalities. It is more common in firstborns, females, breech presentations, oligohydramnios, and patients with a family history of developmental dysplasia. Physical examination screening is recommended by the American Academy of Pediatrics and the Pediatric Orthopaedic Society of North America. The Ortolani, reducing a dislocated hip, and the Barlow, dislocating an unstable hip, maneuvers are the physical examination tests most commonly performed for detection of developmental dysplasia in early infancy. The American Academy of Pediatrics recommends ultrasound screening at six weeks for breech girls, breech boys (optional), and girls with a positive family history of developmental dysplasia of the hip. Infants with developmental hip dyplasia, whether treated or untreated, have a higher incidence of early-onset hip osteoarthritis in adulthood, but the incidence is lowest in those who receive early treatment. Experts are divided with regard to whether hip subluxation can be merely observed during the newborn period, but if there is any question of a hip problem on examination by 2 weeks of age, the recommendation is to refer to a specialist for further testing and treatment. Closed reduction and immobilization in a Pavlik harness, with ultrasonography of the hip to ensure proper positioning, is the treatment of choice until 6 months of age. The patient in the above scenario should be referred for orthopedic consultation.

A 3-year-old previously healthy, fully immunized boy presents after a seizure that occurred two hours prior to arrival. He has never had a seizure before. Today, he experienced a 35 second generalized tonic-clonic seizure. He had a postictal period of 20 minutes and is now at his neurologic baseline. Emergency Medical Services were called and he had a blood glucose level of 84 mg/dL on scene. He has had a cough and runny nose for the past day. Shortly before the seizure, he developed a fever to 39.3°C. On exam, he is a well appearing boy with a supple neck, no rashes, normal tympanic membranes, normal pharynx, clear lungs, and a soft abdomen. What diagnostic study should be obtained on this patient? Electroencephalogram Lumbar puncture No diagnostic studies are indicated Non-contrast computed tomography scan

Correct Answer ( C ) Explanation: Febrile seizures occur in 2-5% of the general pediatric population. Simple febrile seizures are defined as a generalized tonic-clonic seizure lasting < 15 minutes, associated with a fever, in a child 6 months to 5 years of age, and occurring only once in a 24 hour period. Febrile seizures are the most common type of pediatric seizures. The American Academy of Pediatrics states that no diagnostic studies are needed specific to a febrile seizure and the goal of evaluation should be identifying the source of the fever. In this child with a benign exam, rhinorrhea and a cough, a viral upper respiratory infection is the likely cause of the fever. Less than half of children will experience another febrile seizure and the risk of developing epilepsy is similar to the general population risk.

A four-year-old boy presents to the emergency department for inability to walk. He has had two days of leg pain and has been irritable for the past day. The morning of presentation, he refused to get out of bed or bear weight. On his initial examination, he has 1/5 strength in his lower extremities, and 4/5 strength in his upper extremities. Patellar reflexes cannot be elicited. Which of the following is the most likely CSF finding? WBC 120, Glucose 20, Protein 80 WBC 120, Glucose 65, Protein 80 WBC 2, Glucose 65, Protein 135 WBC 2, Glucose 65, Protein 40

Correct Answer ( C ) Explanation: Guillain-Barré syndrome is a postinfectious polyneuropathy that can affect patients of all ages. It typically follows a viral illness (e.g., Epstein Barr virus and cytomegalovirus) but has also been associated with some immunizations. Typically, the weakness begins in the lower extremities, then ascends along the trunk. Bulbar muscles may be involved, and respiratory failure can occur. Young children may present with refusal to bear weight, irritability, and flaccid tetraplegia. Tendon reflexes may be lost early or later in the course. Analysis of the CSF is helpful in making the diagnosis of Guillain-Barré. CSF studies will show an elevated protein level, typically twice the upper limit of normal. The white blood cell count in the CSF is normal. This dissociation between high CSF protein and normal cell counts (albuminocytologic dissociation) is diagnostic for Guillain-Barré syndrome. In addition, the CSF glucose level falls within the normal range. Bacterial and viral CSF cultures will be negative. Imaging studies may support the diagnosis. MRI findings might include contrast enhancement of the intrathecal nerve roots and the cauda equina on postgadolinium T1-weighted images. Aseptic or viral meningitis (B) may have an elevated opening pressure, with clear CSF. The glucose level is normal, but the protein will be elevated (> 50 mg/dL). The white blood cell count ranges from 10-1,000 cells/µL.

A five-year-old boy presents with a progressive rash on his legs and buttocks over the past two days along with intermittent abdominal pain and anorexia. His mother states he has not been as active lately because of pain in his knees and ankles. He was otherwise healthy up until a week ago when he started complaining of intermittent headache and fever which she treated with acetaminophen. Which of the following is a known complication of the patient's diagnosis? Appendicitis Hypertrophic pyloric stenosis Intussusception Necrotizing enterocolitis

Correct Answer ( C ) Explanation: Henoch-Schönlein Purpura (HSP) is the most common vasculitis affecting children. It is usually self-limiting and resolves after approximately four weeks. It is often preceded by a prodrome of headache, pharyngitis, fever, and anorexia. The clinical presentation is influenced by the deposition of IgA in blood vessel walls, which leads to the triad of a palpable purpuric rash seen on the legs and buttocks, abdominal pain, and arthritis. Intussusception is a known, but rare, complication of HSP and occurs more commonly in an ileo-ileal location, as opposed to the ileo-colic location seen in typical intussusceptions. For this reason, ultrasound is the preferred imaging modality over contrast enema in HSP intussusception. Intussusception occurs when one piece of bowel invaginates and telescopes into a more distal segment, causing intermittent abdominal pain whenever there is bowel peristalsis. Multiple studies have demonstrated ultrasound to be highly accurate in the diagnosis of intussusception. It is often the initial diagnostic test since it is non-invasive, does not require radiation exposure and can be reproduced at the bedside. On ultrasound, the intussuscepted bowel commonly has a sonodense center (bowel contents) surrounded by a sonolucent ring (bowel wall), giving a "donut appearance" or "target sign."

A two-year-old boy develops pallor following a viral upper respiratory tract infection. His mother reports a family history of "some kind of low blood levels". His serum studies reveal a normocytic anemia and the presence of spherocytes on the peripheral blood smear. Which of the following tests can be done to confirm the diagnosis? Direct Coombs test Hemoglobin electrophoresis Osmotic fragility test Serum reticulocyte count

Correct Answer ( C ) Explanation: Hereditary spherocytosis is a clinical diagnosis based on family history, peripheral blood smear, and presence of splenomegaly. Blood film should show spherocytes and reticulocytes due to the high turnover of red blood cells from hemolysis. The presence of spherocytes in the blood can be confirmed with an osmotic fragility test. This is done by incubating red blood cells in hypotonic saline in increasing dilutions. This causes the red blood cells to swell and eventually burst. The spherocytes more readily lyse than normal cells in hypotonic solutions. Clinical correlation is necessary since this test is not specific, and normal results can be present in up to 20 percent of patients. Direct Coombs test (A) is done to detect cell lysis resulting from an autoimmune process. The test detects the presence of antibodies directed at a patient's own red blood cells. Hemoglobin electrophoresis (B) is used to detect different types of hemoglobin and is used in the diagnosis of thalassemias and sickle cell disease. Serum reticulocytes (D) are elevated in many conditions that result in high red blood cell turnover. The presence of reticulocytosis alone does not confirm or exclude a diagnosis.

A six-year-old girl presents with a rash on her face, which she has had for the past seven days. It consists of erythematous plaques with yellow crusts as shown above. Further physical examination is unremarkable, and vital signs are stable. Which of the following is the most appropriate treatment? Oral erythromycin Topical hydrocortisone Topical mupirocin Topical terbinafine

Correct Answer ( C ) Explanation: Impetigo is a rash characterized by erythematous plaques with honey-colored crusts. It is most commonly caused by Staphylococcus aureus and group A Streptococcus. Risk factors include poor hygiene, malnutrition, and atopic dermatitis. Treatment of uncomplicated facial impetigo consists of topical mupirocin. Diffuse impetigo can be treated with oral erythromycin or oral cephalexin.

A 12-year-old boy presents to the emergency department with recurrent headaches. The headaches have been present for the past four weeks and are increasing in intensity. They are worse in the morning and when lying flat, and are associated with vomiting but no nausea. For the past few days, he has complained of blurry vision. His initial exam is notable for altered mental status, extensor posturing, and papilledema. Which of the following are the most likely vital sign abnormalities? Heart rate 120 beats/minute, blood pressure 150/100, irregular respirations Heart rate 130 beats/minute, blood pressure 80/30, temperature 40°C Heart rate 40 beats/minute, blood pressure 155/65, irregular respirations Heart rate 50 beats/minute, blood pressure 140/100, temperature 40°C

Correct Answer ( C ) Explanation: Increased intracranial pressure (ICP) may result from many etiologies including swelling, masses, or increases in blood or CSF volumes. Increased ICP leads to physiologic compensatory mechanisms, but uncompensated elevated ICP may result in brain ischemia and ultimately in cerebral herniation. Vital sign and physical exam findings associated with cerebral herniation are systemic hypertension with widened pulse pressure, bradycardia, irregular respirations, pupillary dilation (often unilateral), and extensor posturing. Therefore, a heart rate of 40, BP of 155/65, and irregular respirations is most consistent with elevated intracranial pressure. Management of increased ICP and impending cerebral herniation includes maintaining euvolemia and euglycemia, head of the bed elevation, hyperventilation, and either mannitol or hypertonic saline (3%) infusion. These patients should have a secure airway placed since they are at risk for apnea. A CT scan should be obtained to evaluate for intracranial masses. Complications include seizures, SIADH and salt-wasting, hyperglycemia, and motor and cognitive delays.

A 7-year-old girl presents to urgent care complaining of vaginal irritation over the last several days. Mom noticed that the patient's vaginal area is abnormally red and irritated and that her underwear has been slightly damp. The patient denies bleeding, urinary complaints, or abdominal pain. On exam, the patient's vulva appears erythematous and a small amount of whitish discharge is present. There is no foul odor, bleeding, or abnormal lesions present. Which of the following is the most likely cause of the patient's presentation? Bacterial vulvovaginitis Foreign body Irritant vulvovaginitis Sexual abuse

Correct Answer ( C ) Explanation: Irritant or nonspecific vulvovaginitis is the most common cause of vulvar irritation in prepubertal girls. Patients may present with or without a thin, whitish, non-foul-smelling discharge. Vulvar irritation may occur as the result of poor hygiene, such as wiping from back to front or not wiping adequately after urination, or secondary to chemical irritants, such as bubble bath, perfumed soaps, or clothes washed in strong detergent. Tightly fitting clothing such as leotards, bathing suits, tights, spandex, and nylon underwear can also be irritants. Factors which may increase the overall risk of vulvovaginitis include obesity, a relatively alkaline pH, thin vaginal mucosa, and small labia minora. Vulvovaginitis can be both treated and prevented by adopting thorough hygiene measures, avoiding bubble baths, and keeping soap away from direct contact with the genital area. Wearing 100% cotton underwear, avoiding tight clothes, and quickly removing wet bathing suits should be advised. Bacterial vulvovaginitis (A) typically presents acutely with foul-smelling vaginal discharge associated with erythema, burning, itching, and soreness.

A 12-month-old boy presents with a hoarse, harsh cough and inspiratory stridor following two days of mild cough and nasal congestion. He is in mild respiratory distress. Lungs are clear to auscultation. What is the most likely diagnosis? Bacterial tracheitis Epiglottitis Laryngotracheitis Retropharyngeal abscess

Correct Answer ( C ) Explanation: Laryngotracheitis, also known as croup, is a common cause of inspiratory stridor in children. Croup is common amongst children ages 6 to 36 months and is most commonly caused by parainfluenza virus. The illness typically begins with one to two days of non-specific symptoms such as cough, nasal congestion, and low-grade fever. Children then may develop significant hoarseness, a barky cough, and inspiratory stridor. The stridor may occur only with activity or may occur at rest. A classic "Steeple sign" is present on x-ray, but x-rays are unnecessary for establishment of a diagnosis. The mainstay of treatment for croup is a glucocorticoid such as single-dose dexamethasone. Nebulized racemic epinephrine can be used for a rescue treatment for children with respiratory distress. The cough typically resolves within three days, but other symptoms may persist for a week. Deep neck space abscesses, such as a retropharyngeal abscess (D), often present with drooling and neck extension. Approximately one half of cases are associated with a preceding upper respiratory tract infection, but a barking cough is not characteristic. Stridor may develop as swelling progresses. In infants and children, the retropharyngeal space contains two chains of lymph nodes that drain the nasopharynx, adenoids, middle ear, and Eustachian tube. These lymph nodes may suppurate and eventually lead to a retropharyngeal phlegmon and then abscess. Lateral neck films show a widened prevertebral space.

3-week-old male is brought to your office because of a sudden onset of bilious vomiting of several hours duration. He is irritable and refuses to breastfeed, but stools have been normal. He was delivered at term after a normal pregnancy, and has no health problems to date. A physical examination shows a fussy child with a distended abdomen. Radiography of the abdomen shows a "double bubble" sign. Which one of the following is the most likely diagnosis? Infantile colic Intussusception Midgut volvulus Necrotizing enterocolitis

Correct Answer ( C ) Explanation: Midgut volvulus may present in one of three ways: as a sudden onset of bilious vomiting and abdominal pain in a neonate; as a history of "feeding problems" with bilious vomiting that appears to be a bowel obstruction; or less commonly, as failure to thrive with severe feeding intolerance. The classic finding on abdominal plain films is the "double bubble" sign, which shows a paucity of gas (airless abdomen) with two air bubbles, one in the stomach and one in the duodenum. However, the plain film can be entirely normal. The upper gastrointestinal contrast study is considered the gold standard for diagnosing volvulus.

A full-term neonate is brought to the ED for constant crying for the last three hours. In the ED, he sleeps quietly in his mother's arms. He cries when you examine him but is immediately consoled when he is swaddled and held. His exam is normal. What is the next most appropriate step in his management? ACT scan of the head to rule out intracranial hemorrhage BLumbar puncture to rule out serious bacterial infection CReassurance DSkeletal survey for abuse

Correct Answer ( C ) Explanation: Neonates cry in varying patterns throughout the day. Although crying is typically a sign of hunger or normal uncomfortable states (wet diaper, gas), it may also signal pain or underlying disease. An easily consoled infant without a source of crying after a thorough history and physical exam can be discharged with parental reassurance. A thorough exam for crying includes completely exposing the infant; palpating the fontanelles; fluorescein staining of the eyes for corneal abrasions; examining the fingers, toes, and penis for hair tourniquets; and checking for hernias. It is also important to look for retinal hemorrhages and palpate the extremities for accidental or deliberate trauma.

The typical flow of blood across an atrial septal defect occurs in which of the following fashions? Deoxygenated blood from left to right atria Deoxygenated blood from right to left anterior descending coronary arteries Oxygenated blood from left to right atria Oxygenated blood from right to left atria

Correct Answer ( C ) Explanation: One of the congenital cardiac abnormalities is atrial septal defect (ASD). This condition is characterized by a defect in the interatrial septum. This condition initially results in a left-to-right shunt as oxygenated blood from the higher-pressure left atrium passes into the right atrium. This congenital defect can go undiagnosed for 40-50 years, as it is typically asymptomatic. In fact, the majority of cases (70%) are diagnosed in the fifth decade. Over time, a large defect may lead to pulmonary overcirculation, pulmonary hypertension, Eisenmenger syndrome, right-to-left shunting, and cyanosis. When symptomatic, patients complain of fatigability, exertional dyspnea or recurrent respiratory infections. During childhood, many atrial septal defects will spontaneously close, however, ASDs that persist into adulthood usually require surgical closure. The classic finding on cardiac auscultation is a fixed, split S2.

A 10-year-old boy is seen in your clinic with his grandmother due to right ear pain. His grandmother informs you that the boy is part of the school's swimming team. Physical examination reveals tragal tenderness, palpable tender lymphadenopathy in the right periauricular region with associated swelling of the pinna and periauricular skin. Tympanic membrane cannot be visualized due to ear canal swelling. Which of the following etiologic agents is the most common cause of the patient's condition? Moraxella catarrhalis Nontypeable Haemophilus influenzae Pseudomonas aeruginosa Streptococcus pneumoniae

Correct Answer ( C ) Explanation: Otitis externa (or swimmer's ear, although it can occur without swimming) is caused most commonly by Pseudomonas aeruginosa. It results from chronic irritation and maceration from excessive moisture in the canal. The loss of protective cerumen may play a role, as may trauma, but cerumen impaction with trapping of water also can cause infections. The predominant symptom is acute onset of ear pain (otalgia), often severe, exacerbated by manipulation of the pinna, pressure on the tragus or jaw motion. The severity of the pain and tenderness may be disproportionate to the degree of inflammation because the skin of the external ear canal is tightly adhered to the underlying perichondrium and periosteum. Edema of the ear canal, erythema, and thick, lumpy otorrhea are prominent signs of the acute disease. The cerumen usually is white and soft in consistency, as opposed to its usual yellow color and firmer consistency. The canal often is so tender and swollen that the entire ear canal and tympanic membrane cannot be adequately visualized, and complete otoscopic examination may be delayed until the acute swelling subsides. Other physical findings may include palpable, tender lymph nodes in the periauricular region, and erythema and swelling of the pinna and periauricular skin.

An uncircumcised 2-year-old boy presents with penis pain. Examination reveals a swollen, nonreducible foreskin which compresses the penis, resulting in glans erythema and edema. Based on these physical examination findings, which of the following is the most likely diagnosis? Balanitis Hypospadia Paraphimosis Posthitis

Correct Answer ( C ) Explanation: Paraphimosis is foreskin that, once retracted, cannot be brought to usual position, resulting in constriction. If this persists for several hours, the continued retraction can cause a strangulation effect on the glans arterial blood supply, leading to ischemia and possible gangrene. As such, this is considered a urologic emergency. Causes include improper handling of the foreskin, as in repeated or prolonged retraction during cleaning, examination, catheterization or cystoscopy, infection and pathologic phimosis. Symptoms include inability to reduce the foreskin, foreskin erythema and edema, glans erythema and edema and pain. Paraphimosis can be prevented by returning the foreskin back to its flaccid position after retraction is no longer necessary. Treatment includes manual manipulation of the swollen foreskin back into its flaccid position, which may require the use of lubricants, cold compression or anesthetics. If unsuccessful, an emergent dorsal slit procedure or Dundee procedure may be tried.

A 12-year-old boy presents to the ED with swelling and pain in his penis. He reports that over the past 6 hours he has been unable to reduce his foreskin over the glans and that the area has become progressively more swollen. He denies dysuria, discharge, testicular pain, or any other symptoms prior to his foreskin being retracted earlier today. On exam, his foreskin cannot be reduced and the glans is red, swollen, and tender, as seen above. Which of the following is the most likely diagnosis? Balanoposthitis Orchitis Paraphimosis Phimosis

Correct Answer ( C ) Explanation: Paraphimosis is the inability to reduce the foreskin back to its anatomic position over the glans of the penis. It has several causes, including infection, trauma, hair tourniquets, and sexual activity. Distal venous congestion develops and can lead to arterial compromise, necrosis, and gangrene. It is a urologic emergency. You have to call the "paramedics" because "paraphimosis" is an emergency. This patient's absence of penile discharge, testicular symptoms, and sudden development of symptoms after not being able to reduce his foreskin strongly point to paraphimosis as the diagnosis. In addition to manual reduction, substances with a high solute concentration can be used to osmotically draw out fluid from the edematous glans and foreskin prior to manual reduction. Granulated sugar spread over the glans and foreskin for 2 hours has been shown to facilitate manual reduction. Balanoposthitis (A) is inflammation of the glans and foreskin, often caused by infection, trauma, and poor hygiene. Though it is more common in uncircumcised male patients, the foreskin should be reducible, unlike in this case

Which one of the following children should undergo testing for streptococcal pharyngitis? A 16-year-old with pharyngitis, rhinorrhea, and voice hoarseness A 2-year-old with fever and cough A 5-year-old with fever, cough, and tender anterior cervical lymphadenopathy A 9-year-old asymptomatic household contact whose brother was recently diagnosed with acute streptococcal pharyngitis

Correct Answer ( C ) Explanation: Pharyngitis has many etiologies including infectious (bacterial, viral, fungal), GERD, trauma, toxins, and malignancy. Bacterial pharyngitis is most commonly caused by group A strep (GAS). Testing and treatment for group A strep are based on the Centor criteria (1) Temperature >38°C; (2) Tender anterior cervical lymphadenopathy; (3) Absence of cough; (4) Presence of pharyngotonsillar exudates; (5) Age <15 years; (6) Age >45 years subtracts a point. Rapid antigen testing should be obtained in patients suspected of having bacterial pharyngitis who have 2-3 positive features of the Centor criteria. When an appropriate sample is obtained, the rapid antigen test has a sensitivity around 70% to 90%. However, a negative rapid strep test still requires a throat culture given the high false negatives. In patients who have 4 out of 4 elements of the Centor criteria, the positive predictive value for strep pharyngitis is 60% and the patient should be treated empirically and no rapid strep test obtained. For 2-3 Centor criteria, a rapid strep test should be sent and treatment initiated if positive. If negative, a confirmatory throat culture should be sent and treatment held until culture results are available. Centor criteria of 0-1 has a negative predictive value of 70%+ and no testing or treatment should be initiated. Treatment of strep pharyngitis remains to be penicillin. The patient in the above scenario has three Centor criteria (age < 15, fever, and tender anterior cervical lymphadenopathy) and should undergo rapid antigen testing for group A streptococcal pharyngitis.

An 8-year-old boy presents to your office with complaints of fever, headache and rash. His mother tells you that he recently returned from visiting relatives in North Carolina. Physical exam reveals a maculopapular rash on the patient's wrists and ankles that appears to be spreading to his trunk. His temperature is 102°F (38.9°C). Which of the following is the most appropriate therapy? Amoxicillin Cephalexin Doxycycline Penicillin

Correct Answer ( C ) Explanation: Rocky Mountain spotted fever (RMSF) is a tick-borne illness most commonly found in the Southeast, South Central and certain parts of the Northeastern United States. It is the most common cause of tick-borne fatalities in the United States. RMSF is often described as "the great imitator" of other diseases because of its diverse clinical presentation. The classic triad of headache, fever and a rash is seen in most patients by the second week of infection. Patients may also present with myalgias, central nervous system symptoms such as confusion and lethargy, or gastrointestinal symptoms such as nausea, vomiting, diarrhea or abdominal pain. RMSF is a clinical diagnosis made when exposure to ticks is determined. Doxycycline is the drug of choice to treat RMSF for both adults and children. Doxycycline is also used to treat ehrlichiosis, Lyme disease, and relapsing fever, which can all be confused with RMSF. In 1997, the American Academy of Pediatrics endorsed the use of doxycycline in children with RMSF because of the potential for fatalities. Short courses in children were not found to cause dental staining.

An eight-year-old boy presents to your office with complaints of fever, headache, and rash. His mother tells you that he recently returned from visiting relatives in North Carolina. Physical exam reveals a maculopapular rash on the patient's wrists and ankles that appears to be spreading to his trunk. His temperature is 102°F (38.9°C). Which of the following is the most appropriate therapy? Amoxicillin Cephalexin Doxycycline Penicillin

Correct Answer ( C ) Explanation: Rocky Mountain spotted fever (RMSF) is a tick-borne illness most commonly found in the Southeast, South Central, and certain parts of the Northeastern United States. It is the most common cause of tick-borne fatalities in the United States. RMSF is often described as "the great imitator" of other diseases because of its diverse clinical presentation. The classic triad of headache, fever, and a rash is seen in most patients by the second week of infection. Patients may also present with myalgias, central nervous system symptoms such as confusion and lethargy, or gastrointestinal symptoms such as nausea, vomiting, diarrhea, or abdominal pain. RMSF is a clinical diagnosis made when exposure to ticks is determined. Doxycycline is the drug of choice to treat RMSF for both adults and children. Doxycycline is also used to treat ehrlichiosis, Lyme disease, and relapsing fever, which can all be confused with RMSF. The American Academy of Pediatrics endorsed the use of doxycycline in children with RMSF because of the potential for fatalities. Short courses in children were not found to cause dental staining.

Which of the following is true regarding sudden unexpected death of infancy (SUDI)? Most cases occur in infants younger than two months old Normal-term infants have a higher incidence of Sudden Unexplained Death of Infancy (SUDI) than preterm infants Peak incidence is between two and four months of age The diagnosis is usually made in the emergency department

Correct Answer ( C ) Explanation: Some 95% of sudden unexpected death of infancy (SUDI) infants die before six to eight months, with a peak occurring between two and four months of age. SUDI may occur at any time during the first two years of life, but it is rare in children younger than one month of age and in those older than one year of age. African American, Native American, and Alaskan Native infants have rates 2- to 3-fold greater than the national average. Male babies and multiple births are also risk factors for SUDI.

A four-day-old female product of a normal full-term pregnancy, labor, and delivery presents to the emergency room with a one day history of 102.2°F fever, poor feeding, irritability, and fitful sleeping. Maternal and infant history is unremarkable with the exception that the mother had a urinary tract infection at the end of her second trimester, which was not cultured and low-grade fever after discharge from the hospital on day two after delivery. Physical examination of the neonate demonstrates pallor, mild jaundice, tachycardia, episodes of apnea, difficult arousal, and a bulging fontanel. Peripheral white count is 2000 per milliliter, mostly lymphocytes. Examination of cerebrospinal fluid demonstrated elevated protein, low glucose, very elevated white blood cell count (98 percent neutrophils), and a few red blood cells. Gram stain of the cerebrospinal fluid revealed Gram-positive cocci in chains. What is the diagnosis in this neonate? E. coli sepsis and meningitis Staphylococcus aureus sepsis meningitis Streptococcus agalactiae sepsis and meningitis Streptococcus pyogenes sepsis and meningitis

Correct Answer ( C ) Explanation: Streptococcus agalactiae (Group B Strep/GBS) is currently the major cause of perinatal bacterial infection in the United States. As is the usual case with E. coli and other bacteria that cause perinatal infections, the organism colonizes the birth canal and passes to the infant during labor and delivery. However, there is, less frequently, also hematogenous spread from the mother. Maternal manifestations of GBS include endometritis, chorioamnionitis, and urinary tract infections. Current recommendations to reduce the incidence of and mortality from GBS include screening with vaginal and rectal cultures at the end of the second trimester and between 35 and 37 weeks of gestation. Those women with positive cultures or a GBS urinary tract infection or GBS bacteria should receive intrapartum antimicrobial prophylaxis with intravenous ampicillin. E. coli (A), the second most common cause of perinatal infection, is even less likely because of the Gram-stain demonstrating Gram-positive organisms in chains. Currently, perinatal Staphylococcal (B) infection is a rare cause of perinatal sepsis and meningitis as is Streptococcus pyogenes (D) (Group A Strep/GAS). It should be remembered that GAS was a major cause of puperal fever and mortality prior to the discovery by Dr. Semmelweis in 1847 that hand washing between deliveries almost eliminated "childbed fever."

A 7-year-old boy with scrotal pain for the past 3 days presents to your office. Upon examination, there is a swollen left hemiscrotum with visible overlying erythema. There is a visible dark blue dot present in upper pole of the left testicle. What is the most likely diagnosis? Epidydmitis Testicular torsion Torsion of the appendix testis Varicocele

Correct Answer ( C ) Explanation: The appendix testis is a vestigium of the mullerian ductal system. Torsion of the appendix testis is the most common cause of testicular pain in boys 2-10-years-old. Torsion of the appendix testis is characterized by subacute pain with progressive inflammation of the testis and epididymis causing scrotal pain and erythema. During palpation, a 3-5 mm indurated mass may be felt in the upper pole. In some cases, the tissue that had undergone torsion is visible as a "blue dot" sign through the scrotal skin. Usually the inflammation resolves by 3-10 days and is treated with rest and NSAIDs.

An otherwise healthy 16-year-old boy is seen in clinic for a routine physical examination. He is a straight-A student and does not participate in sports. On exam you note that his height is greater than the ninety-fifth-percentile for age, he has long fingers, a pectus deformity, and auscultate a mid-systolic click followed by a 3/6 late systolic murmur. Which of the following is the most likely diagnosis? Ehlers Danlos syndrome Homocystinuria Marfan syndrome Russell Silver syndrome

Correct Answer ( C ) Explanation: The boy has a constellation of findings that are most suspicious for Marfan syndrome. Marfan syndrome is an autosomal dominant disorder caused by the abnormal biosynthesis of fibrillin-1. It is a multisystem disorder with cardinal manifestations in the skeletal, cardiovascular and ocular systems. The skeletal deformities include disproportionate overgrowth of long bones, anterior chest deformity (pectus carinatum or pectus excavatum), reduced upper to lower segment ratio, joint hypermobility, and arachnodactyly. Common cardiovascular manifestations are prolapse of the mitral or tricuspid valves. Individuals are at increased risk for an aortic aneurysm or dissection or both. Aortic abnormalities are the most life-threatening which require lifelong monitoring by echocardiography. Ectopia lentis or dislocation of the ocular lens is the most common ocular abnormality that is present in 60-70% of patients. Diagnosis of Marfan syndrome is based on clinical criteria and molecular analyses. Management involves a multidisciplinary team due to the complexity of the condition. Ehlers Danlos syndrome (A) has many similarities with Marfan syndrome which includes joint hypermobility, mitral valve prolapse and aortic dissection. The findings that differentiate it are skin laxity, contractures, skin nodules, and easy bruising

A 12-year-old boy has a cognitive disability and multiple café-au-lait macules. Which additional finding is most likely on examination? Annular, hypopigmented macules Facial angiofibromas Neurofibromas Periungal fibromas

Correct Answer ( C ) Explanation: The boy has both café-au-lait macules and cognitive disability, raising concern for neurofibromatosis type 1, or von Recklinghausen's disease. Neurofibromatosis is a neurocutaneous disorder caused by a mutation on NF1 on chromosome 17. Cutaneous findings of neurofibromatosis type 1 include café-au-lait macules, axillary and inguinal freckling, Lisch nodules, and neurofibromas. Neurofibromas are peripheral nerve sheath tumors that may be focal and cutaneous or plexiform. Cutaneous neurofibromas are small, sessile or pedunculated lesions that move with the skin. They are non-tender and occur primarily on the trunk. Cutaneous neurofibromas most commonly develop during adolescence, although subtle lesions may be noted earlier in angled light. The number of neurofibromas on an affected individual may range from a few to thousands. Plexiform neurofibromas are present in half of patients with neurofibromatosis and cause a very different clinical picture. They may be superficial, deep, or both. The entangled nerves can become large, complex, and often disfiguring due to associated overgrowth of surrounding tissues. Deep plexiform neurofibromas may compress internal structures, including the airway and spinal cord. Individuals with neurofibromatosis are also at increased risk of bony abnormalities, including bony dysplasia, pseudoarthroses, scoliosis, osteoporosis, and short stature. Additionally, affected individuals are predisposed to a variety of tumors, including optic nerve gliomas and soft tissue sarcomas such as rhabdomyosarcoma, malignant peripheral nerve sheath tumors, and glomus tumors.

A previously healthy 14-year-old boy presents to clinic with intermittent yellowing of the eyes. His mother is concerned because she notices this each time he has returned home from football practice this summer and wonders if he should quit the team. The boy experiences no symptoms during the episodes and was unaware of the yellowing of his eyes until his mother mentioned it. His exam is currently unremarkable. A liver panel is significant for an unconjugated bilirubin of 3 mg/dL. A CBC, peripheral smear, and reticulocyte count are unrevealing. The remainder of the panel is normal. What is the next best step in management? Obtain a hepatitis antibody panel Referral to hematology clinic Supportive care and reassurance Suspension from the football team until gastroenterology referral is completed

Correct Answer ( C ) Explanation: The child has a clinical course consistent with Gilbert syndrome. In Gilbert syndrome, the activity of uridine diphosphate glucuronosyltransferase (UGT) is decreased compared to that of the general population due to a genetic mutation. The result is a decreased rate of conjugation of serum bilirubin. Clinical manifestations are typically not apparent unless the affected individual is in a state of physical exertion, fasting, illness, stress, hemolysis, or menstruation. In those situations, unconjugated bilirubin rises. The only sign of the hyperbilirubinemia may be scleral icterus, as typically bilirubin levels do not rise enough to cause significant jaundice. As in this patient's case, the signs of Gilbert syndrome often do not appear until adolescence, when sex hormones alter bilirubin metabolism. In the setting of suspected Gilbert syndrome, alternative sources of bilirubin production should be evaluated. A CBC, reticulocyte count, and peripheral smear are appropriate to assess for hemolysis. A liver panel should be obtained to assess for additional hepatic abnormalities such as elevated serum transaminases and alkaline phosphatase. If hemolysis and liver labs are normal, a presumptive diagnosis of Gilbert syndrome can be made. A definitive diagnosis requires serial testing over the following year. The mild rise of bilirubin in Gilbert syndrome has no long term adverse effects. No specific treatment is required, and the patient need not quit the football team. Supportive care and reassurance are all that are necessary.

A five-year-old child is brought to clinic with scalp itching. Adherent white flecks are noted attached to hair proximal to the scalp that cannot be dislodged. What is the treatment of choice? Ketoconazole shampoo Oral ivermectin Topical 1% permethrin Topical lindane

Correct Answer ( C ) Explanation: The child has classic findings of pediculosis capitis, also known as head lice, including nits and intense itching. Pediculosis is caused by infestation with Pediculus humanus capitis, the human head louse. The lice are spread by direct contact or through fomites and can be difficult to eradicate. The mature female louse lays eggs, known as nits, which are strongly adherent to the hair shaft. The treatment of choice for pediculosis capitis is a topical insecticide. An example is topical 1% permethrin, which is applied to saturate wet, freshly shampooed hair for ten minutes prior to rinsing. Treatment should be repeated in seven to 10 days. Although 5% permethrin is also available by prescription, it has not been shown to be more efficacious than 1% permethrin. Other topical treatment options include malathion, benzyl alcohol, spinosad, and ivermectin. Topical lindane (D) is an alternative topical insecticide. However, due to rare reports of neurotoxicity, it is not recommended as first-line therapy. Oral ivermectin (B) is also an effective treatment for pediculosis, but it is recommended only for children who have failed topical therapy. Ketoconazole shampoo (A) is used for the treatment seborrheic dermatitis of the scalp, but it is not an effective treatment for pediculosis. Notably, seborrheic dermatitis may result in hair casts, which are whitish, desquamated skin. Although hair casts may initially appear like nits, they can be distinguished by their ability to slide easily along hair shafts.

A three-year-old girl presents with headache, fatigue, anorexia, and myalgias after returning from a trip to the Dominican Republic. You notice multiple mosquito bites on physical examination. You suspect dengue fever. What is the treatment of choice for this diagnosis? Acyclovir Ceftriaxone Ciprofloxacin Supportive care

Correct Answer ( D ) Explanation: Dengue fever is an RNA virus transmitted to humans by infected mosquitos in the tropical regions of Southeast Asia, Mexico, Central America, Africa, and the Caribbean. Children younger than 15 years old have increased severity and mortality. Associated morbidity and mortality is secondary to hemorrhage from thrombocytopenia, platelet dysfunction, or disseminated intravascular coagulation. Symptoms include abrupt onset of high fever, retro-orbital pain, headache, myalgias, arthralgias, mucosal bleeding, and petechiae. Associated fatigue, nausea, vomiting, and abdominal pain are common. Laboratory testing commonly reveals thrombocytopenia, leukopenia, elevated liver transaminases, and elevated hematocrit. Supportive care is the treatment of choice. Confirmatory testing is rarely obtained but can be obtained by sending serum for reverse transcriptase-PCR of dengue virus or ELISA dengue IgM.

A 2-year-old boy is being evaluated for dysphagia and difficulty breathing. His mother tells you his symptoms suddenly began about 3-hours ago. His immunizations are up to date. He appears restless, irritable, and is drooling. His temperature is 40.0°C (104°F). Physical examination shows suprasternal and subcostal retractions. Auscultation of the lungs reveals inspiratory stridor. Blood cultures are pending. Which of the following is the most appropriate treatment for this patient's condition? Albuterol Ampicillin Ceftriaxone + vancomycin Prednisone

Correct Answer ( C ) Explanation: The constellation of symptoms most likely indicates acute epiglottitis. Epiglottitis is the inflammation of the epiglottis and adjacent supraglottic structures and results from bacteremia and/or direct invasion of the epithelial layer by the pathogenic organism. Although, Haemophilus influenzae type b (Hib) is the most common infectious cause of epiglottitis in children, empiric antimicrobial therapy should be directed towards penicillin-resistant Streptococcus pneumonia, Beta-hemolytic streptococci, and Staphylococcus aureus, including community-acquired methicillin-resistant S. aureus (MRSA) strains. The most effective therapy is a combination of a third-generation cephalosporin (eg, ceftriaxone or cefotaxime) and an antistaphylococcal agent active against MRSA (eg, clindamycin, vancomycin). Due to airway compromise, patients with suspected epiglottitis should have their airway emergently stabilized in the operating room. This should occur concomitantly or prior to antibiotic administration.

A two-month-old infant presents with two weeks of nasal congestion and cough. Over the past two days, the cough has acutely worsened. Her family describes discrete episodes of coughing and gagging, sometimes followed by emesis. She appears well between coughing episodes. What treatment is indicated? Honey Intramuscular ceftriaxone Oral azithromycin Supportive care

Correct Answer ( C ) Explanation: The infant has a presentation consistent with Bordetella pertussis infection. Pertussis is spread through aerosolized droplets. After a seven- to 14-day incubation period, symptoms begin with nasal congestion, cough, and low-grade fever. This period, known as the catarrhal phase, lasts one to two weeks. Subsequently, the infant progresses to the paroxysmal phase, which includes episodic, "rapid fire," repetitive coughing. The coughing may include cyanosis, salivation, and lacrimation. The cough is classically followed by an inspiratory "whoop." The whoop is most common in infants older than six months of age. The whoop is uncommon among older children and adolescents. The intensity of coughing may also result in post-tussive emesis. The paroxysmal stage typically lasts for four weeks. The child will then progress to the convalescent phase, during which he or she will gradually have decreasing frequency and severity of coughing episodes. The convalescent stage may last for a few weeks to several months. Children with pertussis remain contagious until B. pertussis is eradicated from their nasopharynx. Spontaneous clearance typically occurs in two to four weeks. Oral azithromycin is the drug of choice for rapidly eradicating B. pertussis from the nasopharynx. Its dosing is 10 mg/kg for five days, which is notably different from recommended dosing for community-acquired pneumonia. Treatment will not result in symptom resolution, but the child will no longer be contagious.

A 4-month-old female presents with a diaper rash following four days of loose stools. The rash consists of erythematous, raised plaques and papules over the labia, inguinal folds, perineum and buttocks. What treatment is indicated? 1% hydrocortisone ointment Mupirocin ointment Nystatin ointment Zinc oxide ointment

Correct Answer ( C ) Explanation: The infant has classic findings of candidal diaper dermatitis, including beefy red plaques and satellite lesions. In contrast to the rash of irritant diaper dermatitis, the inguinal folds are usually involved. Candidal diaper dermatitis results from fungal invasion of superficial microtears in the skin. Candidal diaper dermatitis is usually clinical apparent, but potassium hydroxide preparation of skin scrapings may be used to confirm a diagnosis in unclear cases. The treatment of choice for candidal diaper dermatitis is a topical antifungal such as nystatin ointment. Other topical antifungal options include azoles such as clotrimazole, miconazole, or ketoconazole. Antifungal ointment should be applied at least two to three times per day and continued until the rash has fully resolved for 48 hours. zinc oxide (D) ointment is a barrier cream used to treat irritant diaper dermatitis but is ineffective in the treatment of candida.

A 5-week-old infant suffers from dyspnea and fluid overload, but not cyanosis. Auscultation reveals a loud, holosystolic murmur at the left sternal border. You suspect a congenital cardiac defect. Echocardiography would most likely show which of the following abnormalities? Aorta is connected to the right ventricle Pulmonary artery is connected to the left ventricle Ventricular septal defect in the membranous portion Ventricular septal defect in the muscular portion

Correct Answer ( C ) Explanation: The most common congenital heart defect is a ventricular septal defect (VSD), one of the acyanotic congenital heart diseases. Defects occur in the ventricular septum, the "wall" between the right and left ventricle. This septum is comprised of an inferior muscular part and a superior membranous part. The majority of VSDs occur in the membranous part, near the atrioventricular node. A pansystolic murmur is nearly pathognomonic for VSD. Occurrence and severity of clinical manifestations depend on the size of the defect. Spontaneous closure occurs in the first six-months of life in up to 40% of affected infants. If present, symptoms may be mild or severe, but usually present as congestive heart failure due to a significant left-to-right intracardiac shunt with resultant pulmonary hypertension. Pharmacotherapy is recommended in these patients. However, surgical correction is typically necessary if the patient fails medical management or shows evidence of pulmonary hypertension (elevated pulmonary artery pressure).

A term infant presents to clinic for hospital follow up on the fifth day of life. His mother reports an uneventful pregnancy with normal labs. Her blood type is B+ but she is unsure of the infant's blood type. Delivery was by uncomplicated cesarean section for prior cesarean section. Total serum bilirubin was 8 mg/dL prior to hospital discharge on the third day of life. He is feeding a combination of breastmilk and formula every two to three hours. He has had six wet diapers in the past 24 hours and has a greenish yellow stool after each feed. Examination is notable for jaundice to the lower chest. He appears well hydrated. His weight is currently decreased 3% from birth weight. A total serum bilirubin level is 12 mg/dL. What is the most likely etiology of the neonate's jaundice? Breast milk jaundice Breastfeeding jaundice Isoimmune hemolytic disease Physiologic jaundice

Correct Answer ( D ) Explanation: During the first week of life, nearly all neonates develop some degree of jaundice due to unconjugated hyperbilirubinemia. Bilirubin is a product of heme catabolism and its primary source is red blood cells. Neonates have both an increased concentration of red blood cells, as well as a shortened red blood cell lifespan. In addition, enzymes of bilirubin conjugation are less active in neonates, and enterohepatic recirculation of bilirubin is higher. All of these immature processes are normal and they result in physiologic jaundice. Physiologic jaundice resolves without intervention as the infant's red blood cells, liver enzymes, and stooling patterns mature. Other pathologic processes may augment the mechanisms of physiologic jaundice, including increased red blood cell hemolysis, abnormalities of the hepatic conjugation system, other causes of abnormal stooling, and dehydration. The infant described above was born at term, is feeding and voiding normally, and appears well hydrated. His weight loss of 3% from his birth weight is acceptable on the fifth day of life. His mother's Rh positivity makes immune-mediated hemolysis unlikely, as Rh incompatibility is by far the most common cause of isoimmune hemolysis. Major blood group incompatibilities and minor antigen incompatibilities rarely result in hemolysis. Moreover, the neonate's total serum bilirubin is well below phototherapy level as indicated by standard phototherapy curves. No intervention is needed for this infant. His mother should be encouraged to continue breastfeeding and to monitor for signs and symptoms of dehydration.

9-year-old boy presents to the ED with bilateral knee pain, low-grade fever, nausea, vomiting, and diarrhea for the past 4 days. His vital signs are blood pressure of 116/80 mm Hg, heart rate of 98 beats per minute, respiratory rate of 14 breaths per minute, and a temperature of 38.1°C. On examination, you note the rash seen above. Urinalysis is positive for hematuria. Which of the following statements is the most accurate? Despite plasma exchange, most patients progress to chronic renal impairment Long-term prednisone therapy improves 5-year survival to greater than 50% The disease is self-limited; most cases resolve within 6-8 weeks Without treatment, the disease carries a mortality rate of 80% at one year

Correct Answer ( C ) Explanation: The patient has Henoch-Schönlein purpura (HSP). This small-vessel vasculitis predominantly occurs in small children. Most cases follow an upper respiratory tract infection. HSP classically presents with fever, abdominal pain, arthritis, hematuria, and a pathognomonic round, palpable, symmetrical rash that appears on the dependent areas of the legs and buttocks. NSAIDs, dapsone, and prednisone have all been shown to relieve symptoms. The course of disease is typically self-limited. Most cases resolve within 6 to 8 weeks, with a recurrence rate of up to 33%. Plasma exchange (A) has been found to be successful in the treatment of microscopic polyangiitis. In HSP, most patients do not progress to chronic renal impairment. Prednisone (B) therapy has increased the 5-year survival rate to greater than 50% in Churg-Strauss Syndrome, which is typically associated with fever, weight loss, malaise, and pulmonary symptoms, but it does not affect survival in HSP. Granulomatosis with polyangiitis (D), not HSP, once carried a 1-year mortality rate of 80%, however, the combination of cyclophosphamide and corticosteroids has been successful in inducing remission in more than 90% of patients.

12-year-old girl presents with right hip pain after kicking a ball to score at her soccer game. She describes hearing a "pop" followed by immediate pain. The patient can limp and localizes pain to the anterior right hip. An X-ray is obtained as seen above. Which of the following is the most appropriate management? Immobilization with a spica cast Muscle relaxant and heat Non-weight-bearing and pain control Surgical pinning

Correct Answer ( C ) Explanation: The patient has a pelvic avulsion fracture of the anterior inferior iliac spine (AIIS). This is the result of forced muscle contraction, causing a fracture at the muscular origin. The commonly fractured pelvic apophysis locations in descending order of incidence include ischial, anterior inferior iliac spine, anterior superior iliac spine, superior pubic ramus, and iliac crest. Risk factors include sports such as gymnastics, soccer, and hurdles. Prevention is through stretching of the commonly involved muscles including the sartorius, rectus femoris, hamstring, iliopsosas, and leg adductor muscles prior to participating in sports. Diagnosis is with plain radiographs of the pelvis in anterior posterior and frog leg views. Pelvic avulsions rarely require pinning and most often successfully heal with crutch use and pain control. Occasionally, this type of injury heals with excessive callus formation. Sport activity should be limited for four to six weeks.

A two-month-old boy is sent to the emergency department by his pediatrician for cough and an abnormal CBC with lymphocytosis. He is up-to-date with immunizations. His older brother, who is four years old, is not up-to-date since the pediatrician suspended his immunizations due to a developing neurologic condition. The brother has also had a febrile illness and has been coughing for more than three weeks. You observe the two-month-old coughing and see a period of peri-oral cyanosis. What method of confirmatory testing has the best combined sensitivity and specificity for diagnosis? Blood cultures Direct fluorescent antibody on nasal swabs Polymerase chain reaction of nasopharyngeal secretions Sputum cultures

Correct Answer ( C ) Explanation: The patient has pertussis. Pertussis is a highly contagious respiratory infection caused by Bordetella pertussis. Pertussis is commonly called whooping cough, secondary to the sound that is made on inspiration immediately following a paroxysm of coughing. There has been a recent resurgence of pertussis in the United States. This resulted in a revision of the booster schedule to require patients older than 10 years to obtain Tdap rather than tetanus (Td) alone. Transmission is via respiratory droplets and has an incubation of seven to 10 days before symptoms begin. Pertussis is a three-phase illness with each phase lasting approximately one to two weeks. First, there is the catarrhal phase with symptoms similar to an upper respiratory infection. Second, is the paroxysmal phase. This includes harsh coughing paroxysms lasting minutes. Finally, the third phase is the convalescent phase. This last phase demonstrates clinical recovery but, unfortunately, the patient remains infectious. Symptoms include persistent cough, posttussive emesis, apnea, cyanosis, fever, and inspiratory whoop. Lab work may reveal an absolute lymphocytosis. The most sensitive and specific testing is via polymerase chain reaction (PCR) of nasopharyngeal secretions.

A mother brings to you a 10-year-old girl because of poor school performance. The girl used to be an excellent student, but now she has difficulty understanding math and science concepts. The mother also complains that she moves sluggishly when given chores. The patient also complains of occasional constipation. On examination, there is dry skin, brittle hair, and enlarged and firm neck mass. Which of the following is the laboratory abnormality that would be expected in this condition? Absence of anti-thyroglobulin antibodies Increased radioactive iodine uptake Presence of anti-thyroid peroxidase antibodies Presence of thyroid-stimulating immunoglobulin

Correct Answer ( C ) Explanation: The patient manifests with signs and symptoms of Hashimoto's thyroiditis or chronic autoimmune thyroiditis. It is characterized by the gradual thyroid failure, with or without goiter formation, due to autoimmune-mediated destruction of the thyroid gland. Expected laboratory results include low T4 with an elevated TSH, positive anti-thyroid peroxidase antibodies, and positive anti-thyroglobulin antibodies. Hashimoto's thyroiditis is the most common cause of hypothyroidism in children resulting in either goiter or thyroid atrophy. Common manifestations are declining growth velocity resulting in short stature, altered school performance, sluggishness, lethargy, cold intolerance, constipation, dry skin, brittle hair, facial puffiness, and muscle aches and pains. Levothyroxine is the treatment of choice in children with hypothyroidism with the goal to restore normal growth and development. Absence of anti-thyroglobulin antibodies (A) is not correct. In Hashimoto's thyroiditis, there is the presence of anti-thyroglobulin antibodies. Increased radioactive iodine uptake (B) and the presence of thyroid stimulating immunoglobulin (D) are wrong because these findings are expected in Graves' disease where there is hyperthyroidism.

A 12-year-old boy presents with penile pain. On exam you note the image above. What management is indicated? Direct application of ice to the skin Follow up with urology Manual reduction Topical clotrimazole

Correct Answer ( C ) Explanation: The patient presents with paraphimosis, a urologic emergency, requiring emergent reduction of the foreskin. Paraphimosis occurs when the foreskin cannot be returned to its anatomic position covering the glans penis. It can be caused by infection, trauma or hair tourniquets. Subsequent venous congestion can lead to arterial compression, penile necrosis and gangrene. The typical presentation is of erythema and engorgement of the penis distal to the obstruction and proximal flaccidity. Treatment is directed at reducing swelling and edema of the foreskin and reducing it to the neutral position. Parenteral anesthesia or a local nerve block may be necessary for manual reduction. Circumferential compression should be held starting at the glans to reduce edema. This should be followed by manual reduction by placing steady pressure on the glans with both thumbs while holding the shaft straight. If these attempts fail, urology should be consulted for dorsal slit procedure. Patients may be discharged home after reduction as long as they are voiding without difficulty and have no signs of severe infection.

A mother brings her 22-month-old child to the ED after noticing bloody bowel movements. The child has a normal birth history. Mom denies any pain associated with these episodes, but she states that the child has become increasingly pale with decreased energy. His vital signs include a blood pressure of 95/60 mm Hg, heart rate of 140 beats per minute, respiratory rate of 24 breaths per minute, and oxygen saturation of 98% on room air. Abdominal examination is unremarkable, but his stool is guaiac positive. Which of the following is most likely to confirm the diagnosis? Abdominal ultrasound Air contrast enema Technetium-99m pertechnetate scintiscan Upper GI series via nasogastric tube

Correct Answer ( C ) Explanation: The patient's presentation is classic for Meckel's diverticulum, the most common congenital malformation of the GI tract, and follows the rule of 2s: the diverticulum is 2 cm wide and 2 inches long and usually located within 2 feet of the ileocecal valve. The condition occurs in 2% of the population, and only 2% of affected patients ever become symptomatic. Many contain heterotropic tissue, the most common of which are gastric mucosa and pancreatic acini. The classic presentation is painless rectal bleeding in a boy younger than five years. Diagnosis is made by radionuclide scanning (technetium-99m pertechnetate scintiscan), also called a Meckel's scan. The test is highly specific.

Which of these children with hematuria needs admission to the hospital? A 10-year-old boy with recent sore throat treated with amoxicillin, tea-colored urine, and normal urine output; UA shows microscopic hematuria and proteinuria; blood tests are normal A 12-year-old girl with lower abdominal pain, dysuria, urgency, frequency, and pink urine; UA shows microscopic hematuria, > 50 WBCs/hpf, + leukocyte esterase, + nitrites; urine pregnancy test is negative A 6-year-old boy with nausea, vomiting, bloody diarrhea, and decreased urine output; UA shows microscopic hematuria and hyaline casts; blood tests show a high WBC count, low hemoglobin, and low platelets An 8-year-old girl with diffuse mild edema, tea-colored urine, and normal urine output; UA shows microscopic hematuria and proteinuria; blood tests show a normal WBC count, hemoglobin, and platelets but low protein

Correct Answer ( C ) Explanation: This boy has hemolytic-uremic syndrome (HUS), a disorder that causes a triad of acute renal failure, thrombocytopenia, and microangiopathic hemolytic anemia. It most often results from antibiotic treatment of gastroenteritis in individuals with Escherichia coli O157:H7 infection as the dying organism releases a highly virulent, Shiga-like toxin (SLT). Symptoms of HUS are more than simple E. coli enteritis and include decreased urine output, irritability, lethargy, seizures, or encephalopathy. Unlike disseminated intravascular coagulopathy (DIC), the PT, aPTT, and fibrinogen are normal. ED management is supportive. All patients with HUS should be hospitalized. Poststreptococcal glomerulonephritis (A) results from immune complex deposits in the glomeruli after acute streptococcal infection. Asymptomatic patients with a normal blood pressure may be discharged in consultation with a pediatric nephrologist. Older children with simple cystitis (B) can be safely managed as an outpatient with oral antibiotics. Nephrotic-nephritic syndrome (D) is caused by increased permeability of the glomerular capillary wall leading to renal protein loss and its associated consequences. The hallmarks are proteinuria, hypoproteinemia, generalized edema, and hypercholesterolemia. Patients with nephrotic syndrome with only mild or moderate edema without respiratory symptoms, signs of infection, or thrombotic complications may be discharged home in consultation with a pediatric nephrologist. The treatment of nephrotic syndrome is oral corticosteroids.

A six-week old infant presents to the ED with his mother who states he has not been feeding well. She notes that he grunts with feeding and that his fingers seem to turn blue. On physical exam, you observe cyanosis under the tongue and in the distal extremities. A chest radiograph is seen above. Which of the following is the likely cause of this infant's presentation? Coarctation of the aorta Patent ductus arteriosus Tetralogy of Fallot Ventricular septal defect

Correct Answer ( C ) Explanation: This child has evidence of congenital heart disease and more specifically tetralogy of Fallot. Congenital heart disease is classified as either cyanotic or acyanotic. Cyanotic congenital heart disease is secondary to a right to left shunt. Tetralogy of Fallot is a cyanotic congenital heart disease that is defined by four key findings: pulmonic stenosis with right ventricular outflow obstruction, right ventricular hypertrophy, a ventricular septal defect, and an overriding aorta. Tetralogy of Fallot is the most common cyanotic heart defect. It may present up to three to four months after birth. Signs and symptoms of tetralogy of Fallot include cyanosis, poor feeding, hemoptysis, squatting to relieve symptoms, a single S2 on cardiac auscultation, and scoliosis. Patients with unrepaired tetralogy of Fallot may experience "tet spells." These episodes typically occur during periods of stress that result in crying and breath-holding. They are defined by acute respiratory distress because of an increase in right ventricular outflow tract obstruction. The pulmonic stenosis murmur becomes weaker at these times due to increased shunting through the ventricular septal defect. Diagnosis of tetralogy of Fallot is with echocardiogram. An ECG may reveal right ventricular hypertrophy or right axis deviation. A chest radiograph will often show a boot-shaped heart due to right ventricular hypertrophy. Management of a tet spell is to decrease pulmonary resistance and increase systemic resistance to blood flow. First, you should place the child in the knees-to-chest position. Supplemental oxygen and morphine should be given. Phenylephrine may be used to increase systemic vascular resistance. Surgical therapy is the mainstay of long-term treatment for these patients.

A 7-year-old boy presents for a well child check. His mother reports that he has always been a healthy child and does well in school. On examination, he is noted to have pectus excavatum and long, tapered fingers. He is wearing glasses, and mother reports a history of upward lens subluxation. What is the most likely genetic abnormality? Ehlers Danlos Syndrome Homocystinuria Marfan Syndrome Osteogenesis Imperfecta

Correct Answer ( C ) Explanation: This child has findings concerning for undiagnosed Marfan Syndrome. Marfan Syndrome is an autosomal dominant connective tissue disorder with a broad range of clinical severity. The condition is caused by a mutation in fibrillin-1 (FBN1) gene. Features of Marfan syndrome include aortic root dilation or dissection, mitral valve prolapse, excessive linear growth of long bones, joint laxity, arachnodactyly, pectus deformities, scoliosis, kyphosis, hindfoot valgus, decreased upper segment to lower segment ratio, increased arm span to height ratio, ectopia lentis, emphysematous pulmonary changes, spontaneous pneumothorax, skin striae, and dural ectasia. The revised Ghent Criteria are used to establish a diagnosis and include combinations of aortic disease, ectopia lentis, FBN1 mutations, family history, and a systemic score. The systemic score includes items such as pectus carinatum, hindfoot deformities, pneumothorax, scoliosis, and facial features. Lens subluxation is typically upward in Marfan Syndrome. In the case of the above patient, pectus excavatum and arachnodactyly should raise concern for Marfan Syndrome rather than Ehlers Danlos Syndrome.

A 3-week-old baby girl with an unknown birth history presents to the emergency department with 3 days of poor feeding and recurrent cyanotic episodes. On exam, she is agitated, tachypneic, tachycardic with central cyanosis and a harsh systolic crescendo decrescendo murmur along the left mid-to-upper sternal border. An arterial oxygen saturation is 78% and a chest X-ray shows a boot-shaped heart. What is the best initial step in management? Administer morphine 0.2 mg/kg IM Administer propanolol 0.01 mg/kg IV Console the child in a knee-chest position Start positive pressure ventilation

Correct Answer ( C ) Explanation: This clinical scenario describes a hypercyanotic episode, also known as a 'tet spell' that can occur in children with Tetrology of Fallot (TOF). The four associated abnormalities in TOF are right ventricular hypertrophy, right ventricular outflow tract obstruction, ventricular septal defect and an over-riding aorta. Classic chest X-ray findings of TOF include a normal sized but boot-shaped heart reflecting the right ventricle with decreased pulmonary vascularity. TOF anomalies can lead to decreased pulmonary flow and right-to-left shunting through the VSD. A 'tet spell' can occur with decreased systemic vascular resistance that can result from hypotension, exertion during feeding, crying, defecation and tachycardia. This leads to increased right-to-left shunting of the blood across the VSD. The cyanotic episode is usually 15 to 30 minutes long but can lead to stroke, seizures and even death. During a spell, patients can have a right ventricular outflow tract obstruction murmur (harsh systolic crescendo decrescendo murmur along the mid-to-upper sternal border) and a VSD murmur (harsh pansystolic murmur heard best at the lower left sternal border). Management of spells aims to increase systemic vascular resistance, correct hypoxia and correct acidemia. Initial steps include administering high-flow oxygen by mask, consoling the child by cradling them in a knee-chest position, and avoiding exacerbating distress. Underlying arrhythmia, hypothermia, and hypoglycemia should also be corrected.

A 13-year-old boy presents to the Emergency Department with thigh pain. His symptoms worsened after jumping into a swimming pool. On physical exam, he is a well-developed, obese boy who ambulates with a limp. The hip is externally rotated and adducted. X-rays of the pelvis are taken and shown above. What is the most likely diagnosis? Congenital hip dislocation Legg-Calvé-Perthes disease Slipped capital femoral epiphysis Transient synovitis

Correct Answer ( C ) Explanation: This patient presents with slipped capital femoral epiphysis (SCFE). SCFE is characterized by displacement of the capital femoral epiphysis from the femoral neck through the growth plate. It affects predominantly obese adolescents. Males and African-American children are more commonly affected. Patients present with dull, aching hip, groin, thigh or knee pain, typically without any significant preceding trauma. Physical exam reveals external rotation and adduction at the hip. The diagnosis is confirmed by AP and lateral X-rays. The slipped epiphysis resembles "ice cream falling off the cone" and an abnormal Klein line. A straight line drawn along the lateral aspect of the femoral neck normally intersects the femoral head, whereas this line passes outside of the epiphysis in patients with SCFE. Orthopedic evaluation is essential, as treatment is almost always operative. The most feared complication of SCFE is osteonecrosis of the femoral head. Congenital hip dislocation (A) typically affects Caucasian firstborn females < 1 year of age. Legg-Calvé-Perthes (B) is an idiopathic avascular necrosis of the femoral head in school-aged children. Transient synovitis (D) presents with hip, thigh or knee pain after a recent upper respiratory infection.

A 3-week-old boy presents with two days of non-bilious projectile vomiting. Examination reveals a mass in the infant's right upper quadrant. On a barium upper GI series report, the radiologist states a "string sign" is present. Which of the following is this infant at greatest risk of developing? Hyperchloremia Hyperkalemia Hypokalemia Hyponatremia

Correct Answer ( C ) Explanation: This patient presents with the classic description of congenital pyloric stenosis. Hypertrophic pyloric stenosis describes the narrowing of the pylorus due to congenital or functional hypertrophy of the surrounding muscle. It causes severe projectile non-bilious vomiting and occurs most commonly in male infants of age 2-8 weeks. The hypertrophy is classically described as a palpable olive-shaped mass in the middle upper or right upper quadrant. This condition also occurs in adults, but it is due to a sclerosing stenosis secondary to chronic peptic ulceration. Ultrasound is used to confirm the diagnosis. A barium upper GI series may also show a thin stream of contrast emanating from the pylorus ("string sign" or "railroad track sign"). Any infant with significant vomiting is at risk for metabolic alkalosis due to excessive loss of gastric acid. In an attempt to compensate, cells exchange extracellular potassium for intracellular hydrogen ions, and as such, total body potassium gets shuttled intracellularly, extracellular potassium stores become depleted and hypokalemia ensues. Treatment includes fluid replacement and electrolyte correction. Intravenous and oral atropine can also be used, however, surgery is usually the necessary definitive treatment.

An otherwise healthy 4-year-old boy presents with his parents for vomiting and diarrhea. Several kids at his school have had similar symptoms. He began to have nonbloody, nonbilious vomiting yesterday and this morning began to have nonbloody diarrhea. He felt warm at home, but he is currently afebrile. He is well-appearing and playful. He is able to drink water without difficulty. Which of the following is the next best step? Administer intravenous fluids Admit for observation Provide reassurance and discharge home Send stool cultures

Correct Answer ( C ) Explanation: Viral gastroenteritis is a common illness in children. Globally, it is one of leading causes of pediatric deaths due to dehydration. Transmission is via the fecal-oral route. Once infected, patients will often have non-bloody, non-bilious vomiting and then develop watery diarrhea. The patient may have fever, and may have some abdominal pain. The duration of illness is typically less than 7 days. Dehydration is the biggest concern. Children who can successfully take oral liquids and appear well are safe for discharge home. The parents should be provided with reassurance, given instructions for oral hydration and red flag symptoms. Some children require pharmacologic control of their nausea. Oral ondansetron is the first line anti-emetic for gastroenteritis.

A 15-year-old boy is sent to you by the athletic trainer of the local football team after suffering his 3rd concussion of the season. Based on current guidelines, your advice pertaining to this patient returning to play should be which of the following? AHe may return to play if asymptomatic after 2 weeks BHe may return to play if asymptomatic after 30 days CHe may return to play if exam results are normal after 48 hours of physical rest DHe should terminate play for the season

Correct Answer ( D ) Explanation: A concussion, also known as mild traumatic brain injury (MTBI), is defined as an alteration in cerebral function secondary to a direct or indirect force on the brain. Current guidelines state that after an athlete's 3rd concussion, he or she should terminate the current season but may return to play the subsequent season if asymptomatic. An athlete can return to play if asymptomatic after 2 weeks (A) after his or her 1st concussion (Grade 2). An athlete can return to play if asymptomatic after 1 month (B) after his or her 1st concussion (Grade 3) or 2nd concussion (Grade 2). An athlete can return to play if they are asymptomatic with activity after 48 hours of physical rest (C).

A 16-year-old African American boy presents with a scalp rash. On examination, it is a 5 x 5 cm boggy and thickened area of the right parietal cap with an overlying scaly and crusty plaque and hair loss. The lesion appears yellowish-green under a Wood's lamp. What is the treatment of choice for this lesion? Clotrimazole ointment Ketoconazole shampoo Oral amphotericin B Oral griseofulvin

Correct Answer ( D ) Explanation: A kerion is best treated with oral griseofulvin. A kerion is an abscess caused by a fungal infection most commonly on the scalp but may also present on the upper limbs and the face. It appears as a boggy, pus-filled lesion with significant inflammation. The overlying skin often has an eczematous, itchy rash as well as hair loss. The patient may also have nearby lymphadenopathy, fevers, and malaise. The fungal infections that can cause a kerion include Microsporum canis and the Trichophyton genus. Exam with a Wood's lamp reveals yellow-green fluorescence. Scrapings and hair samples can be sent for microscopy and fungal culture to confirm the diagnosis. A course of six to eight weeks of an oral anti-fungal agent such as griseofulvin, itraconazole, or terbinafine is recommended. Antibiotics may also be indicated if a bacterial infection is also present.

2-year-old previously healthy boy presents with fussiness, rhinorrhea, and tugging on his right ear for the past day. Which of the following features is most sensitive for the diagnosis of acute otitis media? Absence of cough An erythematous tympanic membrane High-grade fever Impaired tympanic membrane mobility on pneumatic otoscopy

Correct Answer ( D ) Explanation: Acute otitis media refers to acute infection of the middle ear, and can be caused by bacterial, viruses, or fungi. The most common causes of bacterial infection in children are Streptococcus pneumoniae, nontypeable Haemophilus influenzae, and Moraxella catarrhalis. Common viral pathogens include respiratory syncytial virus, parainfluenza virus, rhinovirus, and adenovirus. Symptoms of acute otitis media are nonspecific and include ear pain or pulling on ears, fussiness, low-grade fever, cough, rhinorrhea, and vomiting and diarrhea. Physical exam features which support a diagnosis of otitis media include a tympanic membrane which is bulging and appears cloudy. Tympanic membrane hyperemia may be caused by crying or fever, but a deep red or hemorrhagic tympanic membrane is suggestive of acute otitis media. Air-fluid levels, bubbles behind the tympanic membrane, and an opacified tympanic membrane all indicate a middle ear effusion. Pneumatic otoscopy is helpful in confirming acute otitis media and impaired tympanic membrane mobility indicates a middle ear effusion and is one of the most sensitive indicators (96.5%) for acute otitis media.

A 15-year-old girl who follows a vegetarian diet is evaluated by her pediatrician for chronic fatigue. She is found to have a hemoglobin of 9.2 g/dL and ferritin of 15 ng/mL and is started on iron supplementation. Which of the following foods helps facilitate the absorption of iron? Black tea Bran cereal Milk Orange Juice

Correct Answer ( D ) Explanation: Ascorbic acid, which is found in orange juice, doubles the absorption of non-heme iron. In infants over 4 months of age their iron-fortified cereals can be mixed with applesauce as this also contains ascorbic acid. Iron regulation primarily occurs in the gastrointestinal tract and is determined by the amount of iron stores, the rate of red blood cell production, the amount of iron present in the diet, the type of iron present in the diet and the presence of other foods that can either inhibit or promote iron absorption. Meat, fish and poultry contain heme iron which is 2 to 3 times better absorbed than the non-heme iron found in iron-fortified foods and plants.

A 3-year-old boy is brought to the urgent care clinic by his mother. She tells you he has had a fever and cough for the past 2 days. His temperature is 38.6°C (101.5°F) and his respiratory rate is 35/min. Auscultation of the lungs reveals inspiratory stridor with a prolonged inspiratory phase. A chest X-ray shows subglottic narrowing. Which of the following is most likely the best treatment for this child's condition? Antibiotics Anticholinergics Bronchodilators Corticosteroids

Correct Answer ( D ) Explanation: Based on the constellation of findings, this patient most likely has croup (laryngotracheobronchitis). Croup typically presents with the classic triad of barking cough, inspiratory stridor, and hoarseness. It is most often caused by a viral infection (parainfluenza virus). Outpatient treatment usually consists of oral steroids (eg, oral dexamethasone) or nebulized epinephrine for severe cases. Corticosteroids are beneficial due to their anti-inflammatory actions. Since laryngeal edema is commonly seen in patients with croup, corticosteroids are extremely effective in decreasing this inflammatory process. Improvement is typically seen within 6 hours after administration of oral steroids.

A 10-month-old boy is being evaluated for bilious vomiting, diarrhea, and rectal bleeding. He appears to be in severe distress and is continuously crying. Which of the following should be used to both diagnose and treat this patient's condition? Abdominal CT scan Abdominal ultrasound Abdominal X-ray Contrast enema

Correct Answer ( D ) Explanation: Based on the constellation of findings, this patient most likely has intussusception. Intussusception occurs when one portion of the intestine invaginates into another portion. The most common form is intussusception of the terminal ileum into the right colon (ileocolic intussusception). Intussusception is the most common abdominal emergency in early childhood. Patients with a typical presentation (eg, infant or toddler with sudden onset of intermittent severe abdominal pain with or without rectal bleeding), can proceed directly to nonoperative reduction using a contrast enema (using either barium or air). In these cases, the procedure is both diagnostic and therapeutic. In patients with an atypical presentation of intussusception, ultrasonography is the method of choice to detect intussusception in many institutions. The classic ultrasound image of intussusception is a "target sign" which represents concentric circles of bowel.

A 14-year old obese boy presents with right knee pain. The physical exam reveals a tendency for the right hip to externally rotate. Which of the following is the most likely diagnosis? Legg-Calve-Perthes Medial meniscus tear Osgood-Schlater's disease Slipped capital femoral epiphysis

Correct Answer ( D ) Explanation: Based on the patient's age, weight, sex and physical exam, slipped capital femoral epiphysis (SCFE) is the most likely diagnosis. SCFE is caused by the slippage of the femoral head on the femoral neck. There are multiple forms. Acute SCFE generally presents with pain in the hip, groin or knee, gait abnormalities and decreased range of motion. In chronic SCFE, the most common presentation is intermittent groin, hip, medial thigh pain, or knee pain. This pain is usually dull in nature as opposed to the sharp pain seen in acute SCFE and is usually aggravated with activity. In SCFE, the hip assumes a position of relative external rotation and usually has limited flexion. Both acute and chronic SCFE are associated with a limp. Plain films show a widened and irregular physis, the "blanch sign of Steel" (femoral neck in front of the femoral head) and an abnormal Klein Line (especially in patients without symptoms). Treatment is necessary to prevent further slippage and includes casting, surgery, or both. Complications include osteonecrosis or avascular necrosis of the femoral head. Chondrolysis of the hip cartilage can also occur necessitating early total hip arthroplasty.

A 5-year-old boy with sickle cell disease presents with a limp, which his mother noticed two days ago. On exam, the patient is well appearing. Vital signs are significant for fever with temperature of 38.1°C. Heart rate and blood pressure are normal. The patient demonstrates an antalgic gait, and there is warmth and tenderness to palpation of the soft tissues in the distal thigh. Lab tests show a normal WBC count and elevated CRP and ESR. Plain radiographs of the femur show soft tissue swelling in the distal thigh as well as a periosteal reaction in the distal femur. Knee radiographs are normal. What is the most likely infecting organism? Haemophilus spp. Klebsiella spp Pseudomonas spp. Salmonella spp

Correct Answer ( D ) Explanation: This patient has osteomyelitis of the distal femur. Osteomyelitis, which is a pyogenic infection of the bone, occurs via hematogenous seeding (most common in children), seeding from a contiguous source of infection (eg, diabetic foot ulcer), or direct inoculation (eg, puncture wound, orthopedic procedure). Children will most often present with acute osteomyelitis (< 2 weeks) with signs of systemic infection such as fever and irritability in addition to localized bone pain and swelling. Adults commonly present with subacute or chronic osteomyelitis, and will have pain, swelling, as well as non-healing ulcers more frequently than they will have fever. Plain radiographs are the initial imaging choice in patients with osteomyelitis, though fewer than one third of patients will have any abnormalities in the first seven to ten days of admission. Periosteal reaction is the earliest osseous finding. The most common infecting organism in osteomyelitis for healthy hosts in all age groups except neonates is Staphylococcus aureus. Patients with sickle cell have an increased risk of osteomyelitis overall, especially from Salmonella spp. Additionally, Salmonella spp. are the most common pathogen identified in sickle cell patients with osteomyelitis.

A 7-month-old girl presents to the emergency department with difficulty breathing. Her parents report two days of cough and congestion with one day of increased work of breathing. On exam, her temperature is 37.4°C, heart rate 140, blood pressure 94/65, respiratory rate 65, and oxygen saturation 90%. She has moderate suprasternal and subcostal retractions, mild intercostal retractions and slight head bobbing. On auscultation, she has diffuse polyphonic wheezing with crackles. Which of the following is the next best step? Discharge home with supportive care Initiate bronchodilator therapy Obtain chest radiograph Suction nares

Correct Answer ( D ) Explanation: Bronchiolitis is a common lower respiratory tract infection in infants characterized by diffuse polyphonic wheezing and crackles. It is caused by viral infections, most notably respiratory syncytial virus (RSV), and occurs primarily in the winter season. Clinical findings include respiratory distress, polyphonic wheezing, and rales. Infants may exhibit hypoxemia due to ventilation-perfusion mismatch from mucous plugging of respiratory units. As respiratory failure progresses, patients develop hypercapnia and respiratory acidosis and may require mechanical ventilation. Most infants develop a mild and self-limited disease that can be managed at home with supportive care. However, infants with difficulty feeding, significant respiratory distress (as seen in this patient), or need for supplemental oxygen should be admitted to a hospital for further management and close monitoring. Supportive care remains the first-line therapy while in the hospital, which includes suctioning of the nares with saline drops.

An 11-month-old previously healthy boy presents in January with a two-day history of rhinorrhea, cough, and wheezing. Physical exam is significant for bilateral wheezing and crackles heard at the lung bases. The patient's pulse oximetry is 96 percent on room air. Which of the following is the most appropriate next step in management? Administer racemic epinephrine Albuterol nebulizer treatment every four hours Chest X-ray Observation and supportive care

Correct Answer ( D ) Explanation: Bronchiolitis is a lower respiratory tract viral infection that is caused by damage of the epithelial cells lining the small airways, which leads to acute inflammation, increased mucous production, and bronchospasm. Bronchiolitis peaks between December and March. The clinical presentation includes rhinorrhea, cough, wheezing, low-grade fever, and shortness of breath. Bilateral wheezing and crackles are heard on pulmonary exam. Poor feeding and respiratory distress (tachypnea, nasal flaring, and hypoxemia) are indicators of increased severity. Oxygen saturation less than 95 percent, PO2 less than 65, PCO2 greater than 40, atelectasis on chest X-ray, and respiratory rate greater than 70 are also all indicators of severe disease. Bronchiolitis most commonly occurs between two and seven months of age, but can occur in those up to two years old. The most common virus associated with bronchiolitis is respiratory syncytial virus (RSV). Other causes include parainfluenza virus, metapneumovirus, and adenovirus. Symptoms can last up to five days and generally worsen by day three before subsiding. Most patients can undergo observation and supportive care. Treatment is symptomatic and includes cool mist. Hospitalization is determined by the patient's comorbidities, reliability of parents, duration of symptoms, and condition on presentation (hypoxia, tachypnea, dehydration, age).

A 4-year-old Caucasian boy presents to your office with watery nonbloody diarrhea for the last 3 months. He has occasional abdominal pain prior to the onset of the diarrhea and his mother noted worsening flatulence, especially after meal times. Mom denies any recent travel or camping and the patient does not go to school or daycare. On exam, he is 50th percentile for weight and height, appears well, and has a normal abdominal exam. What is the most likely diagnosis? Celiac disease Inflammatory bowel disease Irritable bowel syndrome Lactase deficiency

Correct Answer ( D ) Explanation: Congenital lactase deficiency is rare and is associated with symptoms occurring on exposure to milk. However, fewer than 50 cases have been reported worldwide. Secondary lactose intolerance follows small bowel mucosal damage from conditions such as celiac disease and rotavirus infection and is usually transient, improving with mucosal healing. Patients can develop diarrhea, abdominal cramps, abdominal pain, and flatus following ingestion of milk products. Symptom severity depends on the amount of lactose ingested and the fat content of the product (skim milk empties from the stomach faster causing more symptoms). It is not associated with a failure to thrive. Lactase deficiency can be diagnosed by H2-breath test or by measurement of lactase activity in mucosa tissue retrieved by small bowel biopsy. Diagnostic testing is not mandatory, and often simple dietary changes that reduce or eliminate lactose from the diet relieve symptoms. Treatment of lactase deficiency consists of a milk-free diet. A lactose-free formula (based on either soy or cow's milk) can be used in infants. In older children, low-lactose milk can be consumed. Addition of lactase to dairy products usually lessens the symptoms.

An 7-year-old boy with a history of cystic fibrosis presents with increased cough and fever. He has been doing well and has not been hospitalized for pneumonia in the last seven months. His X-ray demonstrates pneumonia and he requires supplemental oxygen. Which of the following antibiotic regimens is most appropriate? Ceftriaxone and azithromycin Ceftriaxone and doxycycline Moxifloxacin Piperacillin/tazobactam

Correct Answer ( D ) Explanation: Cystic fibrosis (CF) is caused by a mutation in the CF transmembrane conductance regulator gene and transmitted in an autosomal recessive pattern. The mutation leads to impaired chloride transport across the airway epithelium causing multiple problems within the lung including impaired ciliary clearance of mucus, decreased antimicrobial effect of the airway surface, increased bacterial adherence, and impaired secretion of cytokines. As a result, patients develop recurrent bacterial pneumonias over the course of their life. Approximately 80% of patients with cystic fibrosis are colonized with Pseudomonas aeruginosa by age 18. Therefore, active pneumonia infections are treated with broad-spectrum antibiotics with adequate activity against Pseudomonas, such as Piperacillin/tazobactam. It is also important to evaluate previous culture data for resistance patterns as patients with cystic fibrosis are on frequent antibiotics including some prophylactic agents leading to growing resistance as they age. In addition to antibiotics, patients are treated with bronchodilators and mucolytics to improve airway clearance and chest physiotherapy.

A G2P0010 woman presents with painful bleeding at 37 weeks by last menstrual period. She reports no prenatal care during the pregnancy. She is induced and delivers a male infant who is cyanotic, tachycardic, tachypneic, and noted to have generalized edema. The infant requires multiple interventions for stabilization including intubation. Which of the following is the most likely etiology of his presentation? Erythema marginatum Erythema multiforme Erythema toxicum neonatorum Erythroblastosis fetalis

Correct Answer ( D ) Explanation: Erythroblastosis fetalis is caused by the transplacental passage of maternal antibody active against paternal RBC antigens of the infant and is characterized by an increased rate of RBC destruction that leads to anemia and jaundice in newborn infants. Isoimmune hemolytic anemia occurs in Rh-positive infants born to Rh-negative mothers. This rarely occurs during a first pregnancy, as the mother must form antibodies against the fetal D antigen. Once the mother is sensitized to Rh antibodies, even a small amount of antigen can stimulate antibody production. Maternal IgG antibodies cross the placenta resulting in fetal hemolytic disease. The severity of symptoms can range from mild anemia to hydrops fetalis and death. When the hematopoietic system is exceeded, profound anemia occurs and results in pallor, cardiac decompensation, respiratory distress, and anasarca, and circulatory collapse. Treatment includes supportive care, blood transfusion, exchange transfusion, and IVIG. In order to prevent sensitization, Rh negative should receive anti-D globulin in the following situations: after giving birth to a Rh positive infant, ectopic pregnancy, abdominal trauma in pregnancy, amniocentesis, chorionic villus biopsy, or abortion. The patient in the above scenario had an abortion and likely never received human anti-D antibody, which led to sensitization and erythroblastosis fetalis.

A 22-month-old whose immunizations are up to date presents after a generalized seizure. Mom reports one seizure lasting less than five minutes. She has had rhinorrhea for the last two days and has a temperature of 102°F. On exam, she is well appearing and has returned to baseline. Which of the following is the most appropriate plan? Antiepileptic therapy CBC and blood culture Lumbar puncture Supportive care

Correct Answer ( D ) Explanation: Febrile seizures are defined as the presence of seizure in a febrile child without CNS infection. They are a common cause of seizure in children and most commonly occur between 6 months and 5 years of age with a peak occurrence between 14 and 18 months. The risk of seizure is correlated more with the rapid rise in temperature rather than an absolute number. A simple febrile seizure is brief, single, and nonfocal or generalized. Complex febrile seizures are prolonged, recurrent or focal. Evaluation of the simple febrile seizure is dictated by the evaluation of the fever and not the seizure. Patients should receive supportive care for the post-ictal state and their underlying illness. The use of antipyretics does not decrease the frequency or recurrence of seizures.

A 2-year-old boy presents to the emergency department with difficulty breathing. His mother notes "a cold" that began this morning with a fever to 101.3°F. He then developed a barky cough and harsh breathing. On examination, you note he is tachypneic with suprasternal and subcostal retractions and stridor with every breath. The most appropriate next step is administration of which of the following? Albuterol Dexamethasone Ipratropium Racemic epinephrine

Correct Answer ( D ) Explanation: In a febrile child, croup (laryngotracheitis) is by far the most common cause of stridor. It is a viral infection most commonly due to parainfluenza virus that leads to inflammation of the upper airway. Croup typically affects children between 7 months and 3 years of age, but can happen at any age. It typically starts with an upper respiratory febrile illness that progresses to a barky, "seal-like" cough in the first 1-2 days. Stridor at rest associated with tachypnea or increased work of breathing indicates a severe case. In severe croup, racemic epinephrine should be the first line therapy. Racemic epinephrine stimulates alpha and beta adrenergic receptors in the airway resulting in mucosal constriction and smooth muscle relaxation. Onset is rapid, and the duration of action is about 2 hours

A mother brings her full-term 3-day-old male to the ED after she noticed an episode of dark, tarry stool at home. Mom delivered the newborn vaginally with no complications. Mom is a vegan and is breastfeeding. On exam, the patient is afebrile. His abdomen is soft with no palpable masses. Rectal exam reveals dark, tarry stool that is guaiac positive. Which of the following is the most likely cause of this patient's presentation? AMeckel's diverticulum BMilk protein allergy CNecrotizing enterocolitis DSwallowed maternal blood

Correct Answer ( D ) Explanation: In the neonatal period, the most likely cause of lower gastrointestinal bleeding is swallowed maternal blood, from either delivery or cracked nipples during breast-feeding. The Apt test differentiates fetal from maternal blood. The blood in question is mixed with alkali to detect conversion of oxyhemoglobin to hematin. Fetal hemoglobin is more resistant to denaturation than adult hemoglobin is. If the supernatant stays pink after addition of alkali, the blood is fetal in origin (a positive test). This should not be confused with the Kleihauer-Betke test, used to detect fetal-maternal hemorrhage in the pregnant female.

An 8-month-old boy fails to obtain age-appropriate milestones. His mother feels that he is too fatigued to properly feed. An echocardiogram reveals a large atrial septal defect. Which of the following physical examination abnormalities would you most expect to find in this infant? Bradycardia Cyanosis Prominent left ventricular impulse Widely split and fixed S2

Correct Answer ( D ) Explanation: In the vast majority of cases, an atrial septal defect results in an intracardiac left-to-right shunt, which typically manifests as right ventricular overload and increased stroke volume. ASDs are associated with some physical examination abnormalities, depending on the degree of shunting and the compliance of the ventricles, systemic vasculature and peripheral vasculature. A hyperdynamic right ventricular impulse may be appreciated. Pulmonary artery dilation may produce an ejection click. Splitting of S1 and S2 is also common. In fact, a widely split and fixed S2 is very common in large left-to-right shunts. Large shunts can also produce a left sternal murmur, heard higher as a systolic crescendo-decrescendo murmur, and lower as a middiastolic rumble.

You are seeing in your office a four-week-old boy with vomiting. Which of the following findings is most concerning for pyloric stenosis as an underlying cause? Bilious emesis Hyperkalemic, hypochloremic metabolic alkalosis Vomitus with the appearance of "curdled milk" Weight loss

Correct Answer ( D ) Explanation: Infantile pyloric stenosis should be considered in a neonate with vomiting and weight loss. The classic presentation of infantile pyloric stenosis is in a neonate one to three months of life with progressively forceful, non-bilious emesis. Males predominate over females in 5:1 ratio. A history of erythromycin exposure should also raise suspicion. Individuals with infantile pyloric stenosis typically display immediate hunger after vomiting. The classic exam reveals an olive-shaped mass in the epigastrium with peristaltic waves visible in the left upper quadrant. Classic presentation also includes dehydration and a hypochloremic, hypokalemic metabolic alkalosis. It should be noted that despite the classic "textbook" presentation of infantile pyloric stenosis, patients tend to present earlier, are less dehydrated, and have fever electrolyte abnormalities. This is thought to be due to improved awareness and imaging capabilities.

A 2-month-old child presents with projectile vomiting. The child initially remains hungry following the episodes of vomiting but, in time, loses interest in feeding and presents to the emergency department appearing wasted and severely dehydrated. On physical exam the patient appears dehydrated and a small olive like structure can be palpated in the right upper quadrant. What is the most likely diagnosis? Formula intolerance Gastroesophageal reflux disease Hirschprung disease Pyloric stenosis

Correct Answer ( D ) Explanation: Infants with infantile hypertrophic pyloric stenosis (IHPS) are typically asymptomatic until three to four weeks of age, although a small number may present as early as the first week of life. Initially infants present with mild spitting, which progresses to nonbilious projectile vomiting following feedings. Vomiting may be so forceful that it exits through the nostrils, as well as the mouth. Emesis may contain "coffee ground" material or small amounts of frank blood, but is not bilious. Early in the course the infant remains hungry following vomiting episodes but, with time, loses interest in feeding and may present wasted and severely dehydrated. Infantile hypertrophic pyloric stenosis is a form of gastric outlet obstruction caused by hypertrophy of circular muscle surrounding the pyloric channel. Correction of IHPS is the most common abdominal operative procedure during the first six months of life. Infantile hypertrophic pyloric stenosis is arguably not a true congenital defect because the muscular hypertrophy and obstruction tend to be an evolving process during the postnatal period. On physical examination the infant with IHPS may appear wasted and dehydrated, but the extent is variable and related to severity and duration of symptoms. The classic physical signs are a palpable pyloric mass and visible peristaltic waves. The palpable "olive" is most easily felt in a wasted patient, immediately following emesis or aspiration of the stomach. Boys outnumber girls by a ratio of 4-5:1. The initial therapy for IHPS is fluid and electrolyte replacement to correct dehydration and hypochloremic metabolic alkalosis. Depending on severity, fluid and electrolyte repletion can usually be accomplished within 24 hours. Definitive therapy is the Ramstedt pyloromyotomy, which entails a longitudinal incision through the hypertrophied pyloric muscle down to the submucosa on the anterior surface of the pylorus. Hirschprung disease (C) would present as early failure to pass meconium, vomiting and abdominal distention.

A 5-year-old boy presents with abdominal pain, grogginess and vomiting. His father states his son's stools have had a jelly-like appearance for the past 36 hours. You appreciate a sausage-shaped mass during abdominal palpation. Which of the following diagnostic tests is the most appropriate during this child's diagnostic evaluation? Computed tomography KUB radiograph Magnetic resonance imaging Ultrasonography

Correct Answer ( D ) Explanation: Intussusception occurs when a proximal portion of bowel telescopes into a more distal portion, typically with the ileum inserting through the ileocecal valve. It is the most common cause of bowel obstruction in infants between the ages of 3 and 12 months. Its peak incidence is between the ages of 5 and 9 months, although it can occur anytime through childhood and even rarely in adults. Symptoms include vomiting, abdominal pain and lethargy. The classic quality of the affected patient's stool is "currant jelly", a reference to the bloody appearance of mucus-like stool and is usually a late finding. A right hypochondrial sausage-shaped mass, with emptiness appreciated in the right lower quadrant (positive Dance sign) may be palpable. If the obstruction is complete, the abdomen could be quite distended. Clinical diagnosis can be supported with positive findings found on a barium enema, however, for pediatric cases, many recommend ultrasonography as the first-line diagnostic test, as the sensitivity and specificity of this imaging modality are reported to be above 95%. Treatment involves supportive care and an air contrast enema. Surgical intervention is indicated if the contrast enema fails to reduce the intussusception or if the intussusception recurs. Plain abdominal radiography (KUB) (B) can miss up to 40% of true intussusception. MRI (C) may delay diagnosis and is not indicated in the diagnostic work-up of intussusception.

A 4-day-old, former 38 week gestation newborn presents to clinic for follow up. She was born without complications and was discharged with her mother yesterday. The mother's pregnancy and prenatal labs were unremarkable. The mother is concerned that the whites of the newborn's eyes have begun to appear yellow and her face now also appears yellow. She has been breastfeeding without difficulty, has been awake and alert, and has had five wet diapers and four yellow-green stools today. Which of the following is most likely to be contributing to this newborn's jaundice? Abnormal RBC shape Absence of bilirubin glucuronidation Intestinal obstruction Shortened red blood cell lifespan

Correct Answer ( D ) Explanation: Jaundice is common amongst newborn infants. It results from an elevation of serum bilirubin, a normal product of the breakdown of heme within red blood cells (RBCs). Etiologies of newborn jaundice range from a normal physiologic phenomenon to life-threatening pathology. In otherwise healthy infants, such as the one described above, physiologic jaundice has three main causes. First, newborn's have a higher hematocrit than adults, and their red blood cell lifespan is shortened to less than 90 days. The increased number of red blood cells, as well as their higher turnover rate, results in increased bilirubin production. Hematocrit will naturally decrease with age and RBC lifespan will increase. Second, newborns have delayed clearance of bilirubin due to decreased activity of the enzymes of conjugation within the liver. Bilirubin clearance depends on the conjugation of bilirubin to glucuronic acid, which is then excreted through bile. The enzyme responsible for conjugation is uridine diphosphate glucuronosyltransferase (UGT), and newborns have a relative UGT deficiency. UGT activity will naturally increase in time in otherwise normal newborns. Finally, newborns have increased enterohepatic recirculation of bilirubin compared to older children. Absence of bilirubin glucuronidation (B) occurs in Crigler-Najjar syndrome type-I, which is a rare autosomal recessive condition in which UGT activity is absent. Without continuous phototherapy and liver transplantation, the affected infant is at risk of bilirubin-induced neurological dysfunction. Although normal newborns have decreased UGT activity, it is not absent.

What neonatal reflex is present at birth and remains for life? Moro Palmar/Plantar Grasp Sucking Withdrawal

Correct Answer ( D ) Explanation: Monitoring milestones is an important part of health surveillance and is vital a component in parental education. Absent milestones may indicate developmental delay and an earlier need for intervention. As part of the neurologic examination, the assessment should include primitive and protective reflexes, symmetry, tone, muscle strength, and deep tendon reflexes. A newborn is able to have the following reflexes: rooting, sucking, moro/startle, withdrawal, palmar/plantar grasp, asymmetric tonic neck, babinski, and knee jerk. The withdrawal reflex is the ability of an infant to move the hand or foot from painful stimuli. This reflex is present at birth and remains for life. The moro reflex (A) is a sudden change in position or loud noise that causes an extension and flexion of the infant's arms and fingers. It appears in utero at 28 weeks of gestation and disappears between 3-6 months of age. The palmar/plantar grasp (B) is the curling of fingers or toes around an object. It is present at approximately 32 weeks gestation and is replaced by voluntary grasp between 3-4 months of age. The sucking reflex (C) is the response of an object in the mouth of an infant that causes the infant to suck and draw liquid into the mouth. This reflex begins between 32-36 weeks gestation and disappears around 3 months of age.

A 6-week-old newborn is being evaluated for projectile vomiting and failure to thrive. The mother tells you the infant vomits immediately after every feeding. The vomitus appears to be nonbilious. Palpation of the right upper quadrant of the abdomen reveals an "olive-like" mass. Laboratory studies show a hypochloremic, metabolic alkalosis. Which of the following is the most likely diagnosis? Duodenal atresia Gastroesophageal reflux Hirschsprung's disease Pyloric stenosis

Correct Answer ( D ) Explanation: Nonbilious projectile vomiting in a 6-week-old newborn, is highly suggestive of infantile hypertrophic pyloric stenosis (IHPS). Pyloric stenosis is a narrowing between the distal portion of the stomach (pylorus) and the proximal portion of the duodenum. These newborns classically present with immediate postprandial vomiting and electrolyte imbalances. The hypertrophied pylorus is usually palpated as an "olive-like" mass in the right upper quadrant. This physical examination finding is typically pathognomonic for this condition. Laboratory studies show a hypochloremic, metabolic alkalosis due to the loss of large amounts of gastric acid. In addition to fluid administration and correction of electrolyte imbalances, a pyloromyotomy is usually the definitive form of treatment.

A 13-month-old girl presents with her mother who says that her daughter has been sick for a 10 days. The girl had an initial period of "head-cold symptoms" and is now coughing constantly. She is a well-nourished infant who demonstrates a rapid, consecutive cough with a high-pitched inspiratory "whoop." Records show that the family declined vaccinations. Which of the following interventions is most indicated in managing this patient's illness? Humidification of inspired air Nebulized albuterol Oral amoxicillin Oral clarithromycin

Correct Answer ( D ) Explanation: Oral macrolide antibiotics, using either erythromycin, azithromycin, or clarithromycin, are the first-line treatment for the respiratory infection pertussis. Pertussis is an acute respiratory illness that primarily causes illness in children under 2 years of age. Though immunization exists against the causative pathogen, Bordetella pertussis, neither immunization nor history of infection ensures lasting immunity, and adults often serve as infectious reservoirs for the disease. Pertussis generally begins with a one to two-week period of malaise, sneezing, and anorexia. It is followed by a persistent cough, with the classic, high-pitched inspiratory whoop. Posttussive emesis also suggests pertussis. Non-specific laboratory findings can include a white blood cell count of 15,000-20,000 with up to 80% lymphocytes. However, pertussis is definitively diagnosed by isolating the organism from a nasopharyngeal culture. A 4-7 day course of treatment with a macrolide antibiotic should be used to reduce coughing severity (if given early enough) and shorten the duration of carriage. Close contacts should receive the same treatment as prophylaxis against illness.

A 16-year-old girl presents to clinic with 3 weeks of worsening right knee pain. She attends dance class since age 5 and now dances 20 hours a week. She denies any inciting injury and continues to dance on the injured leg. On exam, she has pain at the inferior and medial pole of the right patella with no swelling or erythema of the knee. Which of the following is the most likely diagnosis? Anterior cruciate ligament rupture Medial meniscus tear Patella dislocation Patellofemoral pain syndrome

Correct Answer ( D ) Explanation: Patellofemoral pain syndrome occurs in about a quarter of those involved in athletics and more commonly in women and those between the ages of 10 and 35 years. Symptoms include pain with going up and down stairs and prolonged sitting or squatting. Individuals may have a sensation of the knee buckling or giving way. Swelling, popping or grinding sensations may be present. Treatments include non-steroidal anti-inflammatory medications, ice, quadriceps strengthening, stretching, patella bracing and orthotics.

Which of the following statements best explains the use for penicillin in patients with acute rheumatic fever? Penicillin is only recommended for patients with carditis during their acute rheumatic fever Penicillin is recommended to prevent Group A Strep colonization, which is more common in patients with acute rheumatic fever Penicillin is used for its anti-inflammatory effect on the affected heart valves Penicillin is used to prevent future episodes of acute rheumatic fever, which can lead to worsening heart disease

Correct Answer ( D ) Explanation: Penicillin is recommended as prophylaxis for patients with rheumatic heart disease to prevent future Group A Strep infections, which can lead to recurrence of acute rheumatic fever and worsening heart disease. Prior to the availability of penicillin prophylaxis, up to 75% of patients with rheumatic heart disease had a recurrence of acute rheumatic fever. Such recurrences can cause worsening of heart disease in patients who had carditis during their initial episode of acute rheumatic fever. The risk of recurrent acute rheumatic fever decreases with time from the initial episode of acute rheumatic fever.

2-year-old girl presents to the ED reluctant to move her left arm. The patient's father states that he grabbed his daughter by her wrist to pull her up from the floor. On exam, there is no swelling or deformity. Which of the following maneuvers should be attempted to correct the injury? Full elbow flexion followed by pronation Hypersupination of the forearm Pronation followed by elbow flexion Supination followed by elbow flexion

Correct Answer ( D ) Explanation: Radial head subluxation (aka nursemaid's elbow) most commonly occurs in children 2 to 3 years of age with a predilection toward females. The injury is due to displacement of the radial annular ligament into the radiocapitellar articulation. Clinically, the child will not move the affected arm but otherwise is in no distress. There are two maneuvers for reduction: supination technique and the hyperpronation technique. For the supination technique, hold the elbow at 90 degrees, firmly supinate the child's wrist, and then flex the child's elbow directing the wrist toward the ipsilateral shoulder. The hyperpronation technique is performed by holding the child's elbow at 90 degrees and then firmly pronating at the wrist. After successful reduction, relief is immediate, and the child typically begins to move the affected extremity within 5 to 10 minutes.

A 12-month-old girl presents with recurrent urinary tract infections. The external genital examination is normal. Renal ultrasonography shows hydronephrosis. Voiding cystourethrography does not reveal any detrusor abnormality. Which of the following is the most likely diagnosis? Bladder diverticulum Cryptorchidism Epispadia Vesicoureteral reflux

Correct Answer ( D ) Explanation: Retrograde passage of urine from the urinary bladder into the ureter is called vesicoureteral reflux (VUR), a condition which causes urinary tract infection in 30-40% of infants and toddlers. Causes are many, but most VUR is caused by a congenital abnormality of ureterovesicular junction, particularly a defect in the normal flap-valve mechanism in the intramural ureteral canal. An upper tract bacterial infection is a common result. Over time, recurrent kidney infections can lead to renal scarring, dysfunction and systemic hypertension in the adult population. Diagnosis is made by ultrasonography and voiding cystourethrography (VCUG). Symptoms may resolve spontaneously over months to years. Treatment may include prophylactic antibiotics and ureteral re-implantation.

What is the most common cause of acute bacterial otitis media in children? Haemophilus influenzae Pseudomonas aeruginosa Staphylococcus aureus Streptococcus pneumoniae

Correct Answer ( D ) Explanation: Streptococcus pneumoniae is the most common cause of acute otitis media in children. Acute otitis media (AOM) is defined as inflammation of the middle ear. AOM is the most common reason for office visits and is the most common reason for antibiotic use in children. AOM is more common in boys than in girls. Other risk factors include family history, day care attendance, lack of breastfeeding, exposure to tobacco smoke, pacifier use, and poor socioeconomic status. In children with AOM, Streptococcus pneumoniae, Haemophilus influenzae, and Moraxella catarrhalis are the most common bacterial pathogens isolated from middle ear aspirates. Otalgia is the most common complaint and is the best predictor of AOM in children. Other symptoms include fever, irritability, headache, vomiting, and diarrhea. A bulging and erythematous tympanic membrane is the key physical exam finding. Pneumatic otoscopy will show decreased tympanic membrane mobility in AOM. The diagnosis of AOM is based on tympanic membrane bulging and signs of acute inflammation. Immediate antibiotic use is recommended in certain populations based on age, degree of symptoms, duration of symptoms, and underlying co-morbidities. Amoxicillin is first-line therapy in most patients. Amoxicillin-clavulanate is first-line in children who have received a beta-lactam antibiotic in the previous 30 days. Cephalosporins and macrolides are first-line in patients allergic to penicillin. Oral ibuprofen or acetaminophen are recommended for pain control. Symptoms usually improve within 24 to 72 hours with appropriate antibiotic use.

An infant with episodes of cyanosis and respiratory difficulty is diagnosed with Tetralogy of Fallot. Which of the following defects would you expect to see on echocardiographic examination? Aorta-pulmonary artery transposition Atrial septal defect Left ventricular hypertrophy Overriding aorta

Correct Answer ( D ) Explanation: Tetralogy of Fallot (TOF) is a congenital heart disease characterized by four defects: ventricular septal defect, overriding aorta, right ventricular hypertrophy and right outflow obstruction. TOF is the most common cyanotic congenital heart disease. Common symptoms compile the classic "tet spell": cyanosis, hyperpnea and irritability, accompanied by a decrease in murmur intensity. Symptomatic treatment includes squatting, which increases peripheral vascular resistance and improves oxygenation. Other treatment includes screening and preventing anemia and subacute bacterial endocarditis prophylaxis. Surgical palliation until the infant is mature to undergo full corrective surgery involves a subclavian-to-pulmonary artery shunt.

Which of the following is a cyanotic congenital heart disease? Atrial septal defect Coarctation of the aorta Patent ductus arteriosus Tetralogy of Fallot

Correct Answer ( D ) Explanation: Tetralogy of Fallot is a cyanotic congenital heart defect. Cyanotic congenital heart disease (CHD) results either from decreased pulmonary blood flow to the lungs or right-to-left shunting of desaturated blood into the systemic circulation. The classic cyanotic CHDs include the 5 "t's" - truncous arteriosus, transposition of the great vessels, tetralogy of Fallot, total anomalous pulmonary venous return and tricuspid atresia. Tetralogy of Fallot accounts for 10% of all CHDs. It consists of 4 abnormalities: right ventricular outflow obstruction, over-riding aorta, large VSD and right ventricular hypertrophy. Together, these defects result in right-to-left shunting of deoxygenated blood and decreased pulmonary blood flow leading to cyanosis.

A 14-year-old girl is in your clinic with her mother because of leg pain. She fell from her bike last month and hit her left leg. Since then, she has complained of pain on her left leg that waxes and wanes. On physical examination, you could palpate a soft tissue mass that is tender. Laboratory findings show elevated alkaline phosphatase and elevated lactate dehydrogenase. Radiograph reveals destruction of the normal trabecular bone pattern and a soft tissue mass that is ossified in a radial or sunburst pattern. Which of the following is the most likely diagnosis? Chondroblastoma Ewing sarcoma Osteomyelitis Osteosarcoma

Correct Answer ( D ) Explanation: The "sunburst" pattern is a type of periosteal reaction giving the appearance of a sunburst, secondary to an aggressive periostitis, classically associated with osteosarcoma. Chondroblastoma (A) is a benign bone tumor and is characterized by a well-defined or sclerotic border, lack of destruction of the cortex, and lack of extension into the soft tissue on radiograph. Ewing sarcoma (B) is a neoplastic disease where radiographs show periosteal reaction that produces layers of reactive bone, deposited in an "onion peel" appearance. Osteomyelitis (C) is infection localized to bone and presents with systemic symptoms (fever) and local findings (tenderness, warmth, erythema, and swelling).

A 5-year-old girl presents to the ED with a rash that started on her face and spread to her neck, axillae, and groin. Mom states that the patient had an upper respiratory infection one week prior. On examination, the patient's rash is tender to the touch. Which of the following statements regarding the diagnosis of this patient's condition is correct? Deep layers of the dermis are involved It often leaves the patient disfigured from scarring Mucous membrane involvement is common The disease is caused by an exotoxin-producing bacterium

Correct Answer ( D ) Explanation: This patient has staphylococcal scalded skin syndrome (SSSS), which is caused by an exotoxin-producing strain of Staphylococcal aureus. SSSS is actually a severe form of bullous impetigo. It is usually seen in children <5 years of age and often follows a URI. The rash usually starts on the face (perioral is classic), neck, axillae, and groin. The rash then spreads, becomes exfoliative, and is followed by the development of flaccid bullae and skin desquamation. The rash is characteristically tender to the touch, and gentle traction on the affected skin produces peeling (positive Nikolsky's sign).

A 32-year-old woman has just delivered a term infant in the hospital parking lot. The infant is crying and moving her arms and legs. She coughs occasionally. Her face and trunk are pink, but her hands and feet are cyanotic. Her heart rate is 130 beats/minute. What is her APGAR score? 10 6 8 9

Correct Answer ( D ) Explanation: The APGAR score for newborns measures five aspects of newborn physiology, and is assessed at one and five minutes following birth. Appearance (A) assesses skin color and scores zero for a pale or cyanotic appearance, one point for normal appearance other than cyanosis of extremities, and two points for normal appearance throughout. Pulse (P) is scored with zero for absent pulse, one point for a pulse < 100 beats/min, and two points for pulse > 100 beats/min. Grimace (G) evaluates reflexes and scores zero for no response, one point for grimacing, and two points for active response (i.e., pushing away and coughing). Activity level (A) is scored with zero for absent muscle tone, one point for arms and legs flexed, and two points for active movement of the neonate. Respirations (R) measures respiratory effort and scores zero for absent respirations, one point for slow or irregular respirations, and two points for normal respirations or crying. Neonates with an APGAR score of seven or less require active resuscitation and should have sequential APGARs assessed every five minutes until the score is seven or above. This neonate has a score of nine. She receives one point for appearance (extremity cyanosis). She receives two points for pulse (pulse is > 100). She receives two points for grimace (coughs). She receives two points for activity (moving her arms and legs actively). She receives two points for respirations (crying).

A 12-year-old girl is brought to the clinic for evaluation of hip pain. She describes the pain as dull, non-radiating and aching pain on the right hip that is increased with physical activity and slightly relieved with rest. The pain has been intermittent for the past four weeks. She denies any trauma. On physical exam, weight is at the 95th percentile, the right anterior hip is tender to palpation, and there is decreased internal rotation, abduction, and flexion of the right hip. The rest of the examination is normal. Radiographs show a blurring of the junction between the metaphysis and the growth plate on AP view and step-off of the epiphysis on the femoral neck on frog-leg view. Which of the following is the next best step in management? AAdvise rest and close follow-up BObtain magnetic resonance imaging CPrescribe an anti-inflammatory medication DRefer to an orthopedic surgeon

Correct Answer ( D ) Explanation: The clinical presentation of the girl is consistent with slipped capital femoral epiphysis (SCFE). It is one of the most common hip disorders of adolescents. It occurs at the time of peak linear growth. The most common presentations are pain and altered gait. The classic feature is that of an obese adolescent complaining of pain in the hip, groin, thigh, or knee and no history of preceding trauma. The pain may be chronic or intermittent, and it is increased by physical activity and relieved with rest. The gait may be antalgic if the SCFE is unilateral. The anterior hip may be tender to palpation. The range of motion of the hip shows decreased internal rotation, abduction, and flexion but may be painful in all directions. The degree of restriction of range of motion depends upon the severity of the slip. The diagnosis is made based upon plain radiographs that reveal an apparent posterior displacement of the femoral epiphysis, like ice cream slipping off a cone. Early plain radiographic changes show widening and irregularity of the physis, with thinning of the proximal epiphysis that are best seen on lateral views. The treatment of SCFE is operative stabilization, and children with SCFE should be referred promptly to an orthopedic surgeon.

A six-year-old presents with his mother who is concerned about a new rash. What is the etiology of the lesions pictured above? Autoimmune Bacterial Fungal Viral

Correct Answer ( D ) Explanation: The condition seen in the case above is molluscum contagiosum. This is a common condition caused by a DNA poxvirus. Lesions are raised, umbilicated papules and often appear in crops of five to 10. It is more common in swimmers and wrestlers due to dermal exposure. Patients with atopic eczema are especially at risk. Spread is via contact with the waxy material found in the central core of the lesion. Autoinoculation is common. Lesions typically persist for six to nine months then spontaneously resolve. Lesions may become irritated and superinfected. Extrusion of the central core debris is curative. Other treatment options include tretinoin 0.1%, cantharidin, or liquid nitrogen. Since spontaneous resolution is common, observation is most commonly used.

A six-year-old girl presents with a limp. The parents noted that the girl was having difficulty walking for the past two days. They deny trauma or fever but did note a cold one week prior. On physical exam, the girl appears well and is afebrile, with the left hip held flexed and abducted. Laboratory tests reveal normal erythrocyte sedimentation rate (ESR), C-reactive protein (CRP), and WBC. Ultrasound shows the presence of joint effusion of the left hip. Which of the following of the most likely diagnosis? Myositis Septic arthritis Stress fracture Transient (toxic) synovitis

Correct Answer ( D ) Explanation: The girl has clinical and laboratory findings most consistent with transient synovitis. Toxic synovitis is a reactive arthritis that affects the hip. It is one of the most common causes of hip pain in children. It can manifest in all age groups with mean onset at six years of age. The majority of affected patients have an upper respiratory illness one to two weeks before the onset of symptoms. The symptoms usually develop acutely with pain in the groin, anterior thigh, knee or hip and children walk with a painful and limping gait. If a significant effusion is present, the hip is held flexed, abducted, or laterally rotated. Affected patients are often afebrile or have a low-grade fever. The diagnosis is made clinically and laboratory and radiographic tests are performed to rule out serious conditions. The ESR, CRP, and WBC are relatively normal. The radiographs of the pelvis when obtained are usually found to be normal. Ultrasonography of the hip may show joint effusion. Treatment is supportive that includes activity limitation and relief of weight bearing until the pain subsides

A 9-year-old boy presents with 3 months of progressive spine stiffness. He also complains of hip, ankle, wrist and digit pain. There is no rash present. Physical and laboratory examination supports a diagnosis of juvenile rheumatoid arthritis. Which of the following is this patient most likely to be prescribed? Colchicine Doxycycline Lamivudine Methotrexate

Correct Answer ( D ) Explanation: The most common childhood arthritis is juvenile idiopathic arthritis (JIA). Onset must be less than 16 years of age and symptoms persist > 6 weeks. Other causes of childhood arthritis include vasculitis, rheumatic fever and septic arthritis. There are three main subtypes. Systemic JIA, also known as Still's disease, occurs in 1-6 year old children, and is characterized by intermittent spiking fever and rash. Rheumatoid factor (RF) is negative in about 98% of these cases. Pauciarticular JIA is an oligoarthritis that occurs in girls earlier than boys, and is characterized by few systemic findings except for serious ophthalmologic complications of uveitis and iridocyclitis. Again, RF is negative in more than 98% of these patients, however, HLA-B27 is commonly positive. The most common subtype, responsible for 90-95% of all JIA, is termed Polyarticular. It is characterized by an onset > 8 years of age, and is marked by insidious edema and stiffness, especially in the spine and hip. This type may lead to growth retardation due to early epiphyseal plate closure. Systemic features are rare, however, when this type occurs in an unremitting pattern, it carries the worst prognosis of all the subtypes. RF is positive in only 5-10% of polyarticular JIA, and is present most commonly in females over the age of 10 years. Overall, 70% of children with JIA improve without serious disability, but 10% succumb to devastating functional impairments. A multidisciplinary approach is necessary for maximum treatment results, and includes physiatry, rheumatology, ophthalmology, physical and occupational therapy and behavior health therapy, as well as NSAIDs, salicylates, methotrexate, sulfasalazine, etanercept, adalimumab, infliximab, intraarticular steroids and possibly orthopedic surgery. Lamivudine (C) is a nucleoside reverse transcriptase inhibitor used in the treatment of HIV infection, not JIA.

A 3-year-old boy presents to the Emergency Department after four episodes of emesis and three episodes of diarrhea starting this evening. He has had a few sips of water at home. His vital signs are reassuring except for mild tachycardia and he appears mildly dehydrated on examination. He is given a dose of ondansetron for nausea in the waiting room. What is the next best step in management? Administer a bolus of 0.9% sodium chloride Admit for maintenance intravenous fluids Discharge home with lactobacillus Trial of oral rehydration solution

Correct Answer ( D ) Explanation: The most likely diagnosis for a child with acute onset of vomiting and diarrhea is viral gastroenteritis. Food poisoning is also a consideration. Both are self-limited conditions and most affected children can be safely managed in the outpatient setting. The most important supportive intervention is maintenance of adequate intravascular volume. In cases of mild to moderate dehydration, oral rehydration therapy is preferred to intravenous hydration. In children with persistent emesis with attempted oral rehydration, anti-nausea medication may be effective in facilitating tolerance of oral intake. Ondansetron, a selective serotonin antagonist, has been shown to decrease the rate of hospitalization in children who receive a single dose in the emergency department. After taking the ondansetron, the boy can then proceed with a trial of oral rehydration solution, the mainstay of treatment for acute gastroenteritis. He should then be monitored for further emesis. If emesis persists despite anti-nausea medication, intravenous hydration may be required until adequate oral hydration can be assured.

A 13-year-old boy presents to the ED because his mother noticed a progressive left-sided limp over the previous three weeks. The patient states that he has pain in his knee. On exam, you elicit pain on internal rotation of the hip. Which of the following is the most likely diagnosis? Legg-Calvé-Perthes disease Osteosarcoma Septic arthritis Slipped capital femoral epiphysis

Correct Answer ( D ) Explanation: The patient has a slipped capital femoral epiphysis (SCFE). SCFE is a Salter-Harris type 1 fracture of the femoral capital epiphysis. The x-ray will show the classic posterior and inferior displacement of the head (capital) of the femur. SCFE occurs in boys twice as often as girls, is commonly seen in children >10 years of age, and classically presents in obese children as subacute hip pain with a limp. Some underlying metabolic disorders such as hypothyroidism, panhypopituitarism, and hypogonadism are also risk factors for development of SCFE. Legg-Calvé-Perthes disease (A) is avascular necrosis of the femoral head, usually seen in young school-aged children. Hip pain and limp are early signs; radiographs may be normal. It occurs bilaterally in 15% of cases. Malignancy (B) could also present with subacute symptoms. Most osteosarcomas arise as solitary lesions within the fastest growing areas of the long bones of children. The top three affected areas are the distal femur, the proximal tibia, and the proximal humerus, but virtually any bone can be involved. The most common presenting symptom of osteosarcoma is pain, particularly with activity.

A 16-year-old boy is brought to ED due to testicular pain. The pain started while he was running in physical education class. He denies any other symptoms. On exam you note that the scrotum is swollen, the left testis is tender and there is absence of cremasteric reflex. Which of the following is the most likely diagnosis? Epididymitis Hydrocoele Inguinal hernia Testicular torsion

Correct Answer ( D ) Explanation: The patient most likely has testicular pain due to testicular torsion. Testicular torsion is caused by inadequate fixation of the testis within the scrotum that results from a redundant tunica vaginalis, allowing excessive mobility of the testis. It requires prompt diagnosis and treatment to salvage the testis. Symptoms include pain that is sudden in onset, nausea and vomiting. It may be associated with exercise or minor genital trauma. On exam there is scrotal wall erythema, scrotal swelling, and absence of the cremasteric reflex. Evaluation is done with color Doppler ultrasonography to assess testicular blood flow and testicular morphologic features. Treatment is surgical exploration and detorsion.

A term 20-day-old infant presents to the ED from urgent care for an abnormal ECG obtained for fussiness. His mother had routine prenatal care and states he has not had fever, is gaining weight, eating well without sweating or cyanosis, and making normal wet and dirty diapers. At this time, his symptoms have resolved. Your exam is unremarkable and his vitals are within normal limits for age. The ECG is only remarkable for a rightward axis of +140°. What is the next best step in the management of this patient? Chest X-ray for heart size Emergent echocardiogram Outpatient holter monitor Reassurance and discharge home

Correct Answer ( D ) Explanation: This infant has no signs of congenital heart defects but has a rightward axis. Infants less than one month normally have a degree of right ventricular hypertrophy because in utero the newborn heart pumps systemic blood from both the left and the right ventricle (through the ductus arteriosus). The right ventricle is more dominant in utero, handling just over half of the combined cardiac output. As the pulmonary vascular resistance drops and less work is required of the right ventricle the left becomes more dominant over the first weeks to months of life. A normal QRS axis in patients less than one month of age can be as right as +160 to 180°, and as right as +115 to 125° at three months of age. A superior axis of 0 to - 180° is rarely seen in a child with a normal heart and should be investigated further. An infant with no signs of congenital heart disease and a normal ECG should be given reassurance and discharged home.

13-year-old boy presents to clinic with a worsening rash. The rash started on his right forearm and has progressed to involve the right upper arm and the left arm. It is not pruritic or painful. The rash has been present for about three weeks. He also complains of headache and fatigue. The patient returned from a camping trip in Pennsylvania one month ago. On exam, he has several erythematous macules with central clearing on the upper extremities and trunk. Which of the following associated findings is this patient most likely to have? Chronic encephalitis Complete heart block Monoarticular arthritis Peripheral facial nerve palsy

Correct Answer ( D ) Explanation: This patient has early disseminated Lyme disease as evidenced by the secondary erythema migrans lesions. Lyme disease is caused by transmission of Borrelia burgdorferi, a spirochete, via the bite of the tick Ixodes. Transmission is most common in the northeast United States. Lyme disease occurs in multiple stages. The initial stage is early localized disease occurring within the first month of exposure, characterized by erythema migrans at the site of the tick bite. Next, early disseminated disease may develop from release of the spirochetes into the bloodstream, three to twelve weeks following exposure. The classic findings in early disseminated disease are secondary erythema migrans lesions, constitutional symptoms, cranial neuropathy, meningitis, and carditis. The most common cranial neuropathy is facial nerve (VN VII) palsy. In endemic areas, Lyme disease is the leading cause of facial nerve palsy.

A 2-month-old girl is found to have positive Barlow and Ortolani signs. Which of the follow is the most appropriate diagnostic study? Computed tomography without contrast Magnetic resonance imaging Two-view plain radiographs Ultrasound

Correct Answer ( D ) Explanation: Ultrasound is the diagnostic test of choice for infants with suspected developmental dysplasia of the hip. Developmental dysplasia of the hip (DDH) is a spectrum of abnormal hip development conditions that encompass acetabular dysplasia to irreducible dislocation at birth. DDH is more common in female infants than in male infants. Other known risk factors include breech presentation, oligohydramnios, family history of DDH, and swaddling. DDH is the result of abnormal contact between the acetabulum and the femoral head. Swaddling and cradleboards place the hips in an adducted and extended position, which increases the incidence of DDH. Clinical screening for DDH is recommended in the first few days of life and at every well-infant and child exam until the child is walking normally. The Ortolani and Barlow maneuvers are the most important tools for evaluating hip stability during the first three months The Barlow maneuver involves flexing and adducting while applying posterior pressure through the hip. A palpable clunk indicates instability and a positive Barlow maneuver. The Ortolani test is used to reduce the hip and involves abducting the hip while using posterior pressure to lift the greater trochanter. The Galeazzi sign (uneven knee heights) and limited abduction are better indicators of DDH in children older than two to three months. Ultrasonography is the diagnostic study of choice for assessing hip instability in infants younger than six months. Orthopedic referral is indicated in all cases of suspected hip instability. Initial treatment often involves a soft abduction support device, such as the Pavlik harness. Prognosis is good if DDH is diagnosed early and treatment initiated. Ultrasonography is used to monitor treatment and determine how long the Pavlik harness is needed.

A 6-month-old infant presents to the pediatrician's office with a two-day history of increased fussiness, poor feeding, low-grade fever and rash. The family returned from a trip to India last week. The rash appears on the trunk in several stages of erythematous macules, vesicular papules and pustules. Which of the following is the most likely diagnosis? Herpes simplex Measles Pityriasis rosea Varicella

Correct Answer ( D ) Explanation: Varicella (chicken pox) is a highly contagious viral infection that is transmitted either through air-borne route or direct contact with the infected individual. The incubation period is 10-21 days. The infected individual is contagious for 2 days before the rash appears and until all the lesions have crusted over. There is a prodromal phase of malaise, headache, conjunctivitis, poor appetite and low-grade fever. The rash typically starts at the hairline, with formation of macules which progress to fluid-filled vesicles (dew drops on a rose pedal). Crops of lesions typically appear at the same time with vesicles in various stages of healing on the body. Infants have some immunity from maternal antibodies until 6 months of age. First dose of the varicella vaccine is given at 12 months of age. In healthy children < 12 years of age, chickenpox is usually a self-limited illness requiring only supportive care. Patients > 12 years of age are at higher risk for severe disease and should receive oral acyclovir. Immunocompromised patients should receive intravenous acyclovir.

Which of the following is the most common congenital heart defect? Atrial septal defect Coarctation of the aorta Patent ductus arteriosus Ventricular septal defect

Correct Answer ( D ) Explanation: Ventricular septal defect (VSD) is most common, responsible for 25% of all congenital heart defects. Most are small and close spontaneously over time. However, larger lesions may persist, eventually leading to pulmonary hypertension and right-sided heart failure. The classic VSD murmur is holosystolic and best heard at the left lower sternal border

A 3-day-old baby girl has a holosystolic murmur along the left lower sternal border. What is the most likely diagnosis? Aortic stenosis Atrial septal defect Patent ductus arteriosus Ventricular septal defect

Correct Answer ( D ) Explanation: Ventricular septal defects (VSD) present with a holosystolic murmur best heard at the left lower sternal border and is frequently accompanied by a thrill or displaced point of maximal impulse. A defect in the membranous (superior) portion of the ventricular septum is more common than in the muscular (inferior) portion. Auscultation is often sufficient to make the diagnosis of a VSD but confirmation can be obtained with echocardiography. The clinical findings of patients with a VSD vary according to the size of the defect and pulmonary blood flow and pressure. Small VSDs with trivial left to right shunts and normal pulmonary arterial pressure are the most common. These patients are asymptomatic, and the cardiac lesion is usually found during routine physical exam. A VSD is considered an acyanotic congential heart defect. However, if uncorrected, over time the left-to-right shunt can reverse to a right-to-left shunt (Eisenmeger phenomenon) and result in cyanosis. Smaller VSDs are associated with louder murmurs but larger defects may lead to tachypnea and sweating with feeds and failure to thrive. Most small VSDs close on their own while larger VSDs may require surgical intervention if there is associated aortic regurgitation, pulmonary hypertension, pulmonic stenosis or congestive cardiac failure resistant to medications. The murmur of aortic stenosis (A) is characterized by a harsh crescendo-decrescendo systolic murmur, heard loudest at the base of the heart and radiating to the neck. In an atrial septal defect (B), the first heart sound may be normal or split, there may be a midsystolic pulmonary ejection murmur at the upper left sternal border, and in older children, the second hart sound may have fixed splitting. A patent ductus arterosus (C) on auscultation presents as a continuous machine-like murmur.

A mother brings in her 16-month-old boy because she is concerned that he appears pale. Vital signs are within normal limits for age except for a mild tachycardia. He appears pale on physical exam, is irritable but consoles appropriately, and appears tired. You hear a systolic flow murmur and tachycardia on cardiac exam, but otherwise, physical exam is normal. Laboratory testing reveals the following: What is the best treatment for this child's anemia? Oral ferritin supplementation Oral iron supplementation Packed red blood cell transfusion Parenteral iron transfusion

orrect Answer ( B ) Explanation: This is a case of severe iron deficiency anemia of childhood. This most commonly occurs in children between the ages of one and two years old and is often found in children who drink cow's milk exclusively for their nutritional intake. Pallor is usually noted when a patient's hemoglobin drops below 8 mg/dL. ​Decreased hemoglobin production, increased erythrocyte destruction, or blood loss can cause anemias of childhood. The most common cause of decreased hemoglobin or erythrocyte production in children is nutritional iron deficiency anemia. The patient's dietary history is the key to this diagnosis, and that may only be elucidated by careful questioning. This is a chronic disease that rarely needs acute parenteral treatment or transfusion, with a gradual worsening in pallor and symptoms. Treatment with oral iron is often initiated in the ED before confirmatory blood testing has been obtained. Occasionally, patients present to the ED when they are challenged by a fever or other acute illness. The increase in cardiac output, in a child with severe anemia, may be enough to cause the patient to go into congestive heart failure. Treatment for any child with severe anemia is not withheld if the child is in extremis. Treatment of iron deficiency anemia, as long as the patient is not in extremis or showing signs of congestive heart failure is oral elemental iron in the form of ferrous sulfate, 3 to 6 mg/kg/day. The medication should be given at night, on an empty stomach, in a single dose. The patient will begin to improve within 72 hours of treatment initiation. As much as iron supplementation is important in the treatment of iron deficiency anemia, it is also critical to restructure the diet of the child to assure adequate nutritional intake. Because improvement occurs very quickly after treatment initiation, it can be difficult to maintain follow-up with parents who no longer perceive their child as ill.

7-year-old boy presents complaining of occasional intermittent lower abdominal pain over the last several weeks associated with a decline in the frequency of bowel movements to one every third day. When he does defecate, stools are hard to pass and sometimes painful. There is no blood in the stools. He has no chronic medical problems, has never had surgery, and takes no medications. Review of systems are negative and physical exam is normal. What is the most appropriate initial intervention for this child? Bisacodyl Phosphate enema Polyethylene glycol Referral to gastroenterologist Soap suds enema

orrect Answer ( C ) Explanation: Polyethylene glycol is an osmotic laxative, and reasonable first line oral medication to use for maintenance therapy of constipation, defined as a two week history or more of delay or difficulty in defecation. Other options include magnesium hydroxide, lactulose and sorbitol. The most common cause of constipation in children is functional constipation, meaning constipation without objective evidence of a pathological condition. Behavioral modification including unhurried time on the toilet after meals to discourage withholding of stool, as well as feeding children a diet of whole grains, fruits and vegetables is recommended as part of the treatment of constipation, though studies suggest that medication is often necessary to achieve regular bowel movements. Fever, abdominal distension, anorexia, nausea, vomiting, weight loss, or poor weight gain suggest a more serious etiology and should prompt further work-up with imaging and laboratory studies.

A 6-year-old girl presents with 4 days of lower abdominal pain. The patient complains of dysuria. On exam, the patient is afebrile and has mild tenderness to palpation in the suprapubic area. No costovertebral tenderness is elicited on exam. A clean-catch urine sample is sent for urinalysis. If positive, which of the following is the most specific to confirm the diagnosis? Glucose Leukocyte esterase Nitrites WBCs (>5 per high power field)

orrect Answer ( C ) Explanation: The patient's presentation is consistent with an uncomplicated urinary tract infection (UTI). The most common cause of a UTI in children >1 year of age is E. coli. Nitrites normally are not found in urine but result when bacteria reduce urinary nitrates to nitrites. Many Gram-negative and some Gram-positive organisms are capable of this conversion, and a positive dipstick nitrite test indicates that these organisms are present in significant numbers (i.e., more than 10,000 per mL). This test is specific (92%-100%) but not highly sensitive (19%-48%). A positive result is helpful, but a negative result does not rule out UTI. The nitrite dipstick reagent is sensitive to air exposure, so containers should be closed immediately after removing a strip. After 1 week of exposure, 33% of strips give false-positive results, and after 2 weeks, 75% give false-positive results. Non-nitrate-reducing organisms also may cause false-negative results, and patients who consume a low-nitrate diet may have false-negative results. Leukocyte esterase (B) is produced by neutrophils and may signal pyuria associated with UTI. It has a sensitivity of 72%-97% and specificity of 41%-86%. Leukocyte casts in the urinary sediment can help localize the area of inflammation to the kidney. Organisms such as Chlamydia and Ureaplasma urealyticum should be considered in patients with pyuria and negative cultures. Other causes of sterile pyuria include balanitis, urethritis, tuberculosis, bladder tumors, viral infections, nephrolithiasis, foreign bodies, exercise, glomerulonephritis, and corticosteroid and cyclophosphamide use. Leukocytes (D) may be seen under low- and high-power magnification. Men normally have fewer than 2 white blood cells (WBCs) per HPF; women normally have fewer than 5 WBCs per HPF; >5 WBCs/HPF is associated with a 90%-96% sensitivity and 47%-50% specificity.


Ensembles d'études connexes

Unit 5 - Teamwork Business Result

View Set

PrepU: Chapter 24: Nursing Management of the Newborn at Risk: Acquired and Congenital Newborn Conditions, PrepU: Chapter 23: Nursing Care of the Newborn with Special Needs, Chapter 22: Nursing Management of the Postpartum Woman at Risk (Prep U), Chap...

View Set

System Administration and IT Infrastructure Services

View Set

Home Work Chapter 7 (Principles of Microeconomics)

View Set

Ch 15 Traveling Waves and Sounds

View Set